MS Exam 1 Flashcards

1
Q

After assessment of a patient with pneumonia, the nurse identifies a nursing diagnosis of ineffective airway clearance. Which assessment data best supports this diagnosis?

a. Weak cough effort
b. Profuse green sputum
c. Respiratory rate of 28 breaths/minute
d. Resting pulse oximetry (SpO2) of 85%

A

ANS: A
The weak, nonproductive cough indicates that the patient is unable to clear the airway effectively. The other data would be used to support diagnoses such as impaired gas exchange and ineffective breathing pattern.

How well did you know this?
1
Not at all
2
3
4
5
Perfectly
2
Q

The nurse assesses the chest of a patient with pneumococcal pneumonia. Which finding would the nurse expect?

a. Increased tactile fremitus c. Hyperresonance to percussion
b. Dry, nonproductive cough d. A grating sound on auscultation

A

ANS: A
Increased tactile fremitus over the area of pulmonary consolidation is expected with bacterial pneumonias. Dullness to percussion would be expected. Pneumococcal pneumonia typically presents with a loose, productive cough. Adventitious breath sounds such as crackles and wheezes are typical. A grating sound is more representative of a pleural friction rub rather than pneumonia.

How well did you know this?
1
Not at all
2
3
4
5
Perfectly
3
Q

A patient with bacterial pneumonia has coarse crackles and thick sputum. Which action should the nurse plan to promote airway clearance?

a. Restrict oral fluids during the day.
b. Teach pursed-lip breathing technique.
c. Assist the patient to splint the chest when coughing.
d. Encourage the patient to wear the nasal O2 cannula.

A

ANS: C
Coughing is less painful and more likely to be effective when the patient splints the chest during coughing. Fluids should be encouraged to help liquefy secretions. Nasal O2 will improve gas exchange, but will not improve airway clearance. Pursed-lip breathing is used to improve gas exchange in patients with chronic obstructive pulmonary disease but will not improve airway clearance.

How well did you know this?
1
Not at all
2
3
4
5
Perfectly
4
Q

The nurse provides discharge instructions to a patient who was hospitalized for pneumonia. Which statement, if made by the patient, indicates a good understanding of the instructions?

a. “I will call my health care provider if I still feel tired after a week.”
b. “I will continue to do deep breathing and coughing exercises at home.”
c. “I will schedule two appointments for the pneumonia and influenza vaccines.”
d. “I will cancel my follow-up chest x-ray appointment if I feel better next week.”

A

ANS: B
Patients should continue to cough and deep breathe after discharge. Fatigue is expected for several weeks. The pneumococcal and influenza vaccines can be given at the same time in different arms. A follow-up chest x-ray needs to be done in 6 to 8 weeks to evaluate resolution of pneumonia.

How well did you know this?
1
Not at all
2
3
4
5
Perfectly
5
Q

Which action should the nurse plan to prevent aspiration in a high-risk patient?

a. Turn and reposition an immobile patient at least every 2 hours.
b. Place a patient with altered consciousness in a side-lying position.
c. Insert a nasogastric tube for feeding a patient with high calorie needs.
d. Monitor respiratory symptoms in a patient who is immunosuppressed.

A

ANS: B
With loss of consciousness, the gag and cough reflexes are depressed, and aspiration is more likely to occur. The risk for aspiration is decreased when patients with a decreased level of consciousness are placed in a side-lying or upright position. Frequent turning prevents pooling of secretions in immobilized patients but will not decrease the risk for aspiration in patients at risk. Monitoring of parameters such as breath sounds and O2 saturation will help detect pneumonia in immunocompromised patients, but it will not decrease the risk for aspiration. Conditions that increase the risk of aspiration include decreased level of consciousness (e.g., seizure, anesthesia, head injury, stroke, alcohol intake), difficulty swallowing, and nasogastric intubation with or without tube feeding.

How well did you know this?
1
Not at all
2
3
4
5
Perfectly
6
Q

A patient with right lower-lobe pneumonia has been treated with IV antibiotics for 3 days. Which assessment data obtained by the nurse indicates that the treatment is effective?

a. Bronchial breath sounds are heard at the right base.
b. The patient coughs up small amounts of green mucus.
c. The patient’s white blood cell (WBC) count is 9000/µL.
d. Increased tactile fremitus is palpable over the right chest.

A

ANS: C
The normal WBC count indicates that the antibiotics have been effective. All the other data suggest that a change in treatment is needed.

How well did you know this?
1
Not at all
2
3
4
5
Perfectly
7
Q

The health care provider writes an order for bacteriologic testing for a patient who has a positive tuberculosis skin test. Which action should the nurse take?

a. Teach about the reason for the blood tests.
b. Schedule an appointment for a chest x-ray.
c. Teach the patient about providing specimens for 3 consecutive days.
d. Instruct the patient to collect several separate sputum specimens today.

A

ANS: C
Sputum specimens are obtained on 2 to 3 consecutive days for bacteriologic testing for Mycobacterium tuberculosis. The patient should not provide all the specimens at once. Blood cultures are not used for tuberculosis testing. A chest x-ray is not bacteriologic testing. Although the findings on chest x-ray examination are important, it is not possible to make a diagnosis of TB solely based on chest x-ray findings because other diseases can mimic the appearance of TB.

How well did you know this?
1
Not at all
2
3
4
5
Perfectly
8
Q

A patient is admitted with active tuberculosis (TB). The nurse should question a health care provider’s order to discontinue airborne precautions unless which assessment finding is documented?

a. Chest x-ray shows no upper lobe infiltrates.
b. TB medications have been taken for 6 months.
c. Mantoux testing shows an induration of 10 mm.
d. Sputum smears for acid-fast bacilli are negative.

A

ANS: D
Repeated negative sputum smears indicate that Mycobacterium tuberculosis is not present in the sputum, and the patient cannot transmit the bacteria by the airborne route. Chest x-rays are not used to determine whether treatment has been successful. Taking medications for 6 months is necessary, but the multidrug-resistant forms of the disease might not be eradicated after 6 months of therapy. Repeat Mantoux testing would not be done because the result will not change even with effective treatment.

How well did you know this?
1
Not at all
2
3
4
5
Perfectly
9
Q

The nurse teaches a patient about the transmission of pulmonary tuberculosis (TB). Which statement, if made by the patient, indicates that teaching was effective?

a. “I will take the bus instead of driving.”
b. “I will stay indoors whenever possible.”
c. “My spouse will sleep in another room.”
d. “I will keep the windows closed at home.”

A

ANS: C
Teach the patient how to minimize exposure to close contacts and household members. Homes should be well ventilated, especially the areas where the infected person spends a lot of time. While still infectious, the patient should sleep alone, spend as much time as possible outdoors, and minimize time in congregate settings or on public transportation.

How well did you know this?
1
Not at all
2
3
4
5
Perfectly
10
Q

A patient who is taking rifampin (Rifadin) for tuberculosis calls the clinic and reports having orange discolored urine and tears. Which response by the nurse reflects accurate knowledge about the medication and the patient’s illness?

a. Ask the patient about any visual changes in red-green color discrimination.
b. Question the patient about experiencing shortness of breath, hives, or itching.
c. Explain that orange discolored urine and tears are normal while taking this medication.
d. Advise the patient to stop the drug and report the symptoms to the health care provider.

A

ANS: C
Orange-colored body secretions are a side effect of rifampin. The patient does not have to stop taking the medication. The findings are not indicative of an allergic reaction. Alterations in red-green color discrimination commonly occurs when taking ethambutol, which is a different tuberculosis medication.

How well did you know this?
1
Not at all
2
3
4
5
Perfectly
11
Q

An older patient is receiving standard multidrug therapy for tuberculosis (TB). The nurse should notify the health care provider if the patient exhibits which finding?

a. Yellow-tinged sclera
b. Orange-colored sputum
c. Thickening of the fingernails
d. Difficulty hearing high-pitched voices

A

ANS: A
Noninfectious hepatitis is a toxic effect of isoniazid, rifampin, and pyrazinamide, and patients who develop hepatotoxicity will need to use other medications. Changes in hearing and nail thickening are not expected with the four medications used for initial TB drug therapy. Presbycusis is an expected finding in the older adult patient. Orange discoloration of body fluids is an expected side effect of rifampin and not an indication to call the health care provider.

How well did you know this?
1
Not at all
2
3
4
5
Perfectly
12
Q

A patient diagnosed with active tuberculosis (TB) is homeless and has a history of alcohol abuse. Which intervention by the nurse will be most effective in ensuring adherence with the treatment regimen?

a. Repeat warnings about the high risk for infecting others several times.
b. Give the patient written instructions about how to take the medications.
c. Arrange for a daily meal and drug administration at a community center.
d. Arrange for the patient’s friend to administer the medication on schedule.

A

ANS: C
Directly observed therapy is the most effective means for ensuring compliance with the treatment regimen, and arranging a daily meal will help ensure that the patient is available to receive the medication. The other nursing interventions may be appropriate for some patients but are not likely to be as helpful for this patient’s situation.

How well did you know this?
1
Not at all
2
3
4
5
Perfectly
13
Q

After 2 months of tuberculosis (TB) treatment with isoniazid, rifampin (Rifadin), pyrazinamide, and ethambutol, a patient continues to have positive sputum smears for acid-fast bacilli (AFB). Which action should the nurse take next?

a. Teach about drug-resistant TB.
b. Schedule directly observed therapy.
c. Ask the patient whether medications have been taken as directed.
d. Discuss the need for an injectable antibiotic with the health care provider.

A

ANS: C
The first action should be to determine whether the patient has been compliant with drug therapy because negative sputum smears would be expected if the TB bacillus is susceptible to the medications and if the medications have been taken correctly. Assessment is the first step in the nursing process. Depending on whether the patient has been compliant or not, different medications or directly observed therapy may be indicated. The other options are interventions based on assumptions until an assessment has been completed.

How well did you know this?
1
Not at all
2
3
4
5
Perfectly
14
Q

Employee health test results reveal a tuberculosis (TB) skin test of 16-mm induration and a negative chest x-ray for a staff nurse working on the pulmonary unit. The nurse has no symptoms of TB. Which information should the occupational health nurse plan to teach the staff nurse?

a. Use and side effects of isoniazid
b. Standard four-drug therapy for TB
c. Need for annual repeat TB skin testing
d. Bacille Calmette-Guérin (BCG) vaccine

A

ANS: A
The nurse is considered to have a latent TB infection and should be treated with INH daily for 6 to 9 months. The four-drug therapy would be appropriate if the nurse had active TB. TB skin testing is not done for individuals who have already had a positive skin test result. BCG vaccine is not used in the United States for TB and would not be helpful for this individual, who already has a TB infection.

How well did you know this?
1
Not at all
2
3
4
5
Perfectly
15
Q

The nurse supervises a student nurse who is assigned to take care of a patient with active tuberculosis (TB). Which action, if performed by the student nurse, would require an intervention by the nurse?

a. The patient is offered a tissue from the box at the bedside.
b. A surgical face mask is applied before visiting the patient.
c. A snack is brought to the patient from the unit refrigerator.
d. Hand washing is performed before entering the patient’s room.

A

ANS: B
A high-efficiency particulate-absorbing (HEPA) mask, rather than a standard surgical mask, should be used when entering the patient’s room because the HEPA mask can filter out 100% of small airborne particles. Hand washing before entering the patient’s room is appropriate. Because anorexia and weight loss are frequent problems in patients with TB, bringing food to the patient is appropriate. The student nurse should perform hand washing after handling a tissue that the patient has used, but no precautions are necessary when giving the patient an unused tissue.

How well did you know this?
1
Not at all
2
3
4
5
Perfectly
16
Q

An occupational health nurse works at a manufacturing plant where there is potential exposure to inhaled dust. Which action recommended by the nurse is intended to prevent lung disease?

a. Treat workers with pulmonary fibrosis.
b. Teach about symptoms of lung disease.
c. Require the use of protective equipment.
d. Monitor workers for coughing and wheezing.

A

ANS: C
Prevention of lung disease requires the use of appropriate protective equipment such as masks. The other actions will help in recognition or early treatment of lung disease but will not be effective in prevention of lung damage. Repeated exposure eventually results in diffuse pulmonary fibrosis. Fibrosis is the result of tissue repair after inflammation.

How well did you know this?
1
Not at all
2
3
4
5
Perfectly
17
Q

Which information about prevention of lung disease should the nurse include for a patient with a 42 pack-year history of cigarette smoking?

a. Resources for support in smoking cessation
b. Reasons for annual sputum cytology testing
c. Erlotinib (Tarceva) therapy to prevent tumor risk
d. Computed tomography (CT) screening for cancer

A

ANS: A
Because smoking is the major cause of lung cancer, the most important role for the nurse is teaching patients about the benefits of and means of smoking cessation. CT scanning is currently being investigated as a screening test for high-risk patients. However, if there is a positive finding, the person already has lung cancer. Sputum cytology is a diagnostic test, but does not prevent cancer or disease. Erlotinib may be used in patients who have lung cancer, but it is not used to reduce the risk of developing cancer.

How well did you know this?
1
Not at all
2
3
4
5
Perfectly
18
Q

A lobectomy is scheduled for a patient with stage I non–small cell lung cancer. The patient tells the nurse, “I would rather have chemotherapy than surgery.” Which response by the nurse is most appropriate?

a. “Are you afraid that the surgery will be very painful?”
b. “Did you have bad experiences with previous surgeries?”
c. “Tell me what you know about the treatments available.”
d. “Surgery is the treatment of choice for stage I lung cancer.”

A

ANS: C
More assessment of the patient’s concerns about surgery is indicated. An open-ended response will elicit the most information from the patient. The answer beginning, “Surgery is the treatment of choice” is accurate, but it discourages the patient from sharing concerns about surgery. The remaining two answers indicate that the nurse has jumped to conclusions about the patient’s reasons for not wanting surgery. Chemotherapy is the primary treatment for small cell lung cancer. In non–small cell lung cancer, chemotherapy may be used in the treatment of nonresectable tumors or as adjuvant therapy to surgery.

How well did you know this?
1
Not at all
2
3
4
5
Perfectly
19
Q

An hour after a thoracotomy, a patient complains of incisional pain at a level 7 (based on 0 to 10 scale) and has decreased left-sided breath sounds. The pleural drainage system has 100 mL of bloody drainage and a large air leak. Which action should the nurse take?

a. Clamp the chest tube in two places.
b. Administer the prescribed morphine.
c. Milk the chest tube to remove any clots.
d. Assist the patient with incentive spirometry.

A

ANS: B
Treat the pain. The patient is unlikely to take deep breaths or cough until the pain level is lower. A chest tube output of 100 mL is not unusual in the first hour after thoracotomy. Milking or stripping chest tubes is no longer recommended because these practices can dangerously increase intrapleural pressures and damage lung tissues. Position tubing so that drainage flows freely to negate need for milking or stripping. An air leak is expected in the initial postoperative period after thoracotomy. Clamping the chest tube is not indicated and may lead to dangerous development of a tension pneumothorax.

How well did you know this?
1
Not at all
2
3
4
5
Perfectly
20
Q

A patient with newly diagnosed lung cancer tells the nurse, “I don’t think I’m going to live to see my next birthday.” Which is the best initial response by the nurse?

a. “Are you ready to talk with your family members about dying now?”
b. “Would you like to talk to the hospital chaplain about your feelings?”
c. “Can you tell me what it is that makes you think you will die so soon?”
d. “Do you think that taking an antidepressant medication would be helpful?”

A

ANS: C
The nurse’s initial response should be to collect more assessment data about the patient’s statement. The answer beginning “Can you tell me what it is” is the most open-ended question and will offer the best opportunity for obtaining more data. The remaining answers offer interventions that may be helpful to the patient, but more assessment is needed to determine whether these interventions are appropriate.

How well did you know this?
1
Not at all
2
3
4
5
Perfectly
21
Q

The nurse monitors a patient in the emergency department after chest tube placement for a hemopneumothorax. The nurse is most concerned if which assessment finding is observed?

a. A large air leak in the water-seal chamber
b. 400 mL of blood in the collection chamber
c. Complaint of pain with each deep inspiration
d. Subcutaneous emphysema at the insertion site

A

ANS: B
The large amount of blood may indicate that the patient is in danger of developing hypovolemic shock. An air leak would be expected immediately after chest tube placement for a pneumothorax. Initially, brisk bubbling of air occurs in this chamber when a pneumothorax is evacuated. The pain should be treated but is not as urgent a concern as the possibility of continued hemorrhage. Subcutaneous emphysema should be monitored but is not unusual in a patient with pneumothorax. A small amount of subcutaneous air is harmless and will be reabsorbed.

How well did you know this?
1
Not at all
2
3
4
5
Perfectly
22
Q

A patient experiences a chest wall contusion as a result of being struck in the chest with a baseball bat. The emergency department nurse would be most concerned if which finding is observed during the initial assessment?

a. Paradoxical chest movement c. Heart rate of 110 beats/minute
b. Complaint of chest wall pain d. Large bruised area on the chest

A

ANS: A
Paradoxical chest movement indicates that the patient may have flail chest, which can severely compromise gas exchange and can rapidly lead to hypoxemia. Chest wall pain, a slightly elevated pulse rate, and chest bruising all require further assessment or intervention, but the priority concern is poor gas exchange

How well did you know this?
1
Not at all
2
3
4
5
Perfectly
23
Q

When assessing a patient who has just arrived after an automobile accident, the emergency department nurse notes tachycardia and absent breath sounds over the right lung. For which intervention will the nurse prepare the patient?

a. Emergency pericardiocentesis c. Bronchodilator administration
b. Stabilization of the chest wall d. Chest tube connected to suction

A

ANS: D
The patient’s history and absent breath sounds suggest a right-sided pneumothorax or hemothorax, which will require treatment with a chest tube and drainage to suction. The other therapies would be appropriate for an acute asthma attack, flail chest, or cardiac tamponade, but the patient’s clinical manifestations are not consistent with these problems.

How well did you know this?
1
Not at all
2
3
4
5
Perfectly
24
Q

A patient who has a right-sided chest tube after a thoracotomy has continuous bubbling in the suction-control chamber of the collection device. Which action by the nurse is appropriate?

a. Adjust the dial on the wall regulator.
b. Continue to monitor the collection device.
c. Document the presence of a large air leak.
d. Notify the surgeon of a possible pneumothorax.

A

ANS: B
Continuous bubbling is expected in the suction-control chamber and indicates that the suction-control chamber is connected to suction. An air leak would be detected in the water-seal chamber. There is no evidence of pneumothorax. Increasing or decreasing the vacuum source will not adjust the suction pressure. The amount of suction applied is regulated by the amount of water in this chamber and not by the amount of suction applied to the system.

How well did you know this?
1
Not at all
2
3
4
5
Perfectly
25
Q

The nurse provides preoperative instruction for a patient scheduled for a left pneumonectomy. Which information should the nurse include about the patient’s postoperative care?

a. Bed rest for the first 24 hours
b. Positioning only on the right side
c. Frequent use of an incentive spirometer
d. Chest tube placement to continuous suction

A

ANS: C
Frequent deep breathing and coughing are needed after chest surgery to prevent atelectasis. To promote gas exchange, patients after pneumonectomy are positioned on the surgical side. Early mobilization decreases the risk for postoperative complications such as pneumonia and deep vein thrombosis. In a pneumonectomy, chest tubes may or may not be placed in the space from which the lung was removed. If a chest tube is used, it is clamped and only released by the surgeon to adjust the volume of serosanguineous fluid that will fill the space vacated by the lung. If the cavity overfills, it could compress the remaining lung and compromise the cardiovascular and pulmonary function. Daily chest x-rays can be used to assess the volume and space.

How well did you know this?
1
Not at all
2
3
4
5
Perfectly
26
Q

The nurse administers prescribed therapies for a patient with cor pulmonale and right-sided heart failure. Which assessment could be used to evaluate the effectiveness of the therapies?

a. Observe for distended neck veins.
b. Auscultate for crackles in the lungs.
c. Palpate for heaves or thrills over the heart.
d. Monitor for elevated white blood cell count.

A

ANS: A
Cor pulmonale is right ventricular failure caused by pulmonary hypertension, so clinical manifestations of right ventricular failure such as peripheral edema, jugular venous distention, and right upper-quadrant abdominal tenderness would be expected. Crackles in the lungs are likely to be heard with left-sided heart failure. Findings in cor pulmonale include evidence of right ventricular hypertrophy on electrocardiography and an increase in intensity of the second heart sound. Heaves or thrills are not common with cor pulmonale. White blood count elevation might indicate infection but is not expected with cor pulmonale.

How well did you know this?
1
Not at all
2
3
4
5
Perfectly
27
Q

A patient with idiopathic pulmonary arterial hypertension (IPAH) is receiving nifedipine (Procardia). Which assessment would best indicate to the nurse that the patient’s condition is improving?

a. Patient’s chest x-ray indicates clear lung fields.
b. Heart rate is between 60 and 100 beats/minute.
c. Patient reports a decrease in exertional dyspnea.
d. Blood pressure (BP) is less than 140/90 mm Hg.

A

ANS: C
Because a major symptom of IPAH is exertional dyspnea, an improvement in this symptom would indicate that the medication was effective. Nifedipine will affect BP and heart rate, but these parameters would not be used to monitor the effectiveness of therapy for a patient with IPAH. The chest x-ray will show clear lung fields even if the therapy is not effective.

How well did you know this?
1
Not at all
2
3
4
5
Perfectly
28
Q

A patient with a pleural effusion is scheduled for a thoracentesis. Which action should the nurse take to prepare the patient for the procedure?

a. Start a peripheral IV line to administer sedatives.
b. Position the patient sitting up on the side of the bed.
c. Obtain a collection device to hold 3 liters of pleural fluid.
d. Remind the patient not to eat or drink anything for 6 hours.

A

ANS: B
When the patient is sitting up, fluid accumulates in the pleural space at the lung bases and can more easily be located and removed. The patient does not usually require sedation for the procedure, and there are no restrictions on oral intake because the patient is not sedated or unconscious. Usually only 1000 to 1200 mL of pleural fluid is removed at one time. Rapid removal of a large volume can result in hypotension, hypoxemia, or pulmonary edema.

How well did you know this?
1
Not at all
2
3
4
5
Perfectly
29
Q

The nurse completes discharge teaching for a patient who has had a lung transplant. Which patient statement indicates to the nurse that the teaching has been effective?

a. “I will make an appointment to see the doctor every year.”
b. “I will stop taking the prednisone if I experience a dry cough.”
c. “I will not worry if I feel a little short of breath with exercise.”
d. “I will call the health care provider right away if I develop a fever.”

A

ANS: D
Low-grade fever may indicate infection or acute rejection so the patient should notify the health care provider immediately if the temperature is elevated. Patients require frequent follow-up visits with the transplant team. Annual health care provider visits would not be sufficient. Home O2 use is not an expectation after lung transplant. Shortness of breath should be reported. Low-grade fever, fatigue, dyspnea, dry cough, and O2 desaturation are signs of rejection. Immunosuppressive therapy, including prednisone, needs to be continued to prevent rejection.

How well did you know this?
1
Not at all
2
3
4
5
Perfectly
30
Q

A patient has just been admitted with probable bacterial pneumonia and sepsis. Which order should the nurse implement first?

a. Chest x-ray via stretcher
b. Blood cultures from two sites
c. Ciprofloxacin (Cipro) 400 mg IV
d. Acetaminophen (Tylenol) rectal suppository

A

ANS: B
Initiating antibiotic therapy rapidly is essential, but it is important that the cultures be obtained before antibiotic administration. The chest x-ray and acetaminophen administration can be done last.

How well did you know this?
1
Not at all
2
3
4
5
Perfectly
31
Q

The nurse is caring for a patient who has just had a thoracentesis. Which assessment information obtained by the nurse is a priority to communicate to the health care provider?

a. O2 saturation is 88%.
b. Blood pressure is 155/90 mm Hg.
c. Pain level is 5 (on 0 to 10 scale) with a deep breath.
d. Respiratory rate is 24 breaths/minute when lying flat.

A

ANS: A
O2 saturation would be expected to improve after a thoracentesis. A saturation of 88% indicates that a complication such as pneumothorax may be occurring. The other assessment data also indicate a need for ongoing assessment or intervention, but the low O2 saturation is the priority.

How well did you know this?
1
Not at all
2
3
4
5
Perfectly
32
Q

A patient who has just been admitted with community-acquired pneumococcal pneumonia has a temperature of 101.6° F with a frequent cough and is complaining of severe pleuritic chest pain. Which prescribed medication should the nurse give first?

a. Codeine c. Acetaminophen (Tylenol)
b. Guaifenesin d. Piperacillin/tazobactam (Zosyn)

A

ANS: D
Early initiation of antibiotic therapy has been demonstrated to reduce mortality. The other medications are also appropriate and should be given as soon as possible, but the priority is to start antibiotic therapy.

How well did you know this?
1
Not at all
2
3
4
5
Perfectly
33
Q

A patient is diagnosed with both human immunodeficiency virus (HIV) and active tuberculosis (TB) disease. Which information obtained by the nurse is most important to communicate to the health care provider?

a. The Mantoux test had an induration of 7 mm.
b. The chest-x-ray showed infiltrates in the lower lobes.
c. The patient has a cough that is productive of blood-tinged mucus.
d. The patient is being treated with antiretrovirals for HIV infection.

A

ANS: D
Drug interactions can occur between the antiretrovirals used to treat HIV infection and the medications used to treat TB. The other data are expected in a patient with HIV and TB.

How well did you know this?
1
Not at all
2
3
4
5
Perfectly
34
Q

A patient with pneumonia has a fever of 101.4° F (38.6° C), a nonproductive cough, and an O2 saturation of 88%. The patient complains of weakness, fatigue, and needs assistance to get out of bed. Which nursing diagnosis should the nurse assign as the priority?

a. Hyperthermia related to infectious illness
b. Impaired transfer ability related to weakness
c. Ineffective airway clearance related to thick secretions
d. Impaired gas exchange related to respiratory congestion

A

ANS: D
All of these nursing diagnoses are appropriate for the patient, but the patient’s O2 saturation indicates that all body tissues are at risk for hypoxia unless the gas exchange is improved.

How well did you know this?
1
Not at all
2
3
4
5
Perfectly
35
Q

The nurse supervises unlicensed assistive personnel (UAP) who are providing care for a patient with right lower lobe pneumonia. The nurse should intervene if which action by UAP is observed?

a. UAP assist the patient to ambulate to the bathroom.
b. UAP help splint the patient’s chest during coughing.
c. UAP transfer the patient to a bedside chair for meals.
d. UAP lower the head of the patient’s bed to 15 degrees.

A

ANS: D
Positioning the patient with the head of the bed lowered will decrease ventilation. The other actions are appropriate for a patient with pneumonia.

How well did you know this?
1
Not at all
2
3
4
5
Perfectly
36
Q

A patient with a possible pulmonary embolism complains of chest pain and difficulty breathing. The nurse finds a heart rate of 142 beats/min, blood pressure of 100/60 mm Hg, and respirations of 42 breaths/min. Which action should the nurse take first?

a. Administer anticoagulant drug therapy.
b. Notify the patient’s health care provider.
c. Prepare patient for a spiral computed tomography (CT).
d. Elevate the head of the bed to a semi-Fowler’s position.

A

ANS: D
The patient has symptoms consistent with a pulmonary embolism (PE). Elevating the head of the bed will improve ventilation and gas exchange. The other actions can be accomplished after the head is elevated (and O2 is started). A spiral CT may be ordered by the health care provider to identify PE. Anticoagulants may be ordered after confirmation of the diagnosis of PE.

How well did you know this?
1
Not at all
2
3
4
5
Perfectly
37
Q

The nurse receives change-of-shift report on the following four patients. Which patient should the nurse assess first?

a. A 23-yr-old patient with cystic fibrosis who has pulmonary function testing scheduled
b. A 46-yr-old patient on bed rest who is complaining of sudden onset of shortness of breath
c. A 77-yr-old patient with tuberculosis (TB) who has four medications due in 15 minutes
d. A 35-yr-old patient who was admitted with pneumonia and has a temperature of 100.2° F (37.8° C)

A

ANS: B
Patients on bed rest who are immobile are at high risk for deep vein thrombosis (DVT). Sudden onset of shortness of breath in a patient with a DVT suggests a pulmonary embolism and requires immediate assessment and action such as O2 administration. The other patients should also be assessed as soon as possible, but there is no indication that they may need immediate action to prevent clinical deterioration.

How well did you know this?
1
Not at all
2
3
4
5
Perfectly
38
Q

The nurse is performing tuberculosis (TB) skin tests in a clinic that has many patients who have immigrated to the United States. Which question is most important for the nurse to ask before the skin test?

a. “Do you take any over-the-counter (OTC) medications?”
b. “Do you have any family members with a history of TB?”
c. “How long has it been since you moved to the United States?”
d. “Did you receive the bacille Calmette-Guérin (BCG) vaccine for TB?”

A

ANS: D
Patients who have received the BCG vaccine will have a positive Mantoux test. Another method for screening (e.g., chest x-ray) will need to be used in determining whether the patient has a TB infection. The other information also may be valuable but is not as pertinent to the decision about doing TB skin testing.

How well did you know this?
1
Not at all
2
3
4
5
Perfectly
39
Q

A patient is admitted to the emergency department with an open stab wound to the left chest. What action should the nurse take?

a. Keep the head of the patient’s bed positioned flat.
b. Cover the wound tightly with an occlusive dressing.
c. Position the patient so that the left chest is dependent.
d. Tape a nonporous dressing on three sides over the wound.

A

ANS: D
The dressing taped on three sides will allow air to escape when intrapleural pressure increases during expiration, but it will prevent air from moving into the pleural space during inspiration. Placing the patient on the left side or covering the chest wound with an occlusive dressing will allow trapped air in the pleural space and cause tension pneumothorax. The head of the bed should be elevated to 30 to 45 degrees to facilitate breathing.

How well did you know this?
1
Not at all
2
3
4
5
Perfectly
40
Q

The nurse notes that a patient has incisional pain, a poor cough effort, and scattered coarse crackles after a thoracotomy. Which action should the nurse take first?

a. Assist the patient to sit upright in a chair.
b. Splint the patient’s chest during coughing.
c. Medicate the patient with prescribed morphine.
d. Observe the patient use the incentive spirometer.

A

ANS: C
A major reason for atelectasis and poor airway clearance in patients after chest surgery is incisional pain (which increases with deep breathing and coughing). The first action by the nurse should be to medicate the patient to minimize incisional pain. The other actions are all appropriate ways to improve airway clearance, but should be done after the morphine is given.

How well did you know this?
1
Not at all
2
3
4
5
Perfectly
41
Q

The nurse is caring for a patient with idiopathic pulmonary arterial hypertension (IPAH). Which assessment information requires the most immediate action by the nurse?

a. The O2 saturation is 90%.
b. The blood pressure is 98/56 mm Hg.
c. The epoprostenol (Flolan) infusion is disconnected.
d. The international normalized ratio (INR) is prolonged.

A

ANS: C
The half-life of this drug is 6 minutes, so the nurse will need to restart the infusion as soon as possible to prevent rapid clinical deterioration. The other data also indicate a need for ongoing monitoring or intervention, but the priority action is to reconnect the infusion.

How well did you know this?
1
Not at all
2
3
4
5
Perfectly
42
Q

A patient who was admitted the previous day with pneumonia complains of a sharp pain of 7 (on 0 to 10 scale) “whenever I take a deep breath.” Which action will the nurse take next?

a. Auscultate for breath sounds.
b. Administer the PRN morphine.
c. Have the patient cough forcefully.
d. Notify the patient’s health care provider.

A

ANS: A
The patient’s statement indicates that pleurisy or a pleural effusion may have developed and the nurse will need to listen for a pleural friction rub and decreased breath sounds. Assessment should occur before administration of pain medications. The patient is unlikely to be able to cough forcefully until pain medication has been administered. The nurse will want to obtain more assessment data before calling the health care provider.

How well did you know this?
1
Not at all
2
3
4
5
Perfectly
43
Q

A patient has acute bronchitis with a nonproductive cough and wheezes. Which topic should the nurse plan to include in the teaching plan?

a. Purpose of antibiotic therapy
b. Ways to limit oral fluid intake
c. Appropriate use of cough suppressants
d. Safety concerns with home O2 therapy

A

ANS: C
Cough suppressants are frequently prescribed for acute bronchitis. Because most acute bronchitis is viral in origin, antibiotics are not prescribed unless there are systemic symptoms. Fluid intake is encouraged. Home O2 is not prescribed for acute bronchitis, although it may be used for chronic bronchitis.

How well did you know this?
1
Not at all
2
3
4
5
Perfectly
44
Q

Which action by the nurse will be most effective in decreasing the spread of pertussis in a community setting?

a. Providing supportive care to patients diagnosed with pertussis
b. Teaching family members about the need for careful hand washing
c. Teaching patients about the need for adult pertussis immunizations
d. Encouraging patients to complete the prescribed course of antibiotics

A

ANS: C
The increased rate of pertussis in adults is thought to be caused by decreasing immunity after childhood immunization. Immunization is the most effective method of protecting communities from infectious diseases. Hand washing should be taught, but pertussis is spread by droplets and contact with secretions. Supportive care does not shorten the course of the disease or the risk for transmission. Taking antibiotics as prescribed does assist with decreased transmission, but patients are likely to have already transmitted the disease by the time the diagnosis is made.

How well did you know this?
1
Not at all
2
3
4
5
Perfectly
45
Q

An experienced nurse instructs a new nurse about how to care for a patient with dyspnea caused by a pulmonary fungal infection. Which action by the new nurse indicates a need for further teaching?

a. Listening to the patient’s lung sounds several times during the shift
b. Placing the patient on droplet precautions in a private hospital room
c. Monitoring patient serology results to identify the infecting organism
d. Increasing the O2 flow rate to keep the O2 saturation over 90%

A

ANS: B
Fungal infections are not transmitted from person to person. Therefore no isolation procedures are necessary. The other actions by the new nurse are appropriate.

How well did you know this?
1
Not at all
2
3
4
5
Perfectly
46
Q

Which intervention will the nurse include in the plan of care for a patient who is diagnosed with a lung abscess?

a. Teach the patient to avoid the use of over-the-counter expectorants.
b. Assist the patient with chest physiotherapy and postural drainage.
c. Notify the health care provider immediately about any bloody or foul-smelling sputum.
d. Teach about the need for prolonged antibiotic therapy after discharge from the hospital.

A

ANS: D
Long-term antibiotic therapy is needed for effective eradication of the infecting organisms in lung abscess. Chest physiotherapy and postural drainage are not recommended for lung abscess because they may lead to spread of the infection. Foul-smelling and bloody sputum are common clinical manifestations in lung abscess. Expectorants may be used because the patient is encouraged to cough.

How well did you know this?
1
Not at all
2
3
4
5
Perfectly
47
Q

The nurse provides discharge teaching for a patient who has two fractured ribs from an automobile accident. Which statement, if made by the patient, would indicate that teaching has been effective?

a. “I am going to buy a rib binder to wear during the day.”
b. “I can take shallow breaths to prevent my chest from hurting.”
c. “I should plan on taking the pain pills only at bedtime so I can sleep.”
d. “I will use the incentive spirometer every hour or two during the day.”

A

ANS: D
Prevention of the complications of atelectasis and pneumonia is a priority after rib fracture. This can be ensured by deep breathing and coughing. Use of a rib binder, shallow breathing, and taking pain medications only at night are likely to result in atelectasis.

How well did you know this?
1
Not at all
2
3
4
5
Perfectly
48
Q

The nurse is caring for a patient who has a right-sided chest tube after a right lower lobectomy. Which nursing action can the nurse delegate to the unlicensed assistive personnel (UAP)?

a. Document the amount of drainage every 8 hours.
b. Obtain samples of drainage for culture from the system.
c. Assess patient pain level associated with the chest tube.
d. Check the water-seal chamber for the correct fluid level.

A

ANS: A
UAP education includes documentation of intake and output. The other actions are within the scope of practice and education of licensed nursing personnel.

How well did you know this?
1
Not at all
2
3
4
5
Perfectly
49
Q

After change-of-shift report, which patient should the nurse assess first?

a. A 72-yr-old with cor pulmonale who has 4+ bilateral edema in his legs and feet
b. A 28-yr-old with a history of a lung transplant and a temperature of 101° F (38.3° C)
c. A 40-yr-old with a pleural effusion who is complaining of severe stabbing chest pain
d. A 64-yr-old with lung cancer and tracheal deviation after subclavian catheter insertion

A

ANS: D
The patient’s history and symptoms suggest possible tension pneumothorax, a medical emergency. The other patients also require assessment as soon as possible, but tension pneumothorax will require immediate treatment to avoid death from inadequate cardiac output or hypoxemia.

How well did you know this?
1
Not at all
2
3
4
5
Perfectly
50
Q

Which factors will the nurse consider when calculating the CURB-65 score for a patient with pneumonia (select all that apply)?

a. Age d. O2 saturation
b. Blood pressure e. Presence of confusion
c. Respiratory rate f. Blood urea nitrogen (BUN) level

A

ANS: A, B, C, E, F
Data collected for the CURB-65 are mental status (confusion), BUN (elevated), blood pressure (decreased), respiratory rate (increased), and age (65 years and older). The other information is also essential to assess, but are not used for CURB-65 scoring.

How well did you know this?
1
Not at all
2
3
4
5
Perfectly
51
Q

The nurse notes new onset confusion in an older patient who is normally alert and oriented. In which order should the nurse take the following actions? (Put a comma and a space between each answer choice [A, B, C, D].)

a. Obtain the O2 saturation.
b. Check the patient’s pulse rate.
c. Document the change in status.
d. Notify the health care provider.

A

ANS:
A, B, D, C

Assessment for physiologic causes of new onset confusion such as pneumonia, infection, or perfusion problems should be the first action by the nurse. Airway and oxygenation should be assessed first, then circulation. After assessing the patient, the nurse should notify the health care provider. Finally, documentation of the assessments and care should be done

How well did you know this?
1
Not at all
2
3
4
5
Perfectly
52
Q

A 74-yr-old patient has just arrived in the emergency department. After assessment reveals a pulse deficit of 46 beats, the nurse will anticipate that the patient may require

a. emergent cardioversion.
b. a cardiac catheterization.
c. hourly blood pressure (BP) checks.
d. electrocardiographic (ECG) monitoring.

A

ANS: D
Pulse deficit is a difference between simultaneously obtained apical and radial pulses. It indicates that there may be a cardiac dysrhythmia that would best be detected with ECG monitoring. Frequent BP monitoring, cardiac catheterization, and emergent cardioversion are used for diagnosis and/or treatment of cardiovascular disorders but would not be as helpful in determining the immediate reason for the pulse deficit.

How well did you know this?
1
Not at all
2
3
4
5
Perfectly
53
Q

The nurse is reviewing the 12-lead electrocardiograph (ECG) for a healthy 74-yr-old patient who is having an annual physical examination. What finding is of most concern to the nurse?

a. A right bundle-branch block. c. The QRS duration is 0.13 seconds.
b. The PR interval is 0.21 seconds. d. The heart rate (HR) is 41 beats/min.

A

ANS: D
The resting HR does not change with aging, so the decrease in HR requires further investigation. Bundle-branch block and slight increases in PR interval or QRS duration are common in older individuals because of increases in conduction time through the AV node, bundle of His, and bundle branches.

How well did you know this?
1
Not at all
2
3
4
5
Perfectly
54
Q

During a physical examination of an older patient, the nurse palpates the point of maximal impulse (PMI) in the sixth intercostal space lateral to the left midclavicular line. The best follow-up action for the nurse to take will be to

a. ask about risk factors for atherosclerosis.
b. determine family history of heart disease.
c. assess for symptoms of left ventricular hypertrophy.
d. auscultate carotid arteries for the presence of a bruit.

A

ANS: C
The PMI should be felt at the intersection of the fifth intercostal space and left midclavicular line. A PMI located outside these landmarks indicates possible cardiac enlargement, such as with left ventricular hypertrophy (LVH). The other assessments are part of a general cardiac assessment but do not represent follow-up for LVH. Cardiac enlargement is not necessarily associated with atherosclerosis or carotid artery disease.

How well did you know this?
1
Not at all
2
3
4
5
Perfectly
55
Q

To auscultate for S3 or S4 gallops in the mitral area, the nurse listens with the

a. diaphragm of the stethoscope with the patient lying flat.
b. bell of the stethoscope with the patient in the left lateral position.
c. diaphragm of the stethoscope with the patient in a supine position.
d. bell of the stethoscope with the patient sitting and leaning forward.

A

ANS: B
Gallop rhythms generate low-pitched sounds and are most easily heard with the bell of the stethoscope. Sounds associated with the mitral valve are accentuated by turning the patient to the left side, which brings the heart closer to the chest wall. The diaphragm of the stethoscope is best to use for the higher pitched sounds such as S1 and S2.

How well did you know this?
1
Not at all
2
3
4
5
Perfectly
56
Q

To determine the effects of therapy for a patient who is being treated for heart failure, which laboratory test result will the nurse plan to review?

a. Troponin c. Low-density lipoprotein (LDL)
b. Homocysteine (Hcy) d. B-type natriuretic peptide (BNP)

A

ANS: D
Increased levels of BNP are a marker for heart failure. The other laboratory results would be used to assess for myocardial infarction (troponin) or risk for coronary artery disease (Hcy and LDL).

How well did you know this?
1
Not at all
2
3
4
5
Perfectly
57
Q

While doing the hospital admission assessment for a thin older adult, the nurse observes pulsation of the abdominal aorta in the epigastric area. Which action should the nurse take next?

a. Teach the patient about aneurysms.
b. Notify the hospital rapid response team.
c. Instruct the patient to remain on bed rest.
d. Document the finding in the patient chart.

A

ANS: D
Visible pulsation of the abdominal aorta is commonly observed in the epigastric area for thin individuals. The nurse should simply document the finding in the admission assessment. Unless there are other abnormal findings (such as a bruit, pain, or hyper/hypotension) associated with the pulsation, the other actions are not necessary.

How well did you know this?
1
Not at all
2
3
4
5
Perfectly
58
Q

A patient is scheduled for a cardiac catheterization with coronary angiography. Before the test, the nurse informs the patient that

a. it will be important not to move at all during the procedure.
b. monitored anesthesia care will be provided during the procedure.
c. a flushed feeling may be noticed when the contrast dye is injected.
d. arterial pressure monitoring will be required for 24 hours after the test.

A

ANS: C
A sensation of warmth or flushing is common when the contrast material is injected, which can be anxiety producing unless it has been discussed with the patient. The patient may receive a sedative drug before the procedure, but monitored anesthesia care is not used. Arterial pressure monitoring is not routinely used after the procedure to monitor blood pressure. The patient is not immobile during cardiac catheterization and may be asked to cough or take deep breaths.

How well did you know this?
1
Not at all
2
3
4
5
Perfectly
59
Q

The nurse notes that a patient who was admitted with heart failure has jugular venous distention (JVD) when lying flat in bed. Which follow-up action should the nurse take next?

a. Obtain vital signs, including oxygen saturation.
b. Have the patient perform the Valsalva maneuver.
c. Document this JVD finding in the patient’s record.
d. Observe for JVD with the patient elevated 45 degrees.

A

ANS: D
When the patient is lying flat, the jugular veins are at the level of the right atrium, so JVD is a common (but not a clinically significant) finding. Obtaining vital signs and oxygen saturation is not warranted at this point. JVD is an expected finding when a patient performs the Valsalva maneuver because right atrial pressure increases. JVD that persists when the patient is sitting at a 30- to 45-degree angle or greater is significant. The nurse will document the JVD in the medical record if it persists when the head is elevated.

How well did you know this?
1
Not at all
2
3
4
5
Perfectly
60
Q

The nurse teaches the patient being evaluated for rhythm disturbances with a Holter monitor to

a. connect the recorder to a computer once daily.
b. exercise more than usual while the monitor is in place.
c. remove the electrodes when taking a shower or tub bath.
d. keep a diary of daily activities while the monitor is worn.

A

ANS: D
The patient is instructed to keep a diary describing daily activities while Holter monitoring is being accomplished to help correlate any rhythm disturbances with patient activities. Patients are taught that they should not take a shower or bath during Holter monitoring and that they should continue with their usual daily activities. The recorder stores the information about the patient’s rhythm until the end of the testing, when it is removed and the data are analyzed.

How well did you know this?
1
Not at all
2
3
4
5
Perfectly
61
Q

When auscultating over the patient’s abdominal aorta, the nurse hears a loud humming sound. The nurse documents this finding as a

a. thrill. c. murmur.
b. bruit. d. normal finding.

A

ANS: B
A bruit is the sound created by turbulent blood flow in an artery. Thrills are palpable vibrations felt when there is turbulent blood flow through the heart or in a blood vessel. A murmur is the sound caused by turbulent blood flow through the heart. Auscultating a bruit in an artery is not normal and indicates pathology.

How well did you know this?
1
Not at all
2
3
4
5
Perfectly
62
Q

The nurse has received the laboratory results for a patient who developed chest pain 4 hours ago and may be having a myocardial infarction. The laboratory test result most helpful in indicating myocardial damage will be

a. myoglobin. c. homocysteine (Hcy)
b. troponins T and I. d. creatine kinase-MB (CK-MB).

A

ANS: B
Cardiac troponins start to elevate 4 to 6 hours after myocardial injury and are highly specific to myocardium. They are the preferred diagnostic marker for myocardial infarction. Myoglobin rises in response to myocardial injury within 30 to 60 minutes. It is rapidly cleared from the body, thus limiting its use in the diagnosis of myocardial infarction. Low-density lipoprotein cholesterol is useful in assessing cardiovascular risk but is not helpful in determining whether a patient is having an acute myocardial infarction. Creatine kinase (CK-MB) is specific to myocardial injury and infarction and increases 4 to 6 hours after the infarction occurs. It is often trended with troponin levels. Homocysteine (Hcy) is an amino acid that is produced during protein catabolism. Elevated Hcy levels can be either hereditary or acquired from dietary deficiencies of vitamin B6, cobalamin (vitamin B12), or folate. Elevated levels of Hcy have been linked to a higher risk of CVD, peripheral vascular disease, and stroke.

How well did you know this?
1
Not at all
2
3
4
5
Perfectly
63
Q

When assessing a newly admitted patient, the nurse notes a murmur along the left sternal border. To acquire more information about the murmur, which action will the nurse take?

a. Palpate the peripheral pulses.
b. Determine the timing of the sound.
c. Find the point of maximal impulse.
d. Compare apical and radial pulse rates.

A

ANS: B
Murmurs are caused by turbulent blood flow, such as occurs when blood flows through a damaged valve. Relevant information includes the position in which the murmur is heard best (e.g., sitting and leaning forward), the timing of the murmur in relation to the cardiac cycle (e.g., systole, diastole), and where on the thorax the murmur is heard best. The other information is important in the cardiac assessment but will not provide information that is relevant to the murmur.

How well did you know this?
1
Not at all
2
3
4
5
Perfectly
64
Q

The nurse hears a murmur between the S1 and S2 heart sounds at the patient’s left fifth intercostal space and midclavicular line. How will the nurse record this information?

a. Systolic murmur heard at mitral area
b. Systolic murmur heard at Erb’s point
c. Diastolic murmur heard at aortic area
d. Diastolic murmur heard at the point of maximal impulse

A

ANS: A
The S1 signifies the onset of ventricular systole. S2 signifies the onset of diastole. A murmur occurring between these two sounds is a systolic murmur. The mitral area is the intersection of the left fifth intercostal space and the midclavicular line. The other responses describe murmurs heard at different landmarks on the chest and/or during the diastolic phase of the cardiac cycle.

How well did you know this?
1
Not at all
2
3
4
5
Perfectly
65
Q

A registered nurse (RN) is observing a student nurse who is doing a physical assessment on a patient. The RN will need to intervene immediately if the student nurse

a. presses on the skin over the tibia for 10 seconds to check for edema.
b. palpates both carotid arteries simultaneously to compare pulse quality.
c. documents a murmur heard along the right sternal border as a pulmonic murmur.
d. places the patient in the left lateral position to check for the point of maximal impulse.

A

ANS: B
The carotid pulses should never be palpated at the same time to avoid vagal stimulation, dysrhythmias, and decreased cerebral blood flow. The other assessment techniques also need to be corrected. However, they are not dangerous to the patient.

How well did you know this?
1
Not at all
2
3
4
5
Perfectly
66
Q

Which action will the nurse implement for a patient who arrives for a calcium-scoring CT scan?

a. Insert an IV catheter.
b. Administer oral sedative medications.
c. Teach the patient about the procedure.
d. Confirm that the patient has been fasting.

A

ANS: C
The nurse will need to teach the patient that the procedure is rapid and involves little risk. None of the other actions are necessary.

How well did you know this?
1
Not at all
2
3
4
5
Perfectly
67
Q

Which information obtained by the nurse who is admitting the patient for magnetic resonance imaging (MRI) will be important to report to the health care provider before the MRI?

a. The patient has an allergy to shellfish.
b. The patient has a history of atherosclerosis.
c. The patient has a permanent cardiac pacemaker.
d. The patient took the prescribed heart medications today.

A

ANS: C
MRI is contraindicated for patients with implanted metallic devices such as pacemakers. The other information does not affect whether or not the patient can have an MRI.

How well did you know this?
1
Not at all
2
3
4
5
Perfectly
68
Q

When the nurse is monitoring a patient who is undergoing exercise (stress) testing on a treadmill, which assessment finding requires the most rapid action by the nurse?

a. Patient complaint of feeling tired
b. Sinus tachycardia at a rate of 110 beats/min
c. Inversion of T waves on the electrocardiogram
d. Blood pressure (BP) increase from 134/68 to 150/80 mm Hg

A

ANS: C
ECG changes associated with coronary ischemia (such as T-wave inversions and ST segment depression) indicate that the myocardium is not getting adequate O2 delivery and that the exercise test should be terminated immediately. Increases in BP and heart rate are normal responses to aerobic exercise. Feeling tired is also normal as the intensity of exercise increases during the stress testing.

How well did you know this?
1
Not at all
2
3
4
5
Perfectly
69
Q

The standard policy on the cardiac unit states, “Notify the health care provider for mean arterial pressure (MAP) less than 70 mm Hg.” The nurse will need to call the health care provider about the

a. postoperative patient with a BP of 116/42 mm Hg.
b. newly admitted patient with a BP of 150/87 mm Hg.
c. patient with left ventricular failure who has a BP of 110/70 mm Hg.
d. patient with a myocardial infarction who has a BP of 140/86 mm Hg.

A

ANS: A
The mean arterial pressure (MAP) is calculated using the formula MAP = (systolic BP + 2 diastolic BP)/3. The MAP for the postoperative patient in answer 3 is 67. The MAP in the other three patients is higher than 70 mm Hg.

How well did you know this?
1
Not at all
2
3
4
5
Perfectly
70
Q

When admitting a patient for a cardiac catheterization and coronary angiogram, which information about the patient is most important for the nurse to communicate to the health care provider?

a. The patient’s pedal pulses are +1.
b. The patient is allergic to shellfish.
c. The patient had a heart attack 1 year ago.
d. The patient has not eaten anything today.

A

ANS: B
The contrast dye used for the procedure is iodine based, so patients who have shellfish allergies will require treatment with medications such as corticosteroids and antihistamines before the angiogram. The other information is also communicated to the health care provider but will not require a change in the usual precardiac catheterization orders or medications.

How well did you know this?
1
Not at all
2
3
4
5
Perfectly
71
Q

A transesophageal echocardiogram (TEE) is ordered for a patient with possible endocarditis. Which action included in the standard TEE orders will the nurse need to accomplish first?

a. Start an IV line. c. Place the patient on NPO status.
b. Start O2 per nasal cannula. d. Give lorazepam (Ativan) 1 mg IV.

A

ANS: C
The patient will need to be NPO for 6 hours preceding the TEE, so the nurse should place the patient on NPO status as soon as the order is received. The other actions also will need to be accomplished but not until just before or during the procedure.

How well did you know this?
1
Not at all
2
3
4
5
Perfectly
72
Q

The nurse and unlicensed assistive personnel (UAP) on the telemetry unit are caring for four patients. Which nursing action can be delegated to the UAP?

a. Teaching a patient about exercise electrocardiography
b. Attaching ECG monitoring electrodes after a patient bathes
c. Checking the catheter insertion site for a patient who is recovering from a coronary angiogram
d. Monitoring a patient who has just returned to the unit after a transesophageal echocardiogram

A

ANS: B
UAP can be educated in standardized lead placement for ECG monitoring. Assessment of patients who have had procedures where airway maintenance (transesophageal echocardiography) or bleeding (coronary angiogram) is a concern must be done by the registered nurse (RN). Patient teaching requires RN level education and scope of practice.

How well did you know this?
1
Not at all
2
3
4
5
Perfectly
73
Q

The nurse is reviewing the laboratory results for newly admitted patients on the cardiovascular unit. Which laboratory result is most important to communicate as soon as possible to the health care provider?

a. High troponin I level
b. Increased triglyceride level
c. Very low homocysteine level
d. Elevated high-sensitivity C-reactive protein level

A

ANS: A
The elevation in troponin I indicates that the patient has had an acute myocardial infarction. Further assessment and interventions are indicated. The other laboratory results are indicative of increased risk for coronary artery disease but are not associated with acute cardiac problems that need immediate intervention.

How well did you know this?
1
Not at all
2
3
4
5
Perfectly
74
Q

A patient who has been in the intensive care unit for 4 days has disturbed sensory perception from sleep deprivation. Which action should the nurse include in the plan of care?

a. Administer prescribed sedatives or opioids at bedtime to promote sleep.
b. Cluster nursing activities so that the patient has uninterrupted rest periods.
c. Silence the alarms on the cardiac monitors to allow 30- to 40-minute naps.
d. Eliminate assessments between 2200 and 0600 to allow uninterrupted sleep.

A

ANS: B
Clustering nursing activities and providing uninterrupted rest periods will minimize sleep-cycle disruption. Sedative and opioid medications tend to decrease the amount of rapid eye movement (REM) sleep and can contribute to sleep disturbance and disturbed sensory perception. Silencing the alarms on the cardiac monitors would be unsafe in a critically ill patient, as would discontinuing all assessments during the night.

How well did you know this?
1
Not at all
2
3
4
5
Perfectly
75
Q

Which hemodynamic parameter best reflects the effectiveness of drugs that the nurse gives to reduce a patient’s left ventricular afterload?

a. Mean arterial pressure (MAP)
b. Systemic vascular resistance (SVR)
c. Pulmonary vascular resistance (PVR)
d. Pulmonary artery wedge pressure (PAWP)

A

ANS: B
SVR reflects the resistance to ventricular ejection, or afterload. The other parameters may be monitored but do not reflect afterload as directly.

How well did you know this?
1
Not at all
2
3
4
5
Perfectly
76
Q

While close family members are visiting, a patient has a respiratory arrest, and resuscitation is started. Which action by the nurse is best?

a. Tell the family members that watching the resuscitation will be very stressful.
b. Ask family members if they wish to remain in the room during the resuscitation.
c. Take the family members quickly out of the patient room and remain with them.
d. Assign a staff member to wait with family members just outside the patient room.

A

ANS: B
Evidence indicates that many family members want the option of remaining in the room during procedures such as cardiopulmonary resuscitation (CPR) and that this decreases anxiety and facilitates grieving. The other options may be appropriate if the family decides not to remain with the patient.

How well did you know this?
1
Not at all
2
3
4
5
Perfectly
77
Q

After surgery for an abdominal aortic aneurysm, a patient’s central venous pressure (CVP) monitor indicates low pressures. Which action should the nurse take?

a. Administer IV diuretic medications.
b. Increase the IV fluid infusion per protocol.
c. Increase the infusion rate of IV vasodilators.
d. Elevate the head of the patient’s bed to 45 degrees.

A

ANS: B
A low CVP indicates hypovolemia and a need for an increase in the infusion rate. Diuretic administration will contribute to hypovolemia and elevation of the head or increasing vasodilators may decrease cerebral perfusion.

How well did you know this?
1
Not at all
2
3
4
5
Perfectly
78
Q

When caring for a patient with pulmonary hypertension, which parameter will the nurse use to directly evaluate the effectiveness of the treatment?

a. Central venous pressure (CVP)
b. Systemic vascular resistance (SVR)
c. Pulmonary vascular resistance (PVR)
d. Pulmonary artery wedge pressure (PAWP)

A

ANS: C
PVR is a major contributor to pulmonary hypertension, and a decrease would indicate that pulmonary hypertension was improving. The other parameters may also be monitored but do not directly assess for pulmonary hypertension.

How well did you know this?
1
Not at all
2
3
4
5
Perfectly
79
Q

The intensive care unit (ICU) nurse educator determines that teaching a new staff nurse about arterial pressure monitoring has been effective when the nurse

a. balances and calibrates the monitoring equipment every 2 hours.
b. positions the zero-reference stopcock line level with the phlebostatic axis.
c. ensures that the patient is supine with the head of the bed flat for all readings.
d. rechecks the location of the phlebostatic axis with changes in the patient’s position.

A

ANS: B
For accurate measurement of pressures, the zero-reference level should be at the phlebostatic axis. There is no need to rebalance and recalibrate monitoring equipment every 2 hours. Accurate hemodynamic readings are possible with the patient’s head raised to 45 degrees or in the prone position. The anatomic position of the phlebostatic axis does not change when patients are repositioned.

How well did you know this?
1
Not at all
2
3
4
5
Perfectly
80
Q

When monitoring the effectiveness of treatment for a patient with a large anterior wall myocardial infarction, the most pertinent measurement for the nurse to obtain is

a. central venous pressure (CVP).
b. systemic vascular resistance (SVR).
c. pulmonary vascular resistance (PVR).
d. pulmonary artery wedge pressure (PAWP).

A

ANS: D
PAWP reflects left ventricular end diastolic pressure (or left ventricular preload) and is a sensitive indicator of cardiac function. Because the patient is high risk for left ventricular failure, the PAWP must be monitored. An increase will indicate left ventricular failure. The other values would also provide useful information, but the most definitive measurement of changes in cardiac function is the PAWP.

How well did you know this?
1
Not at all
2
3
4
5
Perfectly
81
Q

Which action should the nurse take when the low pressure alarm sounds for a patient who has an arterial line in the left radial artery?

a. Fast flush the arterial line.
b. Check the left hand for pallor.
c. Assess for cardiac dysrhythmias.
d. Re-zero the monitoring equipment.

A

ANS: C
The low pressure alarm indicates a drop in the patient’s blood pressure, which may be caused by cardiac dysrhythmias. There is no indication to re-zero the equipment. Pallor of the left hand would be caused by occlusion of the radial artery by the arterial catheter, not by low pressure. There is no indication of a need for flushing the line.

How well did you know this?
1
Not at all
2
3
4
5
Perfectly
82
Q

Which nursing action is needed when preparing to assist with the insertion of a pulmonary artery catheter?

a. Determine if the cardiac troponin level is elevated.
b. Auscultate heart sounds before and during insertion.
c. Place the patient on NPO status before the procedure.
d. Attach cardiac monitoring leads before the procedure.

A

ANS: D
Dysrhythmias can occur as the catheter is floated through the right atrium and ventricle, and it is important for the nurse to monitor for these during insertion. Pulmonary artery catheter insertion does not require anesthesia, and the patient will not need to be NPO. Changes in cardiac troponin or heart and breath sounds are not expected during pulmonary artery catheter insertion.

How well did you know this?
1
Not at all
2
3
4
5
Perfectly
83
Q

While assisting with the placement of a pulmonary artery (PA) catheter, the nurse notes that the catheter is correctly placed when the balloon is inflated and the monitor shows a

a. typical PA pressure waveform.
b. tracing of the systemic arterial pressure.
c. tracing of the systemic vascular resistance.
d. typical PA wedge pressure (PAWP) tracing.

A

ANS: D
The purpose of a PA line is to measure PAWP, so the catheter is floated through the pulmonary artery until the dilated balloon wedges in a distal branch of the pulmonary artery, and the PAWP readings are available. After insertion, the balloon is deflated and the PA waveform will be observed. Systemic arterial pressures are obtained using an arterial line, and the systemic vascular resistance is a calculated value, not a waveform.

How well did you know this?
1
Not at all
2
3
4
5
Perfectly
84
Q

Which assessment finding obtained by the nurse when caring for a patient with a right radial arterial line indicates a need for the nurse to take action?

a. The right hand feels cooler than the left hand.
b. The mean arterial pressure (MAP) is 77 mm Hg.
c. The system is delivering 3 mL of flush solution per hour.
d. The flush bag and tubing were last changed 2 days previously.

A

ANS: A
The change in temperature of the right hand suggests that blood flow to the right hand is impaired. The flush system needs to be changed every 96 hours. A mean arterial pressure (MAP) of 75 mm Hg is normal. Flush systems for hemodynamic monitoring are set up to deliver 3 to 6 mL/hr of flush solution.

How well did you know this?
1
Not at all
2
3
4
5
Perfectly
85
Q

The central venous oxygen saturation (ScvO2) is decreasing in a patient who has severe pancreatitis. To determine the possible cause of the decreased ScvO2, the nurse assesses the patient’s

a. lipase level. c. urinary output.
b. temperature. d. body mass index.

A

ANS: B
Elevated temperature increases metabolic demands and O2 use by tissues, resulting in a drop in O2 saturation of central venous blood. Information about the patient’s body mass index, urinary output, and lipase will not help in determining the cause of the patient’s drop in ScvO2.

How well did you know this?
1
Not at all
2
3
4
5
Perfectly
86
Q

An intraaortic balloon pump (IABP) is being used for a patient who is in cardiogenic shock. Which assessment data indicate to the nurse that the goals of treatment with the IABP are being met?

a. Urine output of 25 mL/hr
b. Heart rate of 110 beats/minute
c. Cardiac output (CO) of 5 L/min
d. Stroke volume (SV) of 40 mL/beat

A

ANS: C
A CO of 5 L/min is normal and indicates that the IABP has been successful in treating the shock. The low SV signifies continued cardiogenic shock. The tachycardia and low urine output also suggest continued cardiogenic shock.

How well did you know this?
1
Not at all
2
3
4
5
Perfectly
87
Q

The nurse is caring for a patient who has an intraaortic balloon pump in place. Which action should be included in the plan of care?

a. Avoid the use of anticoagulant medications.
b. Measure the patient’s urinary output every hour.
c. Provide passive range of motion for all extremities.
d. Position the patient supine with head flat at all times.

A

ANS: B
Monitoring urine output will help determine whether the patient’s cardiac output has improved and also help monitor for balloon displacement blocking the renal arteries. The head of the bed can be elevated up to 30 degrees. Heparin is used to prevent thrombus formation. Limited movement is allowed for the extremity with the balloon insertion site to prevent displacement of the balloon.

How well did you know this?
1
Not at all
2
3
4
5
Perfectly
88
Q

While waiting for heart transplantation, a patient with severe cardiomyopathy has a ventricular assist device (VAD) implanted. When planning care for this patient, the nurse should anticipate

a. preparing the patient for a permanent VAD.
b. administering immunosuppressive medications.
c. teaching the patient the reason for complete bed rest.
d. monitoring the surgical incision for signs of infection.

A

ANS: D
The insertion site for the VAD provides a source for transmission of infection to the circulatory system and requires frequent monitoring. Patients with VADs are able to have some mobility and may not be on bed rest. The VAD is a bridge to transplantation, not a permanent device. Immunosuppression is not necessary for nonbiologic devices such as the VAD.

How well did you know this?
1
Not at all
2
3
4
5
Perfectly
89
Q

To verify the correct placement of an oral endotracheal tube (ET) after insertion, the best initial action by the nurse is to

a. obtain a portable chest x-ray.
b. use an end-tidal CO2 monitor.
c. auscultate for bilateral breath sounds.
d. observe for symmetrical chest movement.

A

ANS: B
End-tidal CO2 monitors are currently recommended for rapid verification of ET placement. Auscultation for bilateral breath sounds and checking chest expansion are also used, but they are not as accurate as end-tidal CO2 monitoring. A chest x-ray confirms the placement but is done after the tube is secured.

How well did you know this?
1
Not at all
2
3
4
5
Perfectly
90
Q

To maintain proper cuff pressure of an endotracheal tube (ET) when the patient is on mechanical ventilation, the nurse should

a. inflate the cuff with a minimum of 10 mL of air.
b. inflate the cuff until the pilot balloon is firm on palpation.
c. inject air into the cuff until a manometer shows 15 mm Hg pressure.
d. inject air into the cuff until a slight leak is heard only at peak inflation.

A

ANS: D
The minimal occluding volume technique involves injecting air into the cuff until an air leak is present only at peak inflation. The volume to inflate the cuff varies with the ET and the patient’s size. Cuff pressure should be maintained at 20 to 25 mm Hg. An accurate assessment of cuff pressure cannot be obtained by palpating the pilot balloon.

How well did you know this?
1
Not at all
2
3
4
5
Perfectly
91
Q

The nurse notes premature ventricular contractions (PVCs) while suctioning a patient’s endotracheal tube. Which next action by the nurse is indicated?

a. Plan to suction the patient more frequently.
b. Decrease the suction pressure to 80 mm Hg.
c. Give antidysrhythmic medications per protocol.
d. Stop and ventilate the patient with 100% oxygen.

A

ANS: D
Dysrhythmias during suctioning may indicate hypoxemia or sympathetic nervous system stimulation. The nurse should stop suctioning and ventilate the patient with 100% O2. There is no indication that more frequent suctioning is needed. Lowering the suction pressure will decrease the effectiveness of suctioning without improving the hypoxemia. Because the PVCs occurred during suctioning, there is no need for antidysrhythmic medications (which may have adverse effects) unless they recur when the suctioning is stopped and patient is well oxygenated.

How well did you know this?
1
Not at all
2
3
4
5
Perfectly
92
Q

Which assessment finding obtained by the nurse when caring for a patient receiving mechanical ventilation indicates the need for suctioning?

a. The patient was last suctioned 6 hours ago.
b. The patient’s oxygen saturation drops to 93%.
c. The patient’s respiratory rate is 32 breaths/min.
d. The patient has occasional audible expiratory wheezes.

A

ANS: C
The increase in respiratory rate indicates that the patient may have decreased airway clearance and requires suctioning. Suctioning is done when patient assessment data indicate that it is needed and not on a scheduled basis. Occasional expiratory wheezes do not indicate poor airway clearance, and suctioning the patient may induce bronchospasm and increase wheezing. An O2 saturation of 93% is acceptable and does not suggest that immediate suctioning is needed.

How well did you know this?
1
Not at all
2
3
4
5
Perfectly
93
Q

The nurse notes thick, white secretions in the endotracheal tube (ET) of a patient who is receiving mechanical ventilation. Which intervention will most directly treat this finding?

a. Reposition the patient every 1 to 2 hours.
b. Increase suctioning frequency to every hour.
c. Add additional water to the patient’s enteral feedings.
d. Instill 5 mL of sterile saline into the ET before suctioning.

A

ANS: C
Because the patient’s secretions are thick, better hydration is indicated. Suctioning every hour without any specific evidence for the need will increase the incidence of mucosal trauma and would not address the etiology of the ineffective airway clearance. Instillation of saline does not liquefy secretions and may decrease the SpO2. Repositioning the patient is appropriate but will not decrease the thickness of secretions.

How well did you know this?
1
Not at all
2
3
4
5
Perfectly
94
Q

Four hours after mechanical ventilation is initiated, a patient’s arterial blood gas (ABG) results include a pH of 7.51, PaO2 of 82 mm Hg, PaCO2 of 26 mm Hg, and HCO3– of 23 mEq/L (23 mmol/L). The nurse will anticipate the need to

a. increase the FIO2. c. increase the respiratory rate.
b. increase the tidal volume. d. decrease the respiratory rate.

A

ANS: D
The patient’s PaCO2 and pH indicate respiratory alkalosis caused by too high a respiratory rate. The PaO2 is appropriate for a patient with COPD and increasing the respiratory rate and tidal volume would further lower the PaCO2.

How well did you know this?
1
Not at all
2
3
4
5
Perfectly
95
Q

A patient with respiratory failure has arterial pressure–based cardiac output (APCO) monitoring and is receiving mechanical ventilation with peak end-expiratory pressure (PEEP) of 12 cm H2O. Which information indicates that a change in the ventilator settings may be required?

a. The arterial pressure is 90/46.
b. The stroke volume is increased.
c. The heart rate is 58 beats/minute.
d. The stroke volume variation is 12%.

A

ANS: A
The hypotension suggests that the high intrathoracic pressure caused by the PEEP may be decreasing venous return and (potentially) cardiac output. The other assessment data would not be a direct result of PEEP and mechanical ventilation.

How well did you know this?
1
Not at all
2
3
4
5
Perfectly
96
Q

A nurse is weaning a 68-kg patient who has chronic obstructive pulmonary disease (COPD) from mechanical ventilation. Which patient assessment finding indicates that the weaning protocol should be stopped?

a. The patient’s heart rate is 97 beats/min.
b. The patient’s oxygen saturation is 93%.
c. The patient respiratory rate is 32 breaths/min.
d. The patient’s spontaneous tidal volume is 450 mL.

A

ANS: C
Tachypnea is a sign that the patient’s work of breathing is too high to allow weaning to proceed. The patient’s heart rate is within normal limits, but the nurse should continue to monitor it. An O2 saturation of 93% is acceptable for a patient with COPD. A spontaneous tidal volume of 450 mL is within the acceptable range.

How well did you know this?
1
Not at all
2
3
4
5
Perfectly
97
Q

The nurse is caring for a patient receiving a continuous norepinephrine IV infusion. Which patient assessment finding indicates that the infusion rate may need to be adjusted?

a. Heart rate is slow at 58 beats/min.
b. Mean arterial pressure (MAP) is 56 mm Hg.
c. Systemic vascular resistance (SVR) is elevated.
d. Pulmonary artery wedge pressure (PAWP) is low.

A

ANS: C
Vasoconstrictors such as norepinephrine will increase SVR, and this will increase the work of the heart and decrease peripheral perfusion. The infusion rate may need to be decreased. Bradycardia, hypotension (MAP of 56 mm Hg), and low PAWP are not associated with norepinephrine infusion.

How well did you know this?
1
Not at all
2
3
4
5
Perfectly
98
Q

When evaluating a patient with a central venous catheter, the nurse observes that the insertion site is red and tender to touch and the patient’s temperature is 101.8° F. What should the nurse plan to do?

a. Discontinue the catheter and culture the tip.
b. Use the catheter only for fluid administration.
c. Change the flush system and monitor the site.
d. Check the site more frequently for any swelling.

A

ANS: A
The information indicates that the patient has a local and systemic infection caused by the catheter, and the catheter should be discontinued to avoid further complications such as endocarditis. Changing the flush system, continued monitoring, or using the line for fluids will not help prevent or treat the infection.

How well did you know this?
1
Not at all
2
3
4
5
Perfectly
99
Q

An 81-yr-old patient who has been in the intensive care unit (ICU) for a week is now stable and transfer to the progressive care unit is planned. On rounds, the nurse notices that the patient has new onset confusion. The nurse will plan to

a. give PRN lorazepam (Ativan) and cancel the transfer.
b. inform the receiving nurse and then transfer the patient.
c. notify the health care provider and postpone the transfer.
d. obtain an order for restraints as needed and transfer the patient.

A

ANS: B
The patient’s history and symptoms most likely indicate delirium associated with the sleep deprivation and sensory overload in the ICU environment. Informing the receiving nurse and transferring the patient is appropriate. Postponing the transfer is likely to prolong the delirium. Benzodiazepines and restraints contribute to delirium and agitation.

How well did you know this?
1
Not at all
2
3
4
5
Perfectly
100
Q

The family members of a patient who has been admitted to the intensive care unit (ICU) with multiple traumatic injuries have just arrived in the ICU waiting room. Which action should the nurse take first?

a. Explain ICU visitation policies and encourage family visits.
b. Escort the family from the waiting room to the patient’s bedside.
c. Describe the patient’s injuries and the care that is being provided.
d. Invite the family to participate in an interprofessional care conference.

A

ANS: C
Lack of information is a major source of anxiety for family members and should be addressed first. Family members should be prepared for the patient’s appearance and the ICU environment before visiting the patient for the first time. ICU visiting should be individualized to each patient and family rather than being dictated by rigid visitation policies. Inviting the family to participate in a multidisciplinary conference is appropriate but should not be the initial action by the nurse.

How well did you know this?
1
Not at all
2
3
4
5
Perfectly
101
Q

The nurse is caring for a patient who has an arterial catheter in the left radial artery for arterial pressure–based cardiac output (APCO) monitoring. Which information obtained by the nurse requires a report to the health care provider?

a. The patient has a positive Allen test result.
b. There is redness at the catheter insertion site.
c. The mean arterial pressure (MAP) is 86 mm Hg.
d. The dicrotic notch is visible in the arterial waveform.

A

ANS: B
Redness at the catheter insertion site indicates possible infection. The Allen test is performed before arterial line insertion, and a positive test result indicates normal ulnar artery perfusion. A MAP of 86 mm Hg is normal, and the dicrotic notch is normally present on the arterial waveform.

How well did you know this?
1
Not at all
2
3
4
5
Perfectly
102
Q

The nurse responds to a ventilator alarm and finds the patient lying in bed gasping and holding the endotracheal tube (ET) in her hand. Which action should the nurse take next?

a. Activate the rapid response team.
b. Provide reassurance to the patient.
c. Call the health care provider to reinsert the tube.
d. Manually ventilate the patient with 100% oxygen.

A

ANS: D
The nurse should ensure maximal patient oxygenation by manually ventilating with a bag-valve-mask system. Offering reassurance to the patient, notifying the health care provider about the need to reinsert the tube, and activating the rapid response team are also appropriate after the nurse has stabilized the patient’s oxygenation.

How well did you know this?
1
Not at all
2
3
4
5
Perfectly
103
Q

The nurse notes that a patient’s endotracheal tube (ET), which was at the 22-cm mark, is now at the 25-cm mark, and the patient is anxious and restless. Which action should the nurse take next?

a. Check the O2 saturation.
b. Offer reassurance to the patient.
c. Listen to the patient’s breath sounds.
d. Notify the patient’s health care provider.

A

ANS: C
The nurse should first determine whether the ET tube has been displaced into the right mainstem bronchus by listening for unilateral breath sounds. If so, assistance will be needed to reposition the tube immediately. The other actions are also appropriate, but detection and correction of tube malposition are the most critical actions.

How well did you know this?
1
Not at all
2
3
4
5
Perfectly
104
Q

The nurse educator is evaluating the care that a new registered nurse (RN) provides to a patient receiving mechanical ventilation. Which action by the new RN indicates the need for more education?

a. The RN increases the FIO2 to 100% before suctioning.
b. The RN secures a bite block in place using adhesive tape.
c. The RN asks for assistance to resecure the endotracheal tube.
d. The RN positions the patient with the head of bed at 10 degrees.

A

ANS: D
The head of the patient’s bed should be positioned at 30 to 45 degrees to prevent ventilator-associated pneumonia. The other actions by the new RN are appropriate.

How well did you know this?
1
Not at all
2
3
4
5
Perfectly
105
Q

A patient who is orally intubated and receiving mechanical ventilation is anxious and is “fighting” the ventilator. Which action should the nurse take next?

a. Verbally coach the patient to breathe with the ventilator.
b. Sedate the patient with the ordered PRN lorazepam (Ativan).
c. Manually ventilate the patient with a bag-valve-mask device.
d. Increase the rate for the ordered propofol (Diprivan) infusion.

A

ANS: A
The initial response by the nurse should be to try to decrease the patient’s anxiety by coaching the patient about how to coordinate respirations with the ventilator. The other actions may also be helpful if the verbal coaching is ineffective in reducing the patient’s anxiety.

How well did you know this?
1
Not at all
2
3
4
5
Perfectly
106
Q

The nurse educator is evaluating the performance of a new registered nurse (RN) who is providing care to a patient who is receiving mechanical ventilation with 15 cm H2O of peak end-expiratory pressure (PEEP). Which action indicates that the new RN is safe?

a. The RN plans to suction the patient every 1 to 2 hours.
b. The RN uses a closed-suction technique to suction the patient.
c. The RN tapes the connection between the ventilator tubing and the ET.
d. The RN changes the ventilator circuit tubing routinely every 48 hours.

A

ANS: B
The closed-suction technique is used when patients require high levels of PEEP (>10 cm H2O) to prevent the loss of PEEP that occurs when disconnecting the patient from the ventilator. Suctioning should not be scheduled routinely, but it should be done only when patient assessment data indicate the need for suctioning. Taping connections between the ET and ventilator tubing would restrict the ability of the tubing to swivel in response to patient repositioning. Ventilator tubing changes increase the risk for ventilator-associated pneumonia and are not indicated routinely.

How well did you know this?
1
Not at all
2
3
4
5
Perfectly
107
Q

The nurse is caring for a patient with a subarachnoid hemorrhage who is intubated and placed on a mechanical ventilator with 10 cm H2O of peak end-expiratory pressure (PEEP). When monitoring the patient, the nurse will need to notify the health care provider immediately if the patient develops

a. O2 saturation of 93%.
b. green nasogastric tube drainage.
c. respirations of 20 breaths/minute.
d. increased jugular venous distention.

A

ANS: D
Increases in jugular venous distention in a patient with a subarachnoid hemorrhage may indicate an increase in intracranial pressure (ICP) and that the PEEP setting is too high for this patient. A respiratory rate of 20, O2 saturation of 93%, and green nasogastric tube drainage are within normal limits.

How well did you know this?
1
Not at all
2
3
4
5
Perfectly
108
Q

A patient who is receiving positive pressure ventilation is scheduled for a spontaneous breathing trial (SBT). Which finding by the nurse is most likely to result in postponing the SBT?

a. New ST segment elevation is noted on the cardiac monitor.
b. Enteral feedings are being given through an orogastric tube.
c. Scattered rhonchi are heard when auscultating breath sounds.
d. hydromorphone (Dilaudid) is being used to treat postoperative pain.

A

ANS: A
Myocardial ischemia is a contraindication for ventilator weaning. The ST segment elevation is an indication that weaning should be postponed until further investigation and/or treatment for myocardial ischemia can be done. Ventilator weaning can proceed when opioids are used for pain management, abnormal lung sounds are present, or enteral feedings are being used.

How well did you know this?
1
Not at all
2
3
4
5
Perfectly
109
Q

After change-of-shift report on a ventilator weaning unit, which patient should the nurse assess first?

a. Patient who failed a spontaneous breathing trial and has been placed in a rest mode on the ventilator
b. Patient who is intubated and has continuous partial pressure end-tidal CO2 (PETCO2) monitoring
c. Patient who was successfully weaned and extubated 4 hours ago and has no urine output for the last 6 hours
d. Patient with a central venous O2 saturation (ScvO2) of 69% while on bilevel positive airway pressure (BiPAP)

A

ANS: C
The decreased urine output may indicate acute kidney injury or that the patient’s cardiac output and perfusion of vital organs have decreased. Any of these causes would require rapid action. The data about the other patients indicate that their conditions are stable and do not require immediate assessment or changes in their care. Continuous PETCO2 monitoring is frequently used when patients are intubated. The rest mode should be used to allow patient recovery after a failed SBT, and an ScvO2 of 69% is within normal limits.

How well did you know this?
1
Not at all
2
3
4
5
Perfectly
110
Q

After change-of-shift report, which patient should the progressive care nurse assess first?

a. Patient who was extubated this morning and has a temperature of 101.4°F (38.6°C)
b. Patient with bilevel positive airway pressure (BiPAP) for obstructive sleep apnea and a respiratory rate of 16
c. Patient with arterial pressure monitoring who is 2 hours post–percutaneous coronary intervention and needs to void
d. Patient who is receiving IV heparin for a venous thromboembolism and has a partial thromboplastin time (PTT) of 101 sec

A

ANS: D
The findings for this patient indicate high risk for bleeding from an elevated (nontherapeutic) PTT. The nurse needs to adjust the rate of the infusion (dose) per the health care provider’s parameters. The patient with BiPAP for sleep apnea has a normal respiratory rate. The patient recovering from the percutaneous coronary intervention will need to be assisted with voiding and this task could be delegated to unlicensed assistive personnel. The patient with a fever may be developing ventilator-associated pneumonia, but addressing the bleeding risk is a higher priority.

How well did you know this?
1
Not at all
2
3
4
5
Perfectly
111
Q

A patient’s vital signs are pulse 90, respirations 24, and BP 128/64 mm Hg, and cardiac output is 4.7 L/min. The patient’s stroke volume is _____ mL. (Round to the nearest whole number.)

A

ANS:
52

Stroke volume = Cardiac output/heart rate
52 mL = (4.7 L x 1000 mL/L)/90

How well did you know this?
1
Not at all
2
3
4
5
Perfectly
112
Q

When assisting with oral intubation of a patient who is having respiratory distress, in which order will the nurse take these actions? (Put a comma and a space between each answer choice [A, B, C, D, E].)

a. Obtain a portable chest-x-ray.
b. Position the patient in the supine position.
c. Inflate the cuff of the endotracheal tube after insertion.
d. Attach an end-tidal CO2 detector to the endotracheal tube.
e. Oxygenate the patient with a bag-valve-mask device for several minutes.

A

ANS:
E, B, C, D, A

The patient is pre-oxygenated with a bag-valve-mask system for 3 to 5 minutes before intubation and then placed in a supine position. After the intubation, the cuff on the endotracheal tube is inflated to occlude and protect the airway. Tube placement is assessed first with an end-tidal CO2 sensor and then with chest x-ray examination.

How well did you know this?
1
Not at all
2
3
4
5
Perfectly
113
Q

The nurse is caring for a patient who has an intraaortic balloon pump (IABP) after a massive heart attack. When assessing the patient, the nurse notices blood backing up into the IABP catheter. In which order should the nurse take the following actions? (Put a comma and a space between each answer choice [A, B, C, D].)

a. Confirm that the IABP console has turned off.
b. Assess the patient’s vital signs and orientation.
c. Obtain supplies for insertion of a new IABP catheter.
d. Notify the health care provider of the IABP malfunction.

A

ANS:
A, B, D, C

Blood in the IABP catheter indicates a possible tear in the balloon. The console should shut off automatically to prevent complications such as air embolism. Next, the nurse will assess the patient and communicate with the health care provider about the patient’s assessment and the IABP problem. Finally, supplies for insertion of a new IABP catheter may be needed based on the patient assessment and the decision of the health care provider.

How well did you know this?
1
Not at all
2
3
4
5
Perfectly
114
Q

Which diagnostic test will provide the nurse with the most specific information to evaluate the effectiveness of interventions for a patient with ventilatory failure?

a. Chest x-ray
b. O2 saturation
c. Arterial blood gas analysis
d. Central venous pressure monitoring

A

ANS: C
Arterial blood gas (ABG) analysis is most useful in this setting because ventilatory failure causes problems with CO2 retention, and ABGs provide information about the PaCO2 and pH. The other tests may also be done to help in assessing oxygenation or determining the cause of the patient’s ventilatory failure.

How well did you know this?
1
Not at all
2
3
4
5
Perfectly
115
Q

While caring for a patient who has been admitted with a pulmonary embolism, the nurse notes a change in the patient’s oxygen saturation (SpO2) from 94% to 88%. Which action should the nurse take?

a. Suction the patient’s oropharynx.
b. Increase the prescribed O2 flow rate.
c. Instruct the patient to cough and deep breathe.
d. Help the patient to sit in a more upright position.

A

ANS: B
Increasing O2 flow rate will usually improve O2 saturation in patients with ventilation-perfusion mismatch, as occurs with pulmonary embolism. Because the problem is with perfusion, actions that improve ventilation, such as deep breathing and coughing, sitting upright, and suctioning, are not likely to improve oxygenation.

How well did you know this?
1
Not at all
2
3
4
5
Perfectly
116
Q

A patient with respiratory failure has a respiratory rate of 6 breaths/min and an oxygen saturation (SpO2) of 88%. The patient is increasingly lethargic. Which intervention will the nurse anticipate?

a. Administration of 100% O2 by non-rebreather mask
b. Endotracheal intubation and positive pressure ventilation
c. Insertion of a mini-tracheostomy with frequent suctioning
d. Initiation of continuous positive pressure ventilation (CPAP)

A

ANS: B
The patient’s lethargy, low respiratory rate, and SpO2 indicate the need for mechanical ventilation with ventilator-controlled respiratory rate. Giving high-flow O2 will not be helpful because the patient’s respiratory rate is so low. Insertion of a mini-tracheostomy will facilitate removal of secretions, but it will not improve the patient’s respiratory rate or oxygenation. CPAP requires that the patient initiate an adequate respiratory rate to allow adequate gas exchange.

How well did you know this?
1
Not at all
2
3
4
5
Perfectly
117
Q

The oxygen saturation (SpO2) for a patient with left lower lobe pneumonia is 90%. The patient has wheezes, a weak cough effort, and complains of fatigue. Which action should the nurse take next?

a. Position the patient on the left side.
b. Assist the patient with staged coughing.
c. Place a humidifier in the patient’s room.
d. Schedule a 4-hour rest period for the patient.

A

ANS: B
The patient’s assessment indicates that assisted coughing is needed to help remove secretions, which will improve oxygenation. A 4-hour rest period at this time may allow the O2 saturation to drop further. Humidification will not be helpful unless the secretions can be mobilized. Positioning on the left side may cause a further decrease in oxygen saturation because perfusion will be directed more toward the more poorly ventilated lung.

How well did you know this?
1
Not at all
2
3
4
5
Perfectly
118
Q

A nurse is caring for an obese patient with right lower lobe pneumonia. Which position will be best to improve gas exchange?

a. On the left side c. In the tripod position
b. On the right side d. In the high-Fowler’s position

A

ANS: A
The patient should be positioned with the “good” lung in the dependent position to improve the match between ventilation and perfusion. The obese patient’s abdomen will limit respiratory excursion when sitting in the high-Fowler’s or tripod positions.

How well did you know this?
1
Not at all
2
3
4
5
Perfectly
119
Q

When admitting a patient with possible respiratory failure and a high PaCO2, which assessment information should be immediately reported to the health care provider?

a. The patient is very somnolent.
b. The patient complains of weakness.
c. The patient’s blood pressure is 164/98.
d. The patient’s oxygen saturation is 90%.

A

ANS: A
Increasing somnolence will decrease the patient’s respiratory rate and further increase the PaCO2 and respiratory failure. Rapid action is needed to prevent respiratory arrest. An SpO2 of 90%, weakness, and elevated blood pressure all require ongoing monitoring but are not indicators of possible impending respiratory arrest.

How well did you know this?
1
Not at all
2
3
4
5
Perfectly
120
Q

A patient with acute respiratory distress syndrome (ARDS) and acute kidney injury has the following drugs ordered. Which drug should the nurse discuss with the health care provider before giving?

a. gentamicin 60 mg IV
b. pantoprazole (Protonix) 40 mg IV
c. sucralfate (Carafate) 1 g per nasogastric tube
d. methylprednisolone (Solu-Medrol) 60 mg IV

A

ANS: A
Gentamicin, which is one of the aminoglycoside antibiotics, is potentially nephrotoxic, and the nurse should clarify the drug and dosage with the health care provider before administration. The other drugs are appropriate for the patient with ARDS.

How well did you know this?
1
Not at all
2
3
4
5
Perfectly
121
Q

A patient develops increasing dyspnea and hypoxemia 2 days after heart surgery. To determine whether the patient has acute respiratory distress syndrome (ARDS) or pulmonary edema caused by heart failure, the nurse will plan to assist with

a. obtaining a ventilation-perfusion scan.
b. drawing blood for arterial blood gases.
c. positioning the patient for a chest x-ray.
d. insertion of a pulmonary artery catheter.

A

ANS: D
Pulmonary artery wedge pressures are normal in the patient with ARDS because the fluid in the alveoli is caused by increased permeability of the alveolar-capillary membrane rather than by the backup of fluid from the lungs (as occurs in cardiogenic pulmonary edema). The other tests will not help in differentiating cardiogenic from noncardiogenic pulmonary edema.

How well did you know this?
1
Not at all
2
3
4
5
Perfectly
122
Q

A nurse is caring for a patient with ARDS who is being treated with mechanical ventilation and high levels of positive end-expiratory pressure (PEEP). Which assessment finding by the nurse indicates that the PEEP may need to be reduced?

a. The patient’s PaO2 is 50 mm Hg and the SaO2 is 88%.
b. The patient has subcutaneous emphysema on the upper thorax.
c. The patient has bronchial breath sounds in both the lung fields.
d. The patient has a first-degree atrioventricular heart block with a rate of 58 beats/min.

A

ANS: B
The subcutaneous emphysema indicates barotrauma caused by positive pressure ventilation and PEEP. Bradycardia, hypoxemia, and bronchial breath sounds are all concerns and will need to be addressed, but they are not specific indications that PEEP should be reduced

How well did you know this?
1
Not at all
2
3
4
5
Perfectly
123
Q

Which statement by the nurse when explaining the purpose of positive end-expiratory pressure (PEEP) to the patient’s caregiver is accurate?

a. “PEEP will push more air into the lungs during inhalation.”
b. “PEEP prevents the lung air sacs from collapsing during exhalation.”
c. “PEEP will prevent lung damage while the patient is on the ventilator.”
d. “PEEP allows the breathing machine to deliver 100% O2 to the lungs.”

A

ANS: B
By preventing alveolar collapse during expiration, PEEP improves gas exchange and oxygenation. PEEP will not prevent lung damage (e.g., fibrotic changes that occur with ARDS), push more air into the lungs, or change the fraction of inspired oxygen (FIO2) delivered to the patient.

How well did you know this?
1
Not at all
2
3
4
5
Perfectly
124
Q

When prone positioning is used for a patient with acute respiratory distress syndrome (ARDS), which information obtained by the nurse indicates that the positioning is effective?

a. The patient’s PaO2 is 89 mm Hg, and the SaO2 is 91%.
b. Endotracheal suctioning results in clear mucous return.
c. Sputum and blood cultures show no growth after 48 hours.
d. The skin on the patient’s back is intact and without redness.

A

ANS: A
The purpose of prone positioning is to improve the patient’s oxygenation as indicated by the PaO2 and SaO2. The other information will be collected but does not indicate whether prone positioning has been effective.

How well did you know this?
1
Not at all
2
3
4
5
Perfectly
125
Q

The nurse assesses vital signs for a patient admitted 2 days ago with gram-negative sepsis: temperature of 101.2° F, blood pressure of 90/56 mm Hg, pulse of 92 beats/min, and respirations of 34 breaths/min. Which action should the nurse take next?

a. Give the scheduled IV antibiotic.
b. Give the PRN acetaminophen (Tylenol).
c. Obtain oxygen saturation using pulse oximetry.
d. Notify the health care provider of the patient’s vital signs.

A

ANS: C
The patient’s increased respiratory rate in combination with the admission diagnosis of gram-negative sepsis indicates that acute respiratory distress syndrome (ARDS) may be developing. The nurse should check for hypoxemia, a hallmark of ARDS. The health care provider should be notified after further assessment of the patient. Giving the scheduled antibiotic and the PRN acetaminophen will also be done, but they are not the highest priority for a patient who may be developing ARDS.

How well did you know this?
1
Not at all
2
3
4
5
Perfectly
126
Q

A nurse is caring for a patient who is orally intubated and receiving mechanical ventilation. To decrease the risk for ventilator-associated pneumonia, which action will the nurse include in the plan of care?

a. Elevate head of bed to 30 to 45 degrees.
b. Give enteral feedings at no more than 10 mL/hr.
c. Suction the endotracheal tube every 2 to 4 hours.
d. Limit the use of positive end-expiratory pressure.

A

ANS: A
Elevation of the head decreases the risk for aspiration. Positive end-expiratory pressure is frequently needed to improve oxygenation in patients receiving mechanical ventilation. Suctioning should be done only when the patient assessment indicates that it is necessary. Enteral feedings should provide adequate calories for the patient’s high energy needs.

How well did you know this?
1
Not at all
2
3
4
5
Perfectly
127
Q

A patient admitted with acute respiratory failure has ineffective airway clearance related to thick secretions. Which nursing intervention would specifically address this patient problem?

a. Encourage use of the incentive spirometer.
b. Offer the patient fluids at frequent intervals.
c. Teach the patient the importance of ambulation.
d. Titrate oxygen level to keep O2 saturation above 93%.

A

ANS: B
Because the reason for the poor airway clearance is the thick secretions, the best action will be to encourage the patient to improve oral fluid intake. Patients should be instructed to use the incentive spirometer on a regular basis (e.g., every hour) to facilitate the clearance of the secretions. The other actions may also be helpful in improving the patient’s gas exchange, but they do not address the thick secretions that are causing the poor airway clearance.

How well did you know this?
1
Not at all
2
3
4
5
Perfectly
128
Q

A patient with acute respiratory distress syndrome (ARDS) who is intubated and receiving mechanical ventilation develops a right pneumothorax. Which collaborative action will the nurse anticipate next?

a. Increase the tidal volume and respiratory rate.
b. Decrease the fraction of inspired oxygen (FIO2).
c. Perform endotracheal suctioning more frequently.
d. Lower the positive end-expiratory pressure (PEEP).

A

ANS: D
Because barotrauma is associated with high airway pressures, the level of PEEP should be decreased. The other actions will not decrease the risk for another pneumothorax.

How well did you know this?
1
Not at all
2
3
4
5
Perfectly
129
Q

After receiving change-of-shift report on a medical unit, which patient should the nurse assess first?

a. A patient with cystic fibrosis who has thick, green-colored sputum
b. A patient with pneumonia who has crackles bilaterally in the lung bases
c. A patient with emphysema who has an oxygen saturation of 90% to 92%
d. A patient with septicemia who has intercostal and suprasternal retractions

A

ANS: D
This patient’s history of septicemia and labored breathing suggest the onset of ARDS, which will require rapid interventions such as administration of O2 and use of positive-pressure ventilation. The other patients should also be assessed, but their assessment data are typical of their disease processes and do not suggest deterioration in their status.

How well did you know this?
1
Not at all
2
3
4
5
Perfectly
130
Q

A patient with chronic obstructive pulmonary disease (COPD) arrives in the emergency department complaining of shortness of breath and dyspnea on minimal exertion. Which assessment finding by the nurse is most important to report to the health care provider?

a. The patient has bibasilar lung crackles.
b. The patient is sitting in the tripod position.
c. The patient’s pulse oximetry indicates a 91% O2 saturation.
d. The patient’s respirations have dropped to 10 breaths/minute.

A

ANS: D
A drop in respiratory rate in a patient with respiratory distress suggests the onset of fatigue and a high risk for respiratory arrest. Therefore immediate action such as positive-pressure ventilation is needed. Patients who are experiencing respiratory distress frequently sit in the tripod position because it decreases the work of breathing. Crackles in the lung bases may be the baseline for a patient with COPD. An O2 saturation of 91% is common in patients with COPD and will provide adequate gas exchange and tissue oxygenation.

How well did you know this?
1
Not at all
2
3
4
5
Perfectly
131
Q

When assessing a patient with chronic obstructive pulmonary disease (COPD), the nurse finds a new onset of agitation and confusion. Which action should the nurse take first?

a. Observe for facial symmetry.
b. Notify the health care provider.
c. Attempt to calm and reorient the patient.
d. Assess oxygenation using pulse oximetry.

A

ANS: D
Because agitation and confusion are frequently the initial indicators of hypoxemia, the nurse’s initial action should be to assess O2 saturation. The other actions are also appropriate, but assessment of oxygenation takes priority over other assessments and notification of the health care provider.

How well did you know this?
1
Not at all
2
3
4
5
Perfectly
132
Q

The nurse is caring for a patient who arrived in the emergency department with acute respiratory distress. Which assessment finding by the nurse requires the most rapid action?

a. The patient’s PaO2 is 45 mm Hg.
b. The patient’s PaCO2 is 33 mm Hg.
c. The patient’s respirations are shallow.
d. The patient’s respiratory rate is 32 breaths/min.

A

ANS: A
The PaO2 indicates severe hypoxemia and respiratory failure. Rapid action is needed to prevent further deterioration of the patient. Although the shallow breathing, rapid respiratory rate, and low PaCO2 also need to be addressed, the most urgent problem is the patient’s poor oxygenation.

How well did you know this?
1
Not at all
2
3
4
5
Perfectly
133
Q

The nurse is caring for an older patient who was hospitalized 2 days earlier with community-acquired pneumonia. Which assessment information is most important to communicate to the health care provider?

a. Persistent cough of blood-tinged sputum.
b. Scattered crackles in the posterior lung bases.
c. Oxygen saturation 90% on 100% O2 by nonrebreather mask.
d. Temperature 101.5° F (38.6° C) after 2 days of IV antibiotics.

A

ANS: C
The patient’s low SpO2 despite receiving a high fraction of inspired oxygen (FIO2) indicates the possibility of acute respiratory distress syndrome (ARDS). The patient’s blood-tinged sputum and scattered crackles are not unusual in a patient with pneumonia, although they do require continued monitoring. The continued temperature elevation indicates a possible need to change antibiotics, but this is not as urgent a concern as the progression toward hypoxemia despite an increase in O2 flow rate.

How well did you know this?
1
Not at all
2
3
4
5
Perfectly
134
Q

Which nursing interventions included in the care of a mechanically ventilated patient with acute respiratory failure can the registered nurse (RN) delegate to an experienced licensed practical/vocational nurse (LPN/LVN) working in the intensive care unit?

a. Assess breath sounds every hour.
b. Monitor central venous pressures.
c. Place patient in the prone position.
d. Insert an indwelling urinary catheter.

A

ANS: D
Insertion of indwelling urinary catheters is included in LPN/LVN education and scope of practice and can be safely delegated to an LPN/LVN who is experienced in caring for critically ill patients. Placing a patient who is on a ventilator in the prone position requires multiple staff, and should be supervised by an RN. Assessment of breath sounds and obtaining central venous pressures require advanced assessment skills and should be done by the RN caring for a critically ill patient.

How well did you know this?
1
Not at all
2
3
4
5
Perfectly
135
Q

A nurse is caring for a patient with acute respiratory distress syndrome (ARDS) who is receiving mechanical ventilation using synchronized intermittent mandatory ventilation (SIMV). The settings include fraction of inspired oxygen (FIO2) of 80%, tidal volume of 450, rate of 16/minute, and positive end-expiratory pressure (PEEP) of 5 cm. Which assessment finding is most important for the nurse to report to the health care provider?

a. O2 saturation of 99% c. Crackles audible at lung bases
b. Heart rate 106 beats/minute d. Respiratory rate 22 breaths/minute

A

ANS: A
The FIO2 of 80% increases the risk for O2 toxicity. Because the patient’s O2 saturation is 99%, a decrease in FIO2 is indicated to avoid toxicity. The other patient data would be typical for a patient with ARDS and would not be the most important data to report to the health care provider.

How well did you know this?
1
Not at all
2
3
4
5
Perfectly
136
Q

Which information about a patient who is receiving cisatracurium (Nimbex) to prevent asynchronous breathing with the positive pressure ventilator requires action by the nurse?

a. No sedative has been ordered for the patient.
b. The patient does not respond to verbal stimulation.
c. There is no cough or gag reflex when the patient is suctioned.
d. The patient’s oxygen saturation remains between 90% to 93%.

A

ANS: A
Because neuromuscular blockade is extremely anxiety provoking, it is essential that patients who are receiving neuromuscular blockade receive concurrent sedation and analgesia. Absence of response to stimuli is expected in patients receiving neuromuscular blockade. The O2 saturation is adequate.

How well did you know this?
1
Not at all
2
3
4
5
Perfectly
137
Q

The nurse is caring for a patient who is intubated and receiving positive pressure ventilation to treat acute respiratory distress syndrome (ARDS). Which finding is most important to report to the health care provider?

a. Red-brown drainage from nasogastric tube
b. Blood urea nitrogen (BUN) level 32 mg/dL
c. Scattered coarse crackles heard throughout lungs
d. Arterial blood gases: pH of 7.31, PaCO2 of 50, and PaO2 of 68

A

ANS: A
The nasogastric drainage indicates possible gastrointestinal bleeding or stress ulcer and should be reported. The pH and PaCO2 are slightly abnormal, but current guidelines advocating for permissive hypercapnia indicate that these would not indicate an immediate need for a change in therapy. The BUN is slightly elevated but does not indicate an immediate need for action. Adventitious breath sounds are commonly heard in patients with ARDS.

How well did you know this?
1
Not at all
2
3
4
5
Perfectly
138
Q

During change-of-shift report on a medical unit, the nurse learns that a patient with aspiration pneumonia who was admitted with respiratory distress has become increasingly agitated. Which action should the nurse take first?

a. Give the prescribed PRN sedative drug.
b. Offer reassurance and reorient the patient.
c. Use pulse oximetry to check the oxygen saturation.
d. Notify the health care provider about the patient’s status.

A

ANS: C
Agitation may be an early indicator of hypoxemia. The other actions may also be appropriate, depending on the findings about O2 saturation.

How well did you know this?
1
Not at all
2
3
4
5
Perfectly
139
Q

The nurse reviews the electronic health record for a patient scheduled for a total hip replacement. Which assessment data shown in the accompanying figure increase the patient’s risk for respiratory complications after surgery?

a. Older age and anemia c. Recent arthroscopic procedure
b. Albumin level and weight loss d. Confusion and disorientation to time

A

ANS: B
The patient’s recent weight loss and low protein stores indicate possible muscle weakness, which make it more difficult for an older patient to recover from the effects of general anesthesia and immobility associated with the hip surgery. The other information will also be noted by the nurse but does not place the patient at higher risk for respiratory failure.

How well did you know this?
1
Not at all
2
3
4
5
Perfectly
140
Q

Which actions should the nurse start to reduce the risk for ventilator-associated pneumonia (VAP) (select all that apply)?

a. Obtain arterial blood gases daily.
b. Provide a “sedation holiday” daily.
c. Give prescribed pantoprazole (Protonix).
d. Elevate the head of the bed to at least 30°.
e. Provide oral care with chlorhexidine (0.12%) solution daily.

A

ANS: B, C, D, E
All of these interventions are part of the ventilator bundle that is recommended to prevent VAP. Arterial blood gases may be done daily but are not always necessary and do not help prevent VAP.

How well did you know this?
1
Not at all
2
3
4
5
Perfectly
141
Q

A registered nurse (RN) is the group leader of practical nurses and nursing assistive personnel. Which nursing care model is the RN using?

a. Case management
b. Total patient care
c. Primary nursing
d. Team nursing

A

ANS: D
In team nursing, the RN assumes the role of group or team leader and leads a team made up of other RNs, practical nurses, and nursing assistive personnel. Case management is a care approach that coordinates and links health care services to patients and families while streamlining costs. Total patient care involves an RN being responsible for all aspects of care for one or more patients. The primary nursing model of care delivery was developed to place RNs at the bedside and improve the accountability of nursing for patient outcomes and the professional relationships among staff members.

How well did you know this?
1
Not at all
2
3
4
5
Perfectly
142
Q

A nurse is overseeing the care of patients with severe diabetes and patients with heart failure to improve cost-effectiveness and quality of care. Which nursing care delivery model is the nurse using?

a. Team nursing
b. Total patient care
c. Primary nursing
d. Case management

A

ANS: D
Case management is unique because clinicians, either as individuals or as part of a collaborative group, oversee the management of patients with specific, complex health problems or are held accountable for some standard of cost management and quality. Case management is a care approach that coordinates and links health care services to patients and families while streamlining costs. In the team nursing care model, the RN assumes the role of group or team leader and leads a team made up of other RNs, practical nurses, and nursing assistive personnel. Total patient care involves an RN being responsible for all aspects of care for one or more patients. The primary nursing model of care delivery was developed to place RNs at the bedside and improve the accountability of nursing for patient outcomes and the professional relationships among staff members.

How well did you know this?
1
Not at all
2
3
4
5
Perfectly
143
Q

A nurse is working in an intensive care unit (critical care). Which type of nursing care delivery model will this nurse most likely use?
a. Team nursing

b. Total patient care
c. Primary nursing
d. Case-management

A

ANS: B
Total patient care is found primarily in critical care areas. Total patient care involves an RN being responsible for all aspects of care for one or more patients. In the team nursing care model, the RN assumes the role of group or team leader and leads a team made up of other RNs, practical nurses, and nursing assistive personnel. The primary nursing model of care delivery was developed to place RNs at the bedside and improve the accountability of nursing for patient outcomes and the professional relationships among staff members. Casemanagement is a care approach that coordinates and links health care services to patients and families while streamlining costs.

144
Q

. A nurse manager discovers that the readmission rate of hospitalized patients is very high on the hospital unit. The nurse manager desires improved coordination of care and accountability for cost-effective quality care. Which nursing care delivery model is best suited for these needs?

a. Team nursing
b. Total patient care
c. Primary nursing
d. Case management

A

ANS: D
Case management is a care approach that coordinates and links health care services to patients and families while streamlining costs. In team nursing, the RN assumes the role of group or team leader and leads a team made up of other RNs, practical nurses, and nursing assistive personnel. Total patient care involves an RN being responsible for all aspects of care for one or more patients. The primary nursing model of care delivery was developed to place RNs at the bedside and improve the accountability of nursing for patient outcomes and the professional relationships among staff members.

145
Q

A nurse is working in a facility that has fewer directors with managers and staff able to make shared decisions. In which type of organizational structure is the nurse employed?

a. Delegation
b. Research-based
c. Decentralization
d. Philosophy of care

A

ANS: C
The decentralized management structure often has fewer directors, and managers and staff are able to make shared decisions. The American Nurses Association defines delegation as transferring responsibility for the performance of an activity or task while retaining accountability for the outcome. Research-based means care is based upon evidence. A philosophy of care includes the professional nursing staff’s values and concerns for the way they view and care for patients. For example, a philosophy addresses the purpose of the nursing unit, how staff works with patients and families, and the standards of care for the work unit.

146
Q

A staff member verbalizes satisfaction in working on a particular nursing unit because of the freedom of choice and responsibility for the choices. This nurse highly values which element of shared decision making?

a. Authority
b. Autonomy
c. Responsibility
d. Accountability

A

ANS: B
Autonomy is freedom of choice and responsibility for the choices. Authority refers to legitimate power to give commands and make final decisions specific to a given position. Responsibility refers to the duties and activities that an individual is employed to perform. Accountability refers to individuals being answerable for their actions.

147
Q

A staff nurse delegates a task to a nursing assistive personnel (NAP), knowing that the NAP has never performed the task before. As a result, the patient is injured, and the nurse defensively states that the NAP should have known how to perform such a simple task. Which element of the decision-making process is the nurse lacking?

a. Authority
b. Autonomy
c. Responsibility
d. Accountability

A

ANS: D
Accountability refers to individuals being answerable for their actions. The nurse in this situation is not taking ownership of the inappropriate delegation of a task. Autonomy is freedom of choice and responsibility for the choices. Responsibility refers to the duties and activities that an individual is employed to perform. Authority refers to legitimate power to give commands and make final decisions specific to a given position.

148
Q

A nurse manager sent one of the staff nurses on the unit to a conference about new, evidence-based wound care techniques. The nurse manager asks the staff nurse to prepare a poster to present at the next unit meeting, which will be mandatory for all nursing staff on the unit. Which type of opportunity is the nurse manager providing for the staff?

a. Staff education
b. Interprofessional collaboration
c. Providing a professional shared governance council
d. Establishing a nursing practice committee

A

ANS: A
The nurse manager is planning a staff education opportunity. Staff education is one way the nurse manager supports staff involvement in a shared decision-making model. Interprofessional collaboration between nurses and health care providers (e.g., MD, PT, TR, etc.) is critical to the delivery of quality, safe patient care and the creation of a positive work culture for practitioners. The question does not state that the nurse is establishing a practice committee or a professional shared governance council. Chaired by senior clinical staff nurses, these groups establish and maintain care standards for nursing practice on their work unit.

149
Q

A nurse is making a home visit and discovers that a patient’s wound infection has gotten worse. The nurse cleans and redresses the wound. What should the nurse do next?

a. Notify the health care provider of the findings before leaving the home.
b. Ask the home health facility nurse manager to contact the health care provider.
c. Document the findings and confirm with the patient the date of the next home visit.
d. Tell the patient that the health care provider will be notified before the next home visit.

A

ANS: A
The nurse should notify the health care provider before leaving the home. Regardless of the setting, an enriching professional environment is one in which staff members respect one another’s ideas, share information, and keep one another informed. The manager should avoid taking care of problems for staff. The staff nurse needs to learn how to professionally communicate with other members of the health care team and demonstrate interprofessional collaboration.

150
Q

A nurse manager conducts rounds on the unit and discovers that expired stock medicine is still in the cabinet despite the e-mail that was sent stating that it had to be discarded. The staff nurse dress code is not being adhered to as requested in the same e-mail. Several staff nurses deny having received the e-mail. Which action should the nurse manager take?

a. Close the staff lounge.
b. Enforce a stricter dress code.
c. Include the findings on each staff member’s annual evaluation.
d. Place a hard copy of announcements and unit policies in each staff member’s mailbox.

A

ANS: D
The identified problem is lack of staff communication. Sending an e-mail was not effective; therefore, giving each staff member a hard copy along with e-mailing is another approach the manager can take. An effective manager uses a variety of approaches to communicate quickly and accurately to all staff. For example, many managers distribute biweekly or monthly newsletters of ongoing unit or facility activities. Including the findings on evaluations, closing the lounge, and enforcing stricter dress codes do not address the problem.

151
Q

A new nurse expresses frustration at not being to complete all interventions for a group of patients in a timely manner. The nurse leaves the rounds report sheets at the nurse’s station when caring for patients and reports having to go back and forth between rooms for equipment and supplies. Which type of skill does the nurse need?

a. Interpersonal communication
b. Clinical decision making
c. Organizational
d. Evaluation

A

ANS: C
The clinical care coordination skill this nurse needs to improve on is organization. This nurse needs to keep the patient report sheets in hand to anticipate what equipment and supplies a patient is going to need. Then the nurse may not have to leave the room so often; this will save time. The nurse is not having a problem communicating with others (interpersonal communication). The nurse is not having a problem using the nursing process for clinical decisions. The nurse is not having a problem comparing actual patient outcomes with expected outcomes (evaluation).

152
Q

Which approach will be most appropriate for a nurse to take when faced with the challenge of performing many tasks in one shift?

a. Do as much as possible by oneself before seeking assistance from others.
b. Evaluate the effectiveness of all tasks when all tasks are completed.
c. Complete one task before starting another task.
d. Delegate tasks the nurse does not like doing.

A

ANS: C
The appropriate clinical care coordination skill in these options is to complete one task before starting another task. Good time management involves setting goals to help the nurse complete one task before starting another task. Evaluation is ongoing and should not be completed just at the end of task completion. The nurse should not delegate tasks simply because the nurse does not like doing them. The nurse should use delegation skills and time-management skills instead of trying to do as much as possible with no help.

153
Q

Which assessment of a patient who is 1 day postsurgery to repair a hip fracture requires immediate nursing intervention?

a. Patient ate 40% of clear liquid breakfast.
b. Patient’s oral temperature is 98.9° F.
c. Patient states, “I did not realize I would be so tired after this surgery.”
d. Patient reports severe pain 30 minutes after receiving pain medication

A

ANS: D
It is important to prioritize in all caregiving situations because it allows you to see relationships among patient problems and avoid delays in taking action that possibly leads to serious complications for a patient. The nurse needs to report severe pain that is unrelieved by pain medication to the health care provider. The nurse needs to recognize and differentiate normal from abnormal findings and set priorities. Eating 40% of breakfast, having a slightly elevated temperature, and being tired the day after surgery are expected findings following surgery and do not require immediate intervention.

154
Q

A nurse has a transactional leader as a manager. Which finding will the nurse anticipate from working with this leader?

a. Increased turnover rate
b. Increased patient mortality rate
c. Increased rate of medication errors
d. Increased level of patient satisfaction

A

ANS: D
Research has found that on nursing units where the nurse manager uses transactional leadership there is an increased level of patient satisfaction, a lower patient mortality rate, and a lower rate of medication errors. Turnover rate is decreased since staff retention is increased with transformational leadership.

155
Q

A patient with an indwelling urinary catheter has been given a bed bath by a new nursing assistive personnel. The nurse evaluating the cleanliness of the patient notices crusting at the urinary meatus. Which action should the nurse take next?

a. Ask the nursing assistive personnel to observe while the nurse performs catheter care.
b. Leave the room and ask the nursing assistive personnel to go back and perform proper catheter care.
c. Tell the nursing assistive personnel that catheter care is sloppy.
d. Remove the catheter.

A

ANS: A
If the staff member’s performance is not satisfactory, give constructive and appropriate feedback. You may discover the need to review a procedure with staff and offer demonstration. Because the nursing assistant is new, it is best for the nurse to perform catheter care while the assistant observes. This action will ensure that the assistant has been shown the proper way to perform the task and fosters collaboration rather than leaving the room just to tell the assistant to come back. Telling that catheter care is sloppy does not correct the problem. The catheter does not need to be removed.

156
Q

. A nurse is prioritizing care for four patients. Which patient should the nurse see first?

a. A patient needing teaching about medications
b. A patient with a healed abdominal incision
c. A patient with a slight temperature
d. A patient with difficulty breathing

A

ANS: D
An immediate threat to a patient’s survival or safety must be addressed first, like difficulty breathing. Teaching, healed incision, and slight temperature are not immediate needs.

157
Q

A nursing assistive personnel (NAP) reports seeing a reddened area on the patient’s hip while bathing the patient. Which action should the nurse take?

a. Request a wound nurse consult.
b. Go to the patient’s room to assess the patient’s skin.
c. Document the finding per the NAP’s report.
d. Ask the NAP to apply a dressing over the reddened area.

A

ANS: B
The nurse needs to assess the patient’s skin. Assessment should not be delegated; it is the responsibility of the licensed registered nurse. The nurse needs to document the assessment findings objectively, not subjectively, per the nursing assistive personnel. Before requesting a consult or determining treatment, the nurse needs to assess the skin.

158
Q

. A nurse is assigned to care for the following patients who all need vital signs taken right now. Which patient is most appropriate for the nurse to delegate vital sign measurement to the nursing assistive personnel (NAP)?

a. Patient scheduled for a procedure in the nuclear medicine department
b. Patient transferring from the intensive care unit (ICU)
c. Patient returning from a cardiac catheterization
d. Patient returning from hip replacement surgery

A

ANS: A
The nurse does not assign vital sign measurement or other tasks to NAP when patients are experiencing a change in level of care. The patient awaiting the procedure in nuclear medicine is the only patient who has not experienced a change in level of care. According to the rights of delegation, tasks that are repetitive, require little supervision, are relatively noninvasive, have results that are predictable, and have minimal risk can be delegated to assistive personnel. The patient in this question with the most predictable condition is the patient awaiting the nuclear medicine procedure. Once the nurse determines that the other patients are stable, the nurse could delegate their future vital sign measurement to the NAP. However, it is important for the nurse to assess patients coming from the ICU, the cardiac cath lab, and surgery when they first arrive on the unit.

159
Q

Which staff member does the nurse assign to provide morning care for an older-adult patient who requires assistance with activities of daily living?

a. Licensed practical nurse
b. Cardiac monitor technician
c. Nursing assistive personnel (NAP)
d. Another registered nurse on the floor

A

ANS: C
The NAP is capable of caring for this patient and is the most cost-effective choice. The cardiac monitor technician’s role is to watch the cardiac monitors for patients on the floor. The nurse and the licensed practical nurse are not the most cost-effective options in this case, even though each could assist with activities of daily living. These nurses would be better used to administer medications, perform assessments, etc.

160
Q

A nurse uses the five rights of delegation when providing care. Which “rights” did the nurse use? (Select all that apply.)

a. Right task
b. Right person
c. Right direction
d. Right supervision
e. Right circumstances
f. Right cost-effectiveness

A

ANS: A, B, C, D, E
The five rights of delegation are right task, circumstances, person, direction, and supervision. Cost-effectiveness is not a right.

161
Q

A nurse is prioritizing care. Match the level of priority to the patients.
a. Patient that needs to be turned to prevent pneumonia

b. Patient with acute asthma attack
c. Patient who will be discharged in 2 days who needs teaching

  1. High priority
  2. Intermediate priority
  3. Low priority
A

ANS: 1. B, 2. A, 3. C

162
Q

Four patients in labor all request epidural analgesia to manage their pain at the same time. Which ethical principle is most compromised when only one nurse anesthetist is on call?

a. Justice
b. Fidelity
c. Beneficence
d. Nonmaleficence

A

ANS: A
Justice refers to fairness and is used frequently in discussion regarding access to health care resources. Here the just distribution of resources, in this case pain management, cannot be justly apportioned. Nonmaleficence refers to avoidance of harm; beneficence refers to taking positive actions to help others. Fidelity refers to the agreement to keep promises. Each of these principles is partially expressed in the question; however, justice is most comprised because not all laboring patients have equal access to pain management owing to lack of personnel resources.

163
Q

The patient reports to the nurse of being afraid to speak up regarding a desire to end care for fear of upsetting spouse and children. Which principle in the nursing code of ethics ensures that the nurse will promote the patient’s cause?

a. Advocacy
b. Responsibility
c. Confidentiality
d. Accountability

A

ANS: A
Nurses advocate for patients when they support the patient’s cause. A nurse’s ability to adequately advocate for a patient is based on the unique relationship that develops and the opportunity to better understand the patient’s point of view. Responsibility refers to respecting one’s professional obligations and following through on promises. Confidentiality deals with privacy issues, and accountability refers to answering for one’s actions.

164
Q

The patient’s son requests to view documentation in the medical record. What is the nurse’s best response to this request?

a. “I’ll be happy to get that for you.”
b. “You are not allowed to look at it.”
c. “You will need your mother’s permission.”
d. “I cannot let you see the chart without a doctor’s order.”

A

ANS: C
The mother’s permission is needed. The nurse understands that sharing health information is governed by HIPAA legislation, which defines rights and privileges of patients for protection of privacy. Private health information cannot be shared without the patient’s specific permission. The nurse cannot obtain the records without permission. The son can look at it after approval from the patient. While talking to the physician or getting an order is appropriate, the patient still has to give consent.

165
Q

When professionals work together to solve ethical dilemmas, nurses must examine their own values. What is the best rationale for this step?

a. So fact is separated from opinion
b. So different perspectives are respected
c. So judgmental attitudes can be provoked
d. So the group identifies the one correct solution

A

ANS: B
Values are personal beliefs that influence behavior. To negotiate differences of value, it is important to be clear about your own values: what you value, why, and how you respect your own values even as you try to respect those of others whose values differ from yours. Ethical dilemmas are a problem in that no one right solution exists. It is not to separate fact from opinion. Judgmental attitudes are not to be used, much less provoked.

166
Q

A nurse is experiencing an ethical dilemma with a patient. Which information indicates the nurse has a correct understanding of the primary cause of ethical dilemmas?

a. Unequal power
b. Presence of conflicting values
c. Judgmental perceptions of patients
d. Poor communication with the patient

A

ANS: B
Ethical dilemmas almost always occur in the presence of conflicting values. While unequal power, judgmental perceptions, and poor communication can contribute to the dilemma, these are not causes of a dilemma. Without clarification of values, the nurse may not be able to distinguish fact from opinion or value, and this can lead to judgmental attitudes.

167
Q

. The nurse questions a health care provider’s decision to not tell the patient about a cancer diagnosis. Which ethical principle is the nurse trying to uphold for the patient?
a. Consequentialism

b. Autonomy
c. Fidelity
d. Justice

A

ANS: B
The nurse is upholding autonomy. Autonomy refers to the freedom to make decisions free of external control. Respect for patient autonomy refers to the commitment to include patients in decisions about all aspects of care. Consequentialism is focused on the outcome and is a philosophical approach. Justice refers to fairness and is most often used in discussions about access to health care resources. Fidelity refers to the agreement to keep promises.

168
Q

The nurse finds it difficult to care for a patient whose advance directive states that no extraordinary resuscitation measures should be taken. Which step may help the nurse to find resolution in this assignment?

a. Scrutinize personal values.
b. Call for an ethical committee consult.
c. Decline the assignment on religious grounds.
d. Convince the family to challenge the directive.

A

ANS: A
Clarifying values—your own, your patients’, your co-workers’—is an important and effective part of ethical discourse. Calling for a consult, declining the assignment, and convincing the family to challenge the patient’s directive are not ideal resolutions because they do not address the reason for the nurse’s discomfort, which is the conflict between the nurse’s values and those of the patient. The nurse should value the patient’s decisions over the nurse’s personal values.

169
Q

The nurse values autonomy above all other principles. Which patient assignment will the nurse find most difficult to accept?

a. Older-adult patient who requires dialysis
b. Teenager in labor who requests epidural anesthesia
c. Middle-aged father of three with an advance directive declining life support
d. Family elder who is making the decisions for a young-adult female member

A

ANS: D
Autonomy refers to freedom from external control. A person who values autonomy highly may find it difficult to accept situations where the patient is not the primary decision maker regarding his or her care. A teenager requesting an epidural, a father with an advance directive, and an elderly patient requiring dialysis all describe a patient or family who can make their own decisions and choices regarding care.

170
Q

A nurse must make an ethical decision concerning vulnerable patient populations. Which philosophy of health care ethics would be particularly useful for this nurse?

a. Teleology
b. Deontology
c. Utilitarianism
d. Feminist ethics

A

ANS: D
Feminist ethics particularly focuses on the nature of relationships, especially those where there is a power imbalance or a point of view that is ignored or invisible. Deontology refers to making decisions or “right-making characteristics,” bioethics focuses on consensus building, while utilitarianism and teleology speak to the greatest good for the greatest number.

171
Q

A nurse agrees with regulations for mandatory immunizations of children. The nurse believes that immunizations prevent diseases as well as prevent spread of the disease to others. Which ethical framework is the nurse using?

a. Deontology
b. Ethics of care
c. Utilitarianism
d. Feminist ethics

A

ANS: C
Utilitarianism is a system of ethics that believes that value is determined by usefulness. This system of ethics focuses on the outcome of the greatest good for the greatest number of people. Deontology would not look to consequences of actions but on the “right-making characteristic” such as fidelity and justice. The ethics of care emphasizes the role of feelings. Relationships, which are an important component of feminist ethics, are not addressed in this case.

172
Q

The nurse has become aware of missing narcotics in the patient care area. Which ethical principle obligates the nurse to report the missing medications?

a. Advocacy
b. Responsibility
c. Confidentiality
d. Accountability

A

ANS: B
Responsibility refers to one’s willingness to respect and adhere to one’s professional obligations. It is the nurse’s responsibility to report missing narcotics. Accountability refers to the ability to answer for one’s actions. Advocacy refers to the support of a particular cause. The concept of confidentiality is very important in health care and involves protecting patients’ personal health information.

173
Q

A young woman who is pregnant with a fetus exposed to multiple teratogens consents to have her fetus undergo serial PUBS (percutaneous umbilical blood sampling) to examine how exposure affects the fetus over time. Although these tests will not improve the fetus’s outcomes and will expose it to some risks, the information gathered may help infants in the future. Which ethical principle is at greatest risk?

a. Fidelity
b. Autonomy
c. Beneficence
d. Nonmaleficence

A

ANS: D
Nonmaleficence is the ethical principle that focuses on avoidance of harm or hurt. Repeated PUBS may expose the mother and fetus to some risks. Fidelity refers to the agreement to keep promises (obtain serial PUBS). Autonomy refers to freedom from external control (mother consented), and beneficence refers to taking positive actions to help others (may help infants in the future).

174
Q

A nurse is discussing quality of life issues with another colleague. Which topic will the nurse acknowledge for increased attention paid to quality of life concerns?

a. Health care disparities
b. Aging of the population
c. Abilities of disabled persons
d. Health care financial reform

A

ANS: C
The population of disabled persons in the United States and elsewhere has reshaped the discussion about quality of life (QOL). Health care disparities, an aging population, and health care reform are components impacted by personal definitions of quality but are not the underlying reason why QOL discussions have arisen.

175
Q

Which action by the nurse indicates a safe and efficient use of social networks?

a. Promotes support for a local health charity
b. Posts a picture of a patient’s infected foot
c. Vents about a patient problem at work
d. Friends a patient

A

ANS: A
Social networks can be a supportive source of information about patient care or professional nursing activities. Even if you post an image of a patient without any obvious identifiers, the nature of shared media reposting can result in the image surfacing in a place where just the context of the image provides clues for friends or family to identify the patient. The ANA and NCSBN states, “Effective nurse-patient relationships are built on trust. Patients need to be confident that their most personal information and their basic dignity will be protected by the nurse.” Becoming friends in online chat rooms, Facebook, or other public sites can interfere with your ability to maintain a therapeutic relationship

176
Q

The nurse is caring for a dying patient. Which intervention is considered futile?

a. Giving pain medication for pain
b. Providing oral care every 5 hours
c. Administering the influenza vaccine
d. Supporting lower extremities with pillows

A

ANS: C
Administering the influenza vaccine is futile. A vaccine is administered to prevent or lessen the likelihood of contracting an infectious disease at some time in the future. The term futile refers to something that is hopeless or serves no useful purpose. In health care discussions the term refers to interventions unlikely to produce benefit for a patient. Care delivered to a patient at the end of life that is focused on pain management, oral hygiene, and comfort measures is not futile.

177
Q

. During a severe respiratory epidemic, the local health care organizations decide to give health care workers priority access to ventilators over other members of the community who also need that resource. Which philosophy would give the strongest support for this decision?

a. Deontology
b. Utilitarianism
c. Ethics of care
d. Feminist ethics

A

ANS: B
Utilitarianism focuses on the greatest good for the most people; the organizations decide to ensure that as many health care workers as possible will survive to care for other members of the community. Deontology defines actions as right or wrong based on their “right-making characteristics” such as fidelity to promises, truthfulness, and justice. Feminist ethics looks to the nature of relationships to guide participants in making difficult decisions, especially relationships in which power is unequal or in which a point of view has become ignored or invisible. The ethics of care and feminist ethics are closely related, but ethics of care emphasizes the role of feelings.

178
Q

A nurse is teaching a patient and family about quality of life. Which information should the nurse include in the teaching session about quality of life?

a. It is deeply social.
b. It is hard to define.
c. It is an observed measurement for most people.
d. It is consistent and stable over the course of one’s lifetime.

A

ANS: B
Quality of life remains deeply individual (not social) and difficult to predict. Quality of life is not just a measurable entity but a shared responsibility. Quality of life measures may take into account the age of the patient, the patient’s ability to live independently, his or her ability to contribute to society in a gainful way, and other nuanced measures of quality.

179
Q

The nurse is caring for a patient supported with a ventilator who has been unresponsive since arrival via ambulance 8 days ago. The patient has not been identified, and no family members have been found. The nurse is concerned about the plan of care regarding maintenance or withdrawal of life support measures. Place the steps the nurse will use to resolve this ethical dilemma in the correct order.

  1. The nurse identifies possible solutions or actions to resolve the dilemma.
  2. The nurse reviews the medical record, including entries by all health care disciplines, to gather informationrelevant to this patient’s situation.
  3. Health care providers use negotiation to redefine the patient’s plan of care.
  4. The nurse evaluates the plan and revises it with input from other health care providers as necessary.
  5. The nurse examines the issue to clarify opinions, values, and facts.
  6. The nurse states the problem.

a. 6, 1, 2, 5, 4, 3
b. 5, 6, 2, 3, 4, 1
c. 1, 2, 5, 4, 3, 6
d. 2, 5, 6, 1, 3, 4

A

ANS: D
Step 1. Gather as much information as possible that is relevant to the case.
Step 2. Examine and determine your values about the issues.
Step 3. Verbalize the problem.
Step 4. Consider possible courses of action.
Step 5. Negotiate the outcome.
Step 6. Evaluate the action.

180
Q

A nurse is a member of the ethics committee. Which purposes will the nurse fulfill in this committee? (Select all that apply.)

a. Education
b. Case consultation
c. Purchasing power
d. Direct patient care
e. Policy recommendation

A

ANS: A, B, E
An ethics committee devoted to the teaching and processing of ethical issues and dilemmas exists in most health care facilities. It is generally multidisciplinary and it serves several purposes: education, policy recommendation, and case consultation. It does not have purchasing power or provide direct patient care.

181
Q

A newly hired experienced nurse is preparing to change a patient’s abdominal dressing and hasn’t done it before at this hospital. Which action by the nurse is best?

a. Have another nurse do it so the correct method can be viewed.
b. Change the dressing using the method taught in nursing school.
c. Ask the patient how the dressing change has been recently done.
d. Check the policy and procedure manual for the facility’s method.

A

ANS: D
The Joint Commission requires accredited hospitals to have written nursing policies and procedures. These internal standards of care are specific and need to be accessible on all nursing units. For example, a policy/procedure outlining the steps to follow when changing a dressing or administering medication provides specific information about how nurses are to perform. The nurse being observed may not be doing the procedure according to the facility’s policy or procedure. The procedure taught in nursing school may not be consistent with the policy or procedure for this facility. The patient is not responsible for maintaining the standards of practice.
Patient input is important, but it’s not what directs nursing practice.

182
Q

A new nurse notes that the health care unit keeps a listing of patient names in a closed book behind the front desk of the nursing station so patients can be located easily. Which action is most appropriate for the nurse to take?

a. Talk with the nurse manager about the listing being a violation of the Health Insurance Portability and Accountability Act (HIPAA).
b. Use the book as needed while keeping it away from individuals not involved in patient care.
c. Move the book to the upper ledge of the nursing station for easier access.
d. Ask the nurse manager to move the book to a more secluded area

A

ANS: B
The book is located where only staff would have access so the nurse can use the book as needed. The privacy section of the HIPAA provides standards regarding accountability in the health care setting. These rules include patient rights to consent to the use and disclosure of their protected health information, to inspect and copy their medical record, and to amend mistaken or incomplete information. It is not the responsibility of the new nurse to move items used by others on the patient unit. The listing is protected as long as it is used appropriately as needed to provide care. There is no need to move the book to a more secluded area.

183
Q

. A 17-year-old patient, dying of heart failure, wants to have organs removed for transplantation after death. Which action by the nurse is correct?

a. Instruct the patient to talk with parents about the desire to donate organs.
b. Notify the health care provider about the patient’s desire to donate organs.
c. Prepare the organ donation form for the patient to sign while still oriented.
d. Contact the United Network for Organ Sharing after talking with the patient.

A

ANS: A
In this situation, the parents would need to sign the form because the teenager is under age 18. An individual who is at least 18 may sign the form allowing organ donation upon death. The nurse cannot allow the patient to sign the organ donation document because the patient is younger than age 18. The health care provider will be notified about the patient’s wishes after the parents agree to donate the organs. The United Network for Organ Sharing (UNOS) has a contract with the federal government and sets policies and guidelines for the procurement of organs.

184
Q

An obstetric nurse comes across an automobile accident. The driver seems to have a crushed upper airway, and while waiting for emergency medical services to arrive, the nurse makes a cut in the trachea and inserts a straw from a purse to provide an airway. The patient survives and has a permanent problem with vocal cords, making it difficult to talk. Which statement is true regarding the nurse’s performance?

a. The nurse acted appropriately and saved the patient’s life.
b. The nurse stayed within the guidelines of the Good Samaritan Law.
c. The nurse took actions beyond those that are standard and appropriate.
d. The nurse should have just stayed with the patient and waited for help.

A

ANS: C
An obstetric nurse would not have been trained in performing a tracheostomy (cut in the trachea), and doing so would be beyond what the nurse has been trained or educated to do. If you perform a procedure exceeding your scope of practice and for which you have no training, you are liable for injury that may result from that act. You should only provide care that is consistent with your level of expertise. The nurse did not act appropriately. The nurse is not protected by the Good Samaritan Law because the nurse acted outside the scope of practice and training. The nurse should have acted within what was trained and educated to do in this circumstance, not just stay with the patient.

185
Q

A nurse performs cardiopulmonary resuscitation (CPR) on a 92-year-old with brittle bones and breaks a rib during the procedure, which then punctures a lung. The patient recovers completely without any residual problems and sues the nurse for pain and suffering and for malpractice. Which key point will the prosecution attempt to prove against the nurse?

a. The CPR procedure was done incorrectly.
b. The patient would have died if nothing was done.
c. The patient was resuscitated according to the policy.
d. The older patient with brittle bones might sustain fractures when chest compressions are done

A

ANS: A
Certain criteria are necessary to establish nursing malpractice. The prosecution would try to prove that a breach of duty had occurred (CPR done incorrectly), which had caused injury. The defense team, not the prosecution, would explain the correlation between brittle bones and rib fractures during CPR and that the patient was resuscitated according to policy. In this situation, although harm was caused, it was not because of failure of the nurse to perform a duty according to standards, the way other nurses would have performed in the same situation. The fact that the patient sustained injury as a result of age and physical status does not mean the nurse breached any duty to the patient. The nurse would need to make sure the defense attorney knew that the cardiopulmonary resuscitation (CPR) was done correctly. Without intervention, the patient most likely would not have survived.

186
Q

A recent immigrant who does not speak English is alert and requires hospitalization. What is the initial action that the nurse must take to enable informed consent to be obtained?

a. Ask a family member to translate what the nurse is saying.
b. Request an official interpreter to explain the terms of consent.
c. Notify the nursing manager that the patient doesn’t speak English.
d. Use hand gestures and medical equipment while explaining in English.

A

ANS: B
An official interpreter must be present to explain the terms of consent if a patient speaks only a foreign language. A family member or acquaintance who speaks a patient’s language should not interpret health information. Family members can tell those caring for the patient what the patient is saying, but privacy regarding the patient’s condition, assessment, etc., must be protected. A nurse can take care of requesting an interpreter, and the nurse manager is not needed. Using hand gestures and medical equipment is inappropriate when communicating with a patient who does not understand the language spoken. Certain hand gestures may be acceptable in one culture and not appropriate in another. The medical equipment may be unknown and frightening to the patient, and the patient still doesn’t understand what is being said.

187
Q

A pediatric oncology nurse floats to an orthopedic trauma unit. Which action should the nurse manager of the orthopedic unit take to enable safe care to be given by this nurse?

a. Provide a complete orientation to the functioning of the entire unit.
b. Determine patient acuity and care the nurse can safely provide.
c. Allow the nurse to choose which mealtime works best.
d. Assign nursing assistive personnel to assist with care.

A

ANS: B
Supervisors are liable if they give staff nurses an assignment that they cannot safely handle. Nurses who float must inform the supervisor of any lack of experience in caring for the types of patients on the nursing unit. They should request and receive an orientation to the unit. A basic orientation is needed, whereas a complete orientation of the functioning of the entire unit would take a period of time that would exceed what the nurse has to spend on orientation. Allowing nurses to choose which mealtime they would like is a nice gesture of thanks for the nurse, but it does not enable safe care. Having nursing assistive personnel may help the nurse complete basic tasks such as hygiene and turning, but it does not enable safe nursing care that the nurse and manager are ultimately responsible for.

188
Q

While recovering from a severe illness, a hospitalized patient wants to change a living will, which was signed 9 months ago. Which response by the nurse is most appropriate?

a. “Check with your admitting health care provider whether a copy is on your chart.”
b. “Let me check with someone here in the hospital who can assist you.”
c. “You are not allowed to ever change a living will after signing it.”
d. “Your living will can be changed only once each calendar year.”

A

ANS: B
As long as the patient is not declared legally incompetent or lacks the capacity to make decisions, living wills can be changed. It is the nurse’s responsibility to find an appropriate person in the facility to assist the patient. Checking with the health care provider about the presence of a living will on the chart has nothing to do with the patient’s desire to change the living will. The question states that the patient wants to change a living will. A living will can be changed whenever the patient decides to change it, as long as the patient is competent.

189
Q

A home health nurse notices that a patient’s preschool children are often playing on the sidewalk and in the street unsupervised and repeatedly takes them back to the home and talks with the patient, but the situation continues. Which immediate action by the nurse is mandated by law?

a. Contact the appropriate community child protection facility.
b. Tell the parents that the authorities will be contacted shortly.
c. Take pictures of the children to support the overt child abuse.
d. Discuss with both parents about the safety needs of their children.

A

ANS: A
The nurse has a duty to report this situation to protect the children. Any health care professional who does not report suspected child abuse or neglect may be liable for civil or criminal legal action. Talking with both parents is not mandated by law. There is no obligation to tell the parents that they will be reported to authorities. There is no obligation for the nurse to take pictures of the children.

190
Q

. A confused patient with a urinary catheter, nasogastric tube, and intravenous line keeps touching these needed items for care. The nurse has tried to explain to the patient that these lines should not be touched, but the patient continues. Which is the best action by the nurse at this time?

a. Apply restraints loosely on the patient’s dominant wrist.
b. Notify the health care provider that restraints are needed immediately.
c. Try other approaches to prevent the patient from touching these care items.
d. Allow the patient to pull out lines to prove that the patient needs to be restrained.

A

ANS: C
Restraints can be used when less restrictive interventions are not successful. The nurse must try other approaches than just telling. The situation states that the patient is touching the items, not trying to pull them out. At this time, the patient’s well-being is not at risk so restraints cannot be used at this time nor does the health care provider need to be notified. Allowing the patient to pull out any of these items to prove the patient needs to be restrained is not acceptable.

191
Q

A patient with sepsis as a result of long-term leukemia dies 25 hours after admission to the hospital. A full code was conducted without success. The patient had a urinary catheter, an intravenous line, an oxygen cannula, and a nasogastric tube. Which question is the priority for the nurse to ask the family before beginning postmortem care?

a. “Is an autopsy going to be done?”
b. “Which funeral home do you want to use?”
c. “Would you like to assist in bathing your loved one?”
d. “Do you want me to remove the lines and tubes before you see your loved one?”

A

ANS: A
An autopsy or postmortem examination may be requested by the patient or the patient’s family, as part of an institutional policy, or if required by law. Because the patient’s death occurred as a result of long-term illness and not under suspicious circumstances, whether to conduct a postmortem examination would be decided by the family, and consent would have to be obtained from the family. The nurse needs to know if the lines can be removed or not depending upon the family’s response to the question. Asking about bathing the deceased patient is a valid question but is not a priority, because the nurse needs to know the protocol to follow if an autopsy is to be done. Finding out which funeral home the deceased patient is to be transported to is valid but is not a priority, because other actions must be taken before the deceased patient is transported from the hospital. Asking about removing the lines may not be an option depending on the response of the family to an autopsy.

192
Q

Conjoined twins are in the neonatal department of the community hospital until transfer to the closest medical center. A photographer from the local newspaper gets off the elevator on the neonatal floor and wants to take pictures of the infants. Which initial action should the nurse take?

a. Escort the cameraman to the neonatal unit while a few pictures are taken quietly.
b. Tell the cameraman where the hospital’s public relations department is located.
c. Have the cameraman wait for permission from the health care provider.
d. Ask the cameraman how the pictures are to be used in the newspaper.

A

ANS: B
In some cases, information about a scientific discovery or a major medical breakthrough or an unusual situation is newsworthy. In this case, anyone seeking information needs to contact the hospital’s public relations department to ensure that invasion of privacy does not occur. It is not the nurse’s responsibility to decide independently the legality of disclosing information. The nurse does not have the right to allow the cameraman access to the neonatal unit. This would constitute invasion of privacy. The health care provider has no responsibility regarding this situation and cannot allow the cameraman on the unit. It is not the nurse’s responsibility to find out how the pictures are to be used. This is a task for the public relations department.

193
Q

A nursing student has been written up several times for being late with providing patient care and for omitting aspects of patient care and not knowing basic procedures that were taught in the skills course one term earlier. The nursing student says, “I don’t understand what the big deal is. As my instructor, you are there to protect me and make sure I don’t make mistakes.” What is the best response from the nursing instructor?

a. “You are practicing under the license of the hospital’s insurance.”
b. “You are expected to perform at the level of a professional nurse.”
c. “You are expected to perform at the level of a prudent nursing student.”
d. “You are practicing under the license of the nurse assigned to the patient.”

A

ANS: B
Although nursing students are not employees of the health care facility where they are having their clinical experience, they are expected to perform as professional nurses would in providing safe patient care. Different levels of standards do not apply. No standard is used for nursing students other than that they must meet the standards of a professional nurse. Student nurses do not practice under anybody’s license; nursing students are liable if their actions exceed their scope of practice or cause harm to patients.

194
Q

A nurse works full time on the oncology unit at the hospital and works part time on weekends giving immunizations at the local pharmacy. While giving an injection on a weekend, the nurse caused injury to the patient’s arm and is now being sued. How will the hospital’s malpractice insurance provide coverage for this nurse?

a. It will provide coverage as long as the nurse followed all procedures, protocols, and policies correctly.
b. The hospital’s malpractice insurance covers this nurse only during the time the nurse is working at the hospital.
c. As long as the nurse has never been sued before this incident, the hospital’s malpractice insurance will cover the nurse.
d. The hospital’s malpractice insurance will provide approximately 50% of the coverage the nurse will need.

A

ANS: B
Malpractice insurance provided by the employing institution covers nurses only while they are working within the scope of their employment. It is always wise to find out if malpractice insurance is provided by a secondary place of employment, in this case, the pharmacy, or the nurse should carry an individual malpractice policy to cover situations such as this. The hospital policy would not provide coverage even if the nurse followed all procedures and policies or had never been sued. It will not provide 50% of coverage.

195
Q

A female nursing student in the final term of nursing school is overheard by a nursing faculty member telling another student that she got to insert a nasogastric tube in the emergency department while working as a nursing assistant. Which advice is best for the nursing faculty member to give to the nursing student?

a. “Just be careful when you are doing new procedures and make sure you are following directions by the nurse.”
b. “Review your procedures before you go to work, so you will be prepared to do them if you have a chance.”
c. “The nurse should not have allowed you to insert the nasogastric tube because something bad could have happened.”
d. “You are not allowed to perform any procedures other than those in your job description even with the nurse’s permission.”

A

ANS: D
When nursing students work as nursing assistants or nurse’s aides when not attending classes, they should not perform tasks that do not appear in a job description for a nurse’s aide or assistant. The nursing student should always follow the directions of the nurse, unless doing so violates the institution’s guidelines or job description under which the nursing student was hired, such as inserting a nasogastric tube or giving an intramuscular medication. The nursing student should be able to safely complete the procedures delegated as a nursing assistant, and reviewing those not done recently is a good idea, but it has nothing to do with the situation. The focus of the discussion between the nursing faculty member and the nursing student should be on following the job description under which the nursing student is working.

196
Q

The nurse calculates the medication dose for an infant on the pediatric unit and determines that the dose is twice what it should be based upon the drug book’s information. The pediatrician is contacted and says to administer the medication as ordered. Which actions should the nurse take next? (Select all that apply.)

a. Notify the nursing supervisor.
b. Administer the medication as ordered.
c. Give the amount listed in the drug book.
d. Ask the mother to give the drug to her child.
e. Check the chain of command policy for such situations.

A

ANS: A, E
If the health care provider confirms an order and the nurse still believes that it is inappropriate, the nurse should inform the supervising nurse and follow the established chain of command. Nurses follow health care providers’ orders unless they believe the orders are in error or may harm patients. Therefore, the nurse needs to assess all orders. If an order seems to be erroneous or harmful, further clarification from the health care provider is necessary. The supervising nurse should be able to help resolve the questionable order, but only the health care provider who wrote the order or a health care provider covering for the one who wrote the order can change the order. Harm to the infant could occur if the medication is given as ordered. The nurse cannot change an order by giving the amount listed in the drug book. Asking the mother to give the drug is inappropriate.

197
Q

The nurse hears a health care provider say to the charge nurse that a certain nurse cannot care for patients because the nurse is stupid and won’t follow orders. The health care provider also writes in the patient’s medical records that the same nurse, by name, is not to care for any of the patients because of incompetence.
Which torts has the health care provider committed? (Select all that apply.)
a. Libel

b. Slander
c. Assault
d. Battery
e. Invasion of privacy

A

ANS: A, B
Slander occurred when the health care provider spoke falsely about the nurse, and libel occurred when the health care provider wrote false information in the chart. Both of these situations could cause problems for the nurse’s reputation. Invasion of privacy is the release of a patient’s medical information to an unauthorized person such as a member of the press, the patient’s employer, or the patient’s family. Assault is any action that places a person in reasonable fear of harmful, imminent, or unwelcome contact. No actual contact is required for an assault to occur. Battery is any intentional touching without consent.

198
Q

A patient has approximately 6 months to live and asks about a do not resuscitate (DNR) order. Which statements by the nurse give the patient correct information? (Select all that apply.)

a. “You will be resuscitated unless there is a DNR order in the chart.”
b. “If you want certain procedures or actions taken or not taken, and you might not be able to tell anyone at the time, you need to complete documents ahead of time that give your health care provider this information.”
c. “You will be resuscitated at any time to allow you the longest length of survival.”
d. “If you decide you want a DNR order, you will need to talk to your health care provider.”
e. “If you travel to another state, your living will should cover your wishes.”

A

ANS: A, B, D
Health care providers perform CPR on an appropriate patient unless a do not resuscitate (DNR) order has been placed in the patient’s chart. The statutes assume that all patients will be resuscitated unless a written DNR order is found in the chart. Legally competent adult patients can consent to a DNR order verbally or in writing after receiving appropriate information from the health care provider. A health care proxy or durable power of attorney for health care (DPAHC) is a legal document that designates a person or persons of one’s choosing to make health care decisions when the patient is no longer able to make decisions on his or her own behalf. This agent makes health care treatment decisions based on the patient’s wishes, like a DNR. Resuscitation is performed anytime (not just for the longest length of survival) unless a DNR is written in the chart. Differences among the states have been noted regarding advance directives, so the patient should check state laws to see if a state will honor an advance directive that was originated in another state.

199
Q

. A nurse is teaching the staff about professional negligence or malpractice. Which criteria to establish negligence will the nurse include in the teaching session? (Select all that apply.)

a. Injury did not occur.
b. That duty was breached.
c. Nurse carried out the duty.
d. Duty of care was owed to the patient.
e. Patient understands benefits and risks of a procedure.

A

ANS: B, D
Certain criteria are necessary to establish nursing malpractice: (1) the nurse (defendant) owed a duty of care to the patient (plaintiff), (2) the nurse did not carry out or breached that duty, (3) the patient was injured, and (4) the nurse’s failure to carry out the duty caused the injury. If an injury did not occur and the nurse carried out the duty, no malpractice occurred. When a patient understands benefits and risks of the procedure, that is informed consent, not malpractice.

200
Q

A nurse is discussing nursing actions that can lead to breaches of nursing practice. Match the example to the term it describes.

a. Nurse posts about patient’s loud and unruly family members.
b. Nurse immediately applies restraints to make patient stay in bed.
c. Nurse leaves bed in high position, causing patient to fall and break hip.
d. Nurse states that she will wrap a bandage over patient’s mouth if he won’t be quiet.
e. Nurse applies abdominal bandage after refusal.
f. Nurse gets angry at patient and nurse leaves the hospital.

  1. Assault
  2. Battery
  3. Abandonment
  4. False imprisonment
  5. Invasion of privacy
  6. Malpractice
A

ANS:

  1. D
  2. E
  3. F
  4. B
  5. A
  6. C
201
Q

Which types of nurses make the best communicators with patients?

a. Those who learn effective psychomotor skills
b. Those who develop critical thinking skills
c. Those who like different kinds of people
d. Those who maintain perceptual biases

A

ANS: B
Nurses who develop critical thinking skills make the best communicators. Just liking people does not make an effective communicator because it is important to apply critical thinking standards to ensure sound effective communication. Just learning psychomotor skills does not ensure that the nurse will use those techniques, and communication involves more than psychomotor skills. Critical thinking helps the nurse overcome perceptual biases or human tendencies that interfere with accurately perceiving and interpreting messages from others. Nurses who maintain perceptual biases do not make good communicators.

202
Q

A nurse believes that the nurse-patient relationship is a partnership and that both are equal participants. Which term should the nurse use to describe this belief?

a. Critical thinking
b. Authentic
c. Mutuality
d. Attend

A

ANS: C
Effective interpersonal communication requires a sense of mutuality, a belief that the nurse-patient relationship is a partnership and that both are equal participants. Critical thinking in nursing, based on established standards of nursing care and ethical standards, promotes effective communication and uses standards such as humility, self-confidence, independent attitude, and fairness. To be authentic (one’s self) and to respond appropriately to the other person are important for interpersonal relationships but do not mean mutuality. Attending is giving all of your attention to the patient.

203
Q

A nurse wants to present information about flu immunizations to the older adults in the community. Which type of communication should the nurse use?

a. Public
b. Small group
c. Interpersonal
d. Intrapersonal

A

ANS: A
Public communication is interaction with an audience. Nurses have opportunities to speak with groups of consumers about health-related topics, present scholarly work to colleagues at conferences, or lead classroom discussions with peers or students. When nurses work on committees or participate in patient care conferences, they use a small group communication process. Interpersonal communication is one-on-one interaction between a nurse and another person that often occurs face to face. Intrapersonal communication is a powerful form of communication that you use as a professional nurse. This level of communication is also called selftalk.

204
Q

A nurse is using therapeutic communication with a patient. Which technique will the nurse use to ensure effective communication?

a. Interpersonal communication to change negative self-talk to positive self-talk
b. Small group communication to present information to an audience
c. Electronic communication to assess a patient in another city
d. Intrapersonal communication to build strong teams

A

ANS: C
Electronic communication is the use of technology to create ongoing relationships with patients and their health care team. Intrapersonal communication is self-talk. Interpersonal communication is one-on-one interaction between a nurse and another person that often occurs face to face. Public communication is used to present information to an audience. Small group communication is interaction that occurs when a small number of persons meet. When nurses work on committees or participate in patient care conferences, they use a small group communication process.

205
Q

A nurse is standing beside the patient’s bed.
Nurse: How are you doing?
Patient: I don’t feel good.
Which element will the nurse identify as feedback?

a. Nurse
b. Patient
c. How are you doing?
d. I don’t feel good.

A

ANS: D
“I don’t feel good” is the feedback because the feedback is the message the receiver returns. The sender is the person who encodes and delivers the message, and the receiver is the person who receives and decodes the message. The nurse is the sender. The patient is the receiver. “How are you doing?” is the message.

206
Q

A nurse is sitting at the patient’s bedside taking a nursing history. Which zone of personal space is the nurse using?

a. Socio-consultative
b. Personal
c. Intimate
d. Public

A

ANS: B
Personal space is 18 inches to 4 feet and involves things such as sitting at a patient’s bedside, taking a patient’s nursing history, or teaching an individual patient. Intimate space is 0 to 18 inches and involves things such as performing a physical assessment, bathing, grooming, dressing, feeding, and toileting a patient. The socio-consultative zone is 9 to 12 feet and involves things such as giving directions to visitors in the hallway and giving verbal report to a group of nurses. The public zone is 12 feet and greater and involves things such as speaking at a community forum, testifying at a legislative hearing, or lecturing.

207
Q

A smiling patient angrily states, “I will not cough and deep breathe.” How will the nurse interpret this finding?

a. The patient’s denotative meaning is wrong.
b. The patient’s personal space was violated.
c. The patient’s affect is inappropriate.
d. The patient’s vocabulary is poor.

A

ANS: C
An inappropriate affect is a facial expression that does not match the content of a verbal message (e.g., smiling when describing a sad situation). The patient is smiling but is angry, which indicates an inappropriate affect. The patient’s personal space was not violated. The patient’s vocabulary is not poor. Individuals who use a common language share denotative meaning: baseball has the same meaning for everyone who speaks English, but code denotes cardiac arrest primarily to health care providers. The patient’s denotative meaning is correct for cough and deep breathe.

208
Q

The nurse asks a patient where the pain is, and the patient responds by pointing to the area of pain. Which form of communication did the patient use?

a. Verbal
b. Nonverbal
c. Intonation
d. Vocabulary

A

ANS: B
The patient gestured (pointed), which is a type of nonverbal communication. Gestures emphasize, punctuate, and clarify the spoken word. Pointing to an area of pain is sometimes more accurate than describing its location. Verbal is the spoken word or message. Intonation or tone of voice dramatically affects the meaning of a message. Vocabulary consists of words used for verbal communication.

209
Q

A patient has been admitted to the hospital numerous times. The nurse asks the patient to share a personal story about the care that has been received. Which interaction is the nurse using?

a. Nonjudgmental
b. Socializing
c. Narrative
d. SBAR

A

ANS: C
In a therapeutic relationship, nurses often encourage patients to share personal stories. Sharing stories is called narrative interaction. Socializing is an important initial component of interpersonal communication. It helps people get to know one another and relax. It is easy, superficial, and not deeply personal. Nonjudgmental acceptance of the patient is an important characteristic of the relationship. Acceptance conveys a willingness to hear a message or acknowledge feelings; it is not a technique that involves personal stories. SBAR is a popular communication tool that helps standardize communication among health care providers. SBAR stands for Situation, Background, Assessment, and Recommendation.

210
Q

Before meeting the patient, a nurse talks to other caregivers about the patient. Which phase of the helping relationship is the nurse in with this patient?

a. Preinteraction
b. Orientation
c. Working
d. Termination

A

ANS: A
The time before the nurse meets the patient is called the preinteraction phase. This phase can involve things such as reviewing available data, including the medical and nursing history, talking to other caregivers who have information about the patient, or anticipating health concerns or issues that can arise. The orientation phase occurs when the nurse and the patient meet and get to know one another. This phase can involve things such as setting the tone for the relationship by adopting a warm, empathetic, caring manner. The working phase occurs when the nurse and the patient work together to solve problems and accomplish goals. The termination phase occurs during the ending of the relationship. This phase can involve things such as reminding the patient that termination is near.

211
Q

During the initial home visit, a home health nurse lets the patient know that the visits are expected to end in about a month. Which phase of the helping relationship is the nurse in with this patient?

a. Preinteraction
b. Orientation
c. Working
d. Termination

A

ANS: B
Letting the patient know when to expect the relationship to be terminated occurs in the orientation phase. Preinteraction occurs before the nurse meets the patient. Working occurs when the nurse and the patient work together to solve problems and accomplish goals. Termination occurs during the ending of the relationship.

212
Q

A nurse and a patient work on strategies to reduce weight. Which phase of the helping relationship is the nurse in with this patient?

a. Preinteraction
b. Orientation
c. Working
d. Termination

A

ANS: C
The working phase occurs when the nurse and the patient work together to solve problems and accomplish goals. Preinteraction occurs before the nurse meets the patient. Orientation occurs when the nurse and the patient meet and get to know each other. Termination occurs during the ending of the relationship.

213
Q

A nurse uses SBAR when providing a hands-off report to the oncoming shift. What is the rationale for the nurse’s action?

a. To promote autonomy
b. To use common courtesy
c. To establish trustworthiness
d. To standardize communication

A

ANS: D
SBAR is a popular communication tool that helps standardize communication among health care providers. Common courtesy is part of professional communication but is not the purpose of SBAR. Being trustworthy means helping others without hesitation. Autonomy is being self-directed and independent in accomplishing goals and advocating for others.

214
Q

A patient was admitted 2 days ago with pneumonia and a history of angina. The patient is now having chest pain with a pulse rate of 108. Which piece of data will the nurse use for “B” when using SBAR?

a. Having chest pain
b. Pulse rate of 108
c. History of angina
d. Oxygen is needed

A

ANS: C
The B in SBAR stands for background information. The background information in this situation is the history of angina. Having chest pain is the Situation (S). Pulse rate of 108 is the Assessment (A). Oxygen is needed is the Recommendation (R).

215
Q

A patient just received a diagnosis of cancer. Which statement by the nurse demonstrates empathy?

a. “Tomorrow will be better.”
b. “This must be hard news to hear.”
c. “What’s your biggest fear about this diagnosis?”
d. “I believe you can overcome this because I’ve seen how strong you are.”

A

ANS: B
“This must be hard” is an example of empathy. Empathy is the ability to understand and accept another person’s reality, accurately perceive feelings, and communicate this understanding to the other. An example of false reassurance is “Tomorrow will be better.” “I believe you can overcome this” is an example of sharing hope. “What is your biggest fear?” is an open-ended question that allows patients to take the conversational lead and introduces pertinent information about a topic.

216
Q

A nurse is taking a history on a patient who cannot speak English. Which action will the nurse take?

a. Obtain an interpreter.
b. Refer to a speech therapist.
c. Let a close family member talk.
d. Find a mental health nurse specialist.

A

ANS: A
Interpreters are often necessary for patients who speak a foreign language. Using a family member can lead to legal issues, speech therapists help patients with aphasia, and mental health nurse specialists help angry or highly anxious patients to communicate more effectively.

217
Q

A nurse is using SOLER to facilitate active listening. Which technique should the nurse use for R?

a. Relax
b. Respect
c. Reminisce
d. Reassure

A

ANS: A
In SOLER, the R stands for relax. It is important to communicate a sense of being relaxed and comfortable with the patient. Active listening enhances trust because the nurse communicates acceptance and respect for the patient, but it is not the R in SOLER. Reminisce is a therapeutic communication technique, especially when used with the elderly. Reassuring can be therapeutic if the nurse reassures patients that there are many kinds of hope and that meaning and personal growth can come from illness experiences. However, false reassurance can block communication.

218
Q

. An older-adult patient is wearing a hearing aid. Which technique should the nurse use to facilitate communication?

a. Chew gum.
b. Turn off the television.
c. Speak clearly and loudly.
d. Use at least 14-point print.

A

ANS: B
Turning off the television will facilitate communication. Patients who are hearing impaired benefit when the following techniques are used: check for hearing aids and glasses, reduce environmental noise, get the patient’s attention before speaking, do not chew gum, and speak at normal volume—do not shout. Using at least 14point print is for sight/visually impaired, not hearing impaired.

219
Q

. When making rounds, the nurse finds a patient who is not able to sleep because of surgery in the morning. Which therapeutic response is most appropriate?

a. “You will be okay. Your surgeon will talk to you in the morning.”
b. “Why can’t you sleep? You have the best surgeon in the hospital.”
c. “Don’t worry. The surgeon ordered a sleeping pill to help you sleep.”
d. “It must be difficult not to know what the surgeon will find. What can I do to help?”

A

ANS: D
“It must be difficult not to know what the surgeon will find. What can I do to help?” is using therapeutic communication techniques of empathy and asking relevant questions. False reassurances (“You will be okay” and “Don’t worry”) tend to block communication. Patients frequently interpret “why” questions as accusations or think the nurse knows the reason and is simply testing them.

220
Q

Which situation will cause the nurse to intervene and follow up on the nursing assistive personnel’s (NAP) behavior?

a. The nursing assistive personnel is calling the older-adult patient “honey.”
b. The nursing assistive personnel is facing the older-adult patient when talking.
c. The nursing assistive personnel cleans the older-adult patient’s glasses gently.
d. The nursing assistive personnel allows time for the older-adult patient to respond.

A

ANS: A
The nurse needs to intervene to correct the use of “honey.” Avoid terms of endearment such as “honey,” “dear,” “grandma,” or “sweetheart.” Communicate with older adults on an adult level, and avoid patronizing or speaking in a condescending manner. Facing an older-adult patient, making sure the older adult has clean glasses, and allowing time to respond facilitate communication with older-adult patients and should be encouraged, not stopped.

221
Q

A confused older-adult patient is wearing thick glasses and a hearing aid. Which intervention is the priority to facilitate communication?

a. Focus on tasks to be completed.
b. Allow time for the patient to respond.
c. Limit conversations with the patient.
d. Use gestures and other nonverbal cues.

A

ANS: B
Allowing time for patients to respond will facilitate communication, especially for a confused, older patient. Focusing on tasks to be completed and limiting conversations do not facilitate communication; in fact, they block communication. Using gestures and other nonverbal cues is not effective for visually impaired (thick glasses) patients or for patients who are confused.

222
Q

. The staff is having a hard time getting an older-adult patient to communicate. Which technique should the nurse suggest the staff use?

a. Try changing topics often.
b. Allow the patient to reminisce.
c. Ask the patient for explanations.
d. Involve only the patient in conversations.

A

ANS: B
Encouraging older adults to share life stories and reminisce about the past has a therapeutic effect and increases their sense of well-being. Avoid sudden shifts from subject to subject. It is helpful to include the patient’s family and friends and to become familiar with the patient’s favorite topics for conversation. Asking for explanations is a nontherapeutic technique.

223
Q

A nurse is implementing nursing care measures for patients’ special communication needs. Which patient will need the most nursing care measures?

a. The patient who is oriented, pain free, and blind
b. The patient who is alert, hungry, and has strong self-esteem
c. The patient who is cooperative, depressed, and hard of hearing
d. The patient who is dyspneic, anxious, and has a tracheostomy

A

ANS: D
Facial trauma, laryngeal cancer, or endotracheal intubation often prevents movement of air past vocal cords or mobility of the tongue, resulting in inability to articulate words. An extremely breathless person needs to use oxygen to breathe rather than speak. Persons with high anxiety are sometimes unable to perceive environmental stimuli or hear explanations. People who are alert, have strong self-esteem, and are cooperative and pain free do not cause communication concerns. Although hunger, blindness, and difficulty hearing can cause communication concerns, dyspnea, tracheostomy, and anxiety all contribute to communication concerns.

224
Q

A patient is aphasic, and the nurse notices that the patient’s hands shake intermittently. Which nursing action is most appropriate to facilitate communication?

a. Use a picture board.
b. Use pen and paper.
c. Use an interpreter.
d. Use a hearing aid.

A

ANS: A
Using a pen and paper can be frustrating for a nonverbal (aphasic) patient whose handwriting is shaky; the nurse can revise the care plan to include use of a picture board instead. An interpreter is used for a patient who speaks a foreign language. A hearing aid is used for the hard of hearing, not for an aphasic patient.

225
Q

Which behavior indicates the nurse is using a process recording correctly to enhance communication with patients?

a. Shows sympathy appropriately
b. Uses automatic responses fluently
c. Demonstrates passive remarks accurately
d. Self-examines personal communication skills

A

ANS: D
Analysis of a process recording enables a nurse to evaluate the following: examine whether nursing responses blocked or facilitated the patient’s efforts to communicate. Sympathy is concern, sorrow, or pity felt for the patient and is nontherapeutic. Clichés and stereotyped remarks are automatic responses that communicate the nurse is not taking concerns seriously or responding thoughtfully. Passive responses serve to avoid conflict or to sidestep issues.

226
Q

. A patient says, “You are the worst nurse I have ever had.” Which response by the nurse is most assertive?

a. “I think you’ve had a hard day.”
b. “I feel uncomfortable hearing that statement.”
c. “I don’t think you should say things like that. It is not right.”
d. “I have been checking on you regularly. How can you say that?”

A

ANS: B
Assertive responses contain “I” messages such as “I want,” “I need,” “I think,” or “I feel.” While all of these start with “I,” the only one that is the most assertive is “I feel uncomfortable hearing that statement.” An assertive nurse communicates self-assurance; communicates feelings; takes responsibility for choices; and is respectful of others’ feelings, ideas, and choices. “I think you’ve had a hard day” is not addressing the problem. Arguing (“How can you say that?”) is not assertive or therapeutic. Showing disapproval (using words like right) is not assertive or therapeutic.

227
Q

. Which behaviors indicate the nurse is using critical thinking standards when communicating with patients?
(Select all that apply.)

a. Instills faith
b. Uses humility
c. Portrays self-confidence
d. Exhibits supportiveness
e. Demonstrates independent attitude

A

ANS: B, C, E
A self-confident attitude is important because the nurse who conveys confidence and comfort while communicating more readily establishes an interpersonal helping-trusting relationship. In addition, an independent attitude encourages the nurse to communicate with colleagues and share ideas about nursing interventions. An attitude of humility is necessary to recognize and communicate the need for more information before making a decision. Faith and supportiveness are attributes of caring, not critical thinking standards.

228
Q

A nurse is implementing nursing care measures for patients with challenging communication issues. Which types of patients will need these nursing care measures? (Select all that apply.)

a. A child who is developmentally delayed
b. An older-adult patient who is demanding
c. A female patient who is outgoing and flirty
d. A male patient who is cooperative with treatments
e. An older-adult patient who can clearly see small print
f. A teenager frightened by the prospect of impending surgery

A

ANS: A, B, C, F
Challenging communication situations include patients who are flirtatious, demanding, frightened, or developmentally delayed. A child who has received little environmental stimulation possibly is behind in language development, thus making communication more challenging. Patients who are cooperative and have good eyesight (see small print) do not cause challenging communication situations.

229
Q

A nurse is using AIDET to communicate with patients and families. Match the letters of the acronym to the behavior a nurse will use.

a. Nurse describes procedures and tests.
b. Nurse lets the patient know how long the procedure will last.
c. Nurse recognizes the person with a positive attitude.
d. Nurse thanks the patient.
e. Nurse tells the patient “I am an RN and will be managing your care.”

  1. A
  2. I
  3. D
  4. E
  5. T
A

ANS:

  1. C
  2. E
  3. B
  4. A
  5. D
230
Q

A nurse preceptor is working with a student nurse. Which behavior by the student nurse will require the nurse preceptor to intervene?

a. The student nurse reads the patient’s plan of care.
b. The student nurse reviews the patient’s medical record.
c. The student nurse shares patient information with a friend.
d. The student nurse documents medication administered to the patient.

A

ANS: C
When you are a student in a clinical setting, confidentiality and compliance with the Health Insurance Portability and Accountability Act (HIPAA) are part of professional practice. When a student nurse shares patient information with a friend, confidentiality and HIPAA standards have been violated, causing the preceptor to intervene. You can review your patients’ medical records only to seek information needed to provide safe and effective patient care. For example, when you are assigned to care for a patient, you need to review the patient’s medical record and plan of care. You do not share this information with classmates and you do not access the medical records of other patients on the unit.

231
Q

A nurse exchanges information with the oncoming nurse about a patient’s care. Which action did the nurse
complete?

a. A verbal report
b. An electronic record entry
c. A referral
d. An acuity rating

A

ANS: A
Whether the transfer of patient information occurs through verbal reports, electronic or written documents, you need to follow some basic principles. Reports are exchanges of information among caregivers. A patient’s electronic medical record or chart is a confidential, permanent legal documentation of information relevant to a patient’s health care. Nurses document referrals (arrangements for the services of another care provider). Nurses use acuity ratings to determine the hours of care and number of staff required for a given group of patients every shift or every 24 hours.

232
Q

A nurse is auditing and monitoring patients’ health records. Which action is the nurse taking?

a. Determining the degree to which standards of care are met by reviewing patients’ health records
b. Realizing that care not documented in patients’ health records still qualifies as care provided
c. Basing reimbursement upon the diagnosis-related groups documented in patients’ records
d. Comparing data in patients’ records to determine whether a new treatment had better outcomes than the standard treatment

A

ANS: A
The auditing and monitoring of patients’ health records involve nurses periodically auditing records to determine the degree to which standards of care are met and identifying areas needing improvement and staff development. The mistakes in documentation that commonly result in malpractice include failing to record nursing actions; this is the aspect of legal documentation. The financial billing or reimbursement purpose involves diagnosis-related groups (DRGs) as the basis for establishing reimbursement for patient care. For research purposes, the researcher compares the patient’s recorded findings to determine whether the new method was more effective than the standard protocol. Data analysis contributes to evidence-based nursing practice and quality health care.

233
Q

. After providing care, a nurse charts in the patient’s record. Which entry will the nurse document?

a. Appears restless when sitting in the chair
b. Drank adequate amounts of water
c. Apparently is asleep with eyes closed
d. Skin pale and cool

A

ANS: D
A factual record contains descriptive, objective information about what a nurse observes, hears, palpates, and smells. Objective data is obtained through direct observation and measurement (skin pale and cool). For example, “B/P 80/50, patient diaphoretic, heart rate 102 and regular.” Avoid vague terms such as appears, seems, or apparently because these words suggest that you are stating an opinion, do not accurately communicate facts, and do not inform another caregiver of details regarding behaviors exhibited by the patient. Use of exact measurements establishes accuracy. For example, a description such as “Intake, 360 mL of water” is more accurate than “Patient drank an adequate amount of fluid.”

234
Q

A nurse has provided care to a patient. Which entry should the nurse document in the patient’s record?

a. Status unchanged, doing well
b. Patient seems to be in pain and states, “I feel uncomfortable.”
c. Left knee incision 1 inch in length without redness, drainage, or edema
d. Patient is hard to care for and refuses all treatments and medications. Family is present.

A

ANS: C
Use of exact measurements establishes accuracy. Charting that an abdominal wound is “approximated, 5 cm in length without redness, drainage, or edema,” is more descriptive than “large abdominal incision healing well.” Include objective data to support subjective data, so your charting is as descriptive as possible. Avoid using generalized, empty phrases such as “status unchanged” or “had good day.” It is essential to avoid the use of unnecessary words and irrelevant details or personal opinions. “Patient is hard to care for” is a personal opinion and should be avoided. It is also a critical comment that can be used as evidence for nonprofessional behavior or poor quality of care. Just chart, “Refuses all treatments and medications.”

235
Q

A preceptor is working with a new nurse on documentation. Which situation will cause the preceptor to follow up?

a. The new nurse documents only for self.
b. The new nurse charts consecutively on every other line.
c. The new nurse ends each entry with signature and title.
d. The new nurse keeps the password secure.

A

ANS: B
Chart consecutively, line by line (not every other line); every other line is incorrect and must be corrected by the preceptor. If space is left, draw a line horizontally through it, and place your signature and credentials at the end. Every other line should not be left blank. All the other behaviors are correct and need no follow-up. Documenting only for yourself is an appropriate behavior. End each entry with signature and title/credentials. For computer documentation, keep your password to yourself.

236
Q

A nurse is charting on a patient’s record. Which action will the nurse take that is accurate legally?

a. Charts legibly
b. States the patient is belligerent
c. Writes entry for another nurse
d. Uses correction fluid to correct error

A

ANS: A
Record all entries legibly. Do not write personal opinions (belligerent). Enter only objective and factual observations of patient’s behavior; quote all patient comments. Do not erase, apply correction fluid, or scratch out errors made while recording. Chart only for yourself.

237
Q

A nurse wants to find all the pertinent patient information in one record, regardless of the number of times the patient entered the health care system. Which record should the nurse find?

a. Electronic medical record
b. Electronic health record
c. Electronic charting record
d. Electronic problem record

A

ANS: B
The term electronic health record/EHR is increasingly used to refer to a longitudinal (lifetime) record of all health care encounters for an individual patient by linking all patient data from previous health encounters. An electronic medical record (EMR) is the legal record that describes a single encounter or visit created in hospitals and outpatient health care settings that is the source of data for the EHR. There are no such terms as electronic charting record or electronic problem record that record the lifetime information of a patient.

238
Q

A nurse has taught the patient how to use crutches. The patient went up and down the stairs using crutches with no difficulties. Which information will the nurse use for the “I” in PIE charting?

a. Patient went up and down stairs
b. Demonstrated use of crutches
c. Used crutches with no difficulties
d. Deficient knowledge related to never using crutches

A

ANS: B
A second progress note method is the PIE format. The narrative note includes P—Nursing diagnosis, I—Intervention, and E—Evaluation. The intervention is “Demonstrated use of crutches.” “Patient went up and down stairs” and “Used crutches with no difficulties” are examples of E. “Deficient knowledge regarding crutches” is P.

239
Q

A nurse wants to find the daily weights of a patient. Which form will the nurse use?

a. Database
b. Progress notes
c. Patient care summary
d. Graphic record and flow sheet

A

ANS: D
Within a computerized documentation system, flow sheets and graphic records allow you to quickly and easily enter assessment data about a patient, such as vital signs, admission and or daily weights, and percentage of meals eaten. In the problem-oriented medical record, the database section contains all available assessment information pertaining to the patient (e.g., history and physical examination, nursing admission history and ongoing assessment, physical therapy assessment, laboratory reports, and radiologic test results). Many computerized documentation systems have the ability to generate a patient care summary document that you review and sometimes print for each patient at the beginning and/or end of each shift; it includes information such as basic demographic data, health care provider’s name, primary medical diagnosis, and current orders. Health care team members monitor and record the progress made toward resolving a patient’s problems in progress notes.

240
Q

A nurse is a member of an interdisciplinary team that uses critical pathways. According to the critical pathway, on day 2 of the hospital stay, the patient should be sitting in the chair. It is day 3, and the patient cannot sit in the chair. What should the nurse do?

a. Add this data to the problem list.
b. Focus chart using the DAR format.
c. Document the variance in the patient’s record.
d. Report a positive variance in the next interdisciplinary team meeting.

A

ANS: C
A variance occurs when the activities on the critical pathway are not completed as predicted or the patient does not meet expected outcomes. An example of a negative variance is when a patient develops pulmonary complications after surgery, requiring oxygen therapy and monitoring with pulse oximetry. A positive variance occurs when a patient progresses more rapidly than expected (e.g., use of a Foley catheter may be discontinued a day early). When a nurse is using the problem-oriented medical record, after analyzing data, health care team members identify problems and make a single problem list. A third format used for notes within a POMR is focus charting. It involves the use of DAR notes, which include D—Data (both subjective and objective), A— Action or nursing intervention, and R—Response of the patient (i.e., evaluation of effectiveness).

241
Q

. A nurse needs to begin discharge planning for a patient admitted with pneumonia and a congested cough. When is the best time the nurse should start discharge planning for this patient?

a. Upon admission
b. Right before discharge
c. After the congestion is treated
d. When the primary care provider writes the order

A

ANS: A
Ideally, discharge planning begins at admission. Right before discharge is too late for discharge planning. After the congestion is treated is also too late for discharge planning. Usually the primary care provider writes the order too close to discharge, and nurses do not need an order to begin the teaching that will be needed for discharge. By identifying discharge needs early, nursing and other health care professionals begin planning for discharge to the appropriate level of care, which sometimes includes support services such as home care and equipment needs.

242
Q

A patient is being discharged home. Which information should the nurse include?

a. Acuity level
b. Community resources
c. Standardized care plan
d. Signature for verbal order

A

ANS: B
Discharge documentation includes medications, diet, community resources, follow-up care, and who to contact in case of an emergency or for questions. A patient’s acuity level, usually determined by a computer program, is based on the types and numbers of nursing interventions (e.g., intravenous [IV] therapy, wound care, or ambulation assistance) required over a 24-hour period. Many computerized documentation systems include standardized care plans or clinical practice guidelines (CPGs) to facilitate the creation and documentation of a nursing and or interprofessional plan of care. Each CPG facilitates safe and consistent care for an identified problem by describing or listing institutional standards and evidence-based guidelines that are easily accessed and included in a patient’s electronic health record. Verbal orders occur when a health care provider gives therapeutic orders to a registered nurse while they are standing in proximity to one another.

243
Q

A nurse developed the following discharge summary sheet. Which critical information should the nurse add?

TOPIC DISCHARGE SUMMARY

Medication

Diet

Activity level

Follow-up care

Wound care

Phone numbers

When to call the doctor

Time of discharge

a. Clinical decision support system
b. Admission nursing history
c. Mode of transportation
d. SOAP notes

A

ANS: C
List actual time of discharge, mode of transportation, and who accompanied the patient for discharge summary information. Clinical decision support systems (CDSSs) are computerized programs used within the health care setting, to aid and support clinical decision making. The knowledge base within a CDSS contains rules and logic statements that link information required for clinical decisions in order to generate tailored recommendations for individual patients that are presented to nurses as alerts, warnings, or other information for consideration. A nurse completes a nursing history form when a patient is admitted to a nursing unit, not when the patient is discharged. SOAP notes are not given to patients who are being discharged. SOAP notes are a type of documentation style.

244
Q

. A home health nurse is preparing for an initial home visit. Which information should be included in the patient’s home care medical record?

a. Nursing process form
b. Step-by-step skills manual
c. A list of possible procedures
d. Reports to third-party payers

A

ANS: D
Information in the home care medical record includes patient assessment, referral and intake forms, interprofessional plan of care, a list of medications, and reports to third-party payers. An interprofessional plan of care is used rather than a nursing process form. A step-by-step skills manual and a list of possible procedures are not included in the record.

245
Q

A nurse in a long-term care setting that is funded by Medicare and Medicaid is completing standardized protocols for assessment and care planning for reimbursement. Which task is the nurse completing?

a. A minimum data set
b. An admission assessment and acuity level
c. A focused assessment/specific body system
d. An intake assessment form and auditing phase

A

ANS: A
The Resident Assessment Instrument (RAI), which includes the Minimum Data Set (MDS) and the Care Area Assessment (CAA), is the data set that is federally mandated for use in long-term care facilities by CMS. MDS assessment forms are completed upon admission, and then periodically, within specific guidelines and time frames for all residents in certified nursing homes. The MDS also determines the reimbursement level under the prospective payment system. A focused assessment is limited to a specific body system. An admission assessment and acuity level is performed in the hospital. An intake form is for home health. There is no such thing as an auditing phase in an assessment intake.

246
Q

A nurse is charting. Which information is critical for the nurse to document?

a. The patient had a good day with no complaints.
b. The family is demanding and argumentative.
c. The patient received a pain medication, Lortab.
d. The family is poor and had to go on welfare.

A

ANS: C
Nursing interventions and treatments (e.g., medication administration) must be documented. Avoid using generalized, empty phrases such as “status unchanged” or “had good day.” Do not document retaliatory or critical comments about a patient, like demanding and argumentative. Family is poor is not critical information to chart.

247
Q

A nurse is completing an OASIS data set on a patient. The nurse works in which area?

a. Home health
b. Intensive care unit
c. Skilled nursing facility
d. Long-term care facility

A

ANS: A
Nurses use two different data sets to document the clinical assessments and care provided in the home care setting, the Outcome and Assessment Information Set (OASIS) and the Omaha System. The intensive care unit does not use the OASIS data set. The long-term health care setting includes skilled nursing facilities (SNFs) in which patients receive 24-hour day care.

248
Q

A nurse is preparing to document a patient who has chest pain. Which information is critical for the nurse to include?

a. The family is a “pain.”
b. Pupils equal and reactive to light
c. Had poor results from the pain medication
d. Sharp pain of 8 on a scale of 1 to 10

A

ANS: D
You need to ensure the information within a recorded entry or a report is complete, containing appropriate and essential information (pain of 8). Document subjective and objective assessment. While pupils equal and reactive to light is data, it does not relate to the chest pain; this information would be critical for a head injury. Derogatory or inappropriate comments about the patient or family (“pain”) is not appropriate. This kind of language can be used as evidence for nonprofessional behavior or poor quality of care. Avoid using generalized, empty phrases like “poor results.” Use complete, concise descriptions.

249
Q

A nurse is providing care to a group of patients. Which situation will require the nurse to obtain a telephone order?

a. As the nurse and health care provider leave a patient’s room, the primary care provider gives the nurse an order.
b. At 0100, a patient’s blood pressure drops from 120/80 to 90/50, and the incision dressing is saturated with blood.
c. At 0800, the nurse and health care provider make rounds, and the primary care provider tells the nurse a diet order.
d. A nurse reads an order correctly as written by the health care provider in the patient’s medical record.

A

ANS: B
Telephone orders and verbal orders (VO) usually occur at night or during emergencies (blood pressure dropping); they should be used only when absolutely necessary and not for the sake of convenience. Because the time is 1 AM (0100 military time) and the health care provider is not present, the nurse will need to call the health care provider for a telephone order. A VO involves the health care provider giving orders to a nurse while they are standing in proximity to one another. Just reading an order that is correctly written in the chart does not require a telephone order.

250
Q

A nurse obtained a telephone order from a primary care provider for a patient in pain. Which chart entry should the nurse document?

a. 12/16/20XX 0915 Morphine, 2 mg, IV every 4 hours for incisional pain. VO Dr. Day/J. Winds, RN, read back.
b. 12/16/20XX 0915 Morphine, 2 mg, IV every 4 hours for incisional pain. TO J. Winds, RN, read back.
c. 12/16/20XX 0915 Morphine, 2 mg, IV every 4 hours for incisional pain. TO Dr. Day/J. Winds, RN, read back.
d. 12/16/20XX 0915 Morphine, 2 mg, IV every 4 hours for incisional pain. TO J. Winds, RN.

A

ANS: C
The nurse receiving a TO or VO enters the complete order into the computer using the computerized provider order entry (CPOE) software or writes it out on a physician’s order sheet for entry in the computer as soon as possible. After you have taken the order, read the order back, using the “read back” process, and document that you did this to provide evidence that the information received (such as call back instructions and/or therapeutic orders) was verified with the provider. An example follows: “10/16/2015 (08:15), Change IV fluid to Lactated Ringers with Potassium 20 mEq/L to run at 125 mL/hr. TO: Dr. Knight/J. Woods, RN, read back.” VO stands for verbal order, not telephone order. The health care provider’s name and read back must be included in the chart entry.

251
Q

A nurse is teaching the staff about informatics. Which information from the staff indicates the nurse needs to follow up?

a. To be proficient in informatics, a nurse should be able to discover, retrieve, and use information in practice.
b. A nurse needs to know how to find, evaluate, and use information effectively.
c. If a nurse has computer competency, the nurse is competent in informatics.
d. Nursing informatics is a recognized specialty area of nursing practice.

A

ANS: C
When the staff make an incorrect statement, then the nurse needs to follow up. Competence in informatics is not the same as computer competency. All the rest are correct information so the nurse does not need to follow up. To become competent in informatics, you need to be able to use evolving methods of discovering, retrieving, and using information in practice. This means that you learn to recognize when information is needed and have the skills to find, evaluate, and use that information effectively. Nursing informatics is a specialty that integrates the use of information and computer technology to support all aspects of nursing practice, including direct delivery of care, administration, education, and research.

252
Q

A hospital is using a computer system that allows all health care providers to use a protocol system to document the care they provide. Which type of system/design will the nurse be using?
a. Clinical decision support system

b. Nursing process design
c. Critical pathway design
d. Computerized provider order entry system

A

ANS: C
One design model for Nursing Clinical Information Systems (NCIS) is the protocol or critical pathway design. This design facilitates interdisciplinary management of information because all health care providers use evidence-based protocols or critical pathways to document the care they provide. The knowledge base within a CDSS contains rules and logic statements that link information required for clinical decisions in order to generate tailored recommendations for individual patients, which are presented to nurses as alerts, warnings, or other information for consideration. The nursing process design is the most traditional design for an NCIS. This design organizes documentation within well-established formats such as admission and postoperative assessments, problem lists, care plans, discharge planning instructions, and intervention lists or notes. Computerized provider order entry (CPOE) systems allow health care providers to directly enter orders for patient care into the hospital’s information system.

253
Q

A nurse wants to reduce data entry errors on the computer system. Which action should the nurse take?

a. Use the same password all the time.
b. Share password with only one other staff member.
c. Print out and review computer nursing notes at home.
d. Chart on the computer immediately after care is provided.

A

ANS: D
To increase accuracy and decrease unnecessary duplication, many health care agencies keep records or computers near a patient’s bedside to facilitate immediate documentation of information as it is collected. A good system requires frequent, random changes in personal passwords to prevent unauthorized persons from tampering with records. When using a health care agency computer system, it is essential that you do not share your computer password with anyone under any circumstances. You destroy all papers containing personal information immediately after you use them. Taking nursing notes home is a violation of the Health Insurance Portability and Accountability Act (HIPAA) and confidentiality.

254
Q

Which entry will require follow-up by the nurse manager?

0800 Patient states, “Fell out of bed.” Patient found lying by bed on the floor. Legs equal in length bilaterally with no distortion, pedal pulses strong, leg strength equal and strong, no bruising or bleeding. Neuro checks within normal limits. States, “Did not pass out.” Assisted back to bed. Call bell within reach. Bed monitor on.
——————-Jane More, RN
0810 Notified primary care provider of patient’s status. New orders received. ——————-Jane More, RN
0815 Portable x-ray of L hip taken in room. States, “I feel fine.” ——————-Jane More, RN 0830 Incident report completed and placed on chart.
——————-Jane More, RN

a. 0800
b. 0810
c. 0815
d. 0830

A

ANS: D
Do not include any reference to an incident in the medical record; therefore, the nurse manager must follow up. A notation about an incident report in a patient’s medical record makes it easier for a lawyer to argue that the reference makes the incident report part of the medical record and therefore subject to attorney review. When an incident occurs, document an objective description of what happened, what you observed, and the follow-up actions taken, including notification of the patient’s health care provider in the patient’s medical record. Remember to evaluate and document the patient’s response to the incident.

255
Q

A patient has a diagnosis of pneumonia. Which entry should the nurse chart to help with financial reimbursement?

a. Used incentive spirometer to encourage coughing and deep breathing. Lung congested upon auscultation in lower lobes bilaterally. Pulse oximetry 86%. Oxygen per nasal cannula applied at 2 L/min per standing order.
b. Cooperative, patient coughed and deep breathed using a pillow as a splint. Stated, “felt better.” Finally, patient had no complaints.
c. Breathing without difficulty. Sitting up in bed watching TV. Had a good day.
d. Status unchanged. Remains stable with no abnormal findings. Checked every 2 hours.

A

ANS: A
Accurately documenting services provided, including the supplies and equipment used in a patient’s care, clarifies the type of treatment a patient received. This documentation also supports accurate and timely reimbursement to a health care agency and/or patient. None of the other options had equipment or supplies listed. Avoid using generalized, empty phrases such as “status unchanged” or “had good day.” Do not enter personal opinions—stating that the patient is cooperative is a personal opinion and should be avoided. “Finally, patient had no complaints” is a critical comment about the patient and if charted can be used as evidence of nonprofessional behavior or poor quality of care.

256
Q

A nurse is teaching the staff about health care reimbursement. Which information should the nurse include in the teaching session?

a. Home health, long-term care, and hospital nurses’ documentation can affect reimbursement for health care.
b. A clinical information system must be installed by 2014 to obtain health care reimbursement.
c. A “near miss” helps determine reimbursement issues for health care.
d. HIPAA is the basis for establishing reimbursement for health care.

A

ANS: A
Nurses’ documentation practices in home health, long-term care, and hospitals can determine reimbursement for health care. A “near miss” is an incident where no property was damaged and no patient or personnel were injured, but given a slight shift in time or position, damage or injury could have easily occurred. A clinical information system (CIS) does not have to be installed by 2014 to obtain reimbursement. CIS programs include monitoring systems; order entry systems; and laboratory, radiology, and pharmacy systems. Diagnosis-related groups (DRGs) are the basis for establishing reimbursement for patient care, not HIPAA. Legislation to protect patient privacy regarding health information is the Health Insurance Portability and Accountability Act (HIPAA).

257
Q

A nurse is discussing the advantages of a nursing clinical information system. Which advantage should the nurse describe?

a. Varied clinical databases
b. Reduced errors of omission
c. Increased hospital costs
d. More time to read charts

A

ANS: B
Advantages associated with the nursing information system include reduced errors of omission; better access to information (not more time to read charts); enhanced quality of documentation; reduced, not increased, hospital costs; increased nurse job satisfaction; compliance with requirements of accrediting agencies (e.g., TJC); and development of a common, not varied, clinical database.

258
Q

Which behaviors indicate the student nurse has a good understanding of confidentiality and the Health Insurance Portability and Accountability Act (HIPAA)? (Select all that apply.)

a. Writes the patient’s room number and date of birth on a paper for school
b. Prints/copies material from the patient’s health record for a graded care plan
c. Reviews assigned patient’s record and another unassigned patient’s record
d. Gives a change-of-shift report to the oncoming nurse about the patient
e. Reads the progress notes of assigned patient’s record
f. Discusses patient care with the hospital volunteer

A

ANS: D, E
When you are a student in a clinical setting, confidentiality and compliance with HIPAA are part of professional practice. Reading the progress notes of an assigned patient’s record and giving a change-of-shift report to the oncoming nurse about the patient are behaviors that follow HIPAA and confidentiality guidelines. Do not share information with other patients or health care team members who are not caring for a patient. Not only is it unethical to view medical records of other patients, but breaches of confidentiality lead to disciplinary action by employers and dismissal from work or nursing school. To protect patient confidentiality, ensure that written materials used in your student clinical practice do not include patient identifiers (e.g., room number, date of birth, demographic information), and never print material from an electronic health record for personal use.

259
Q

A nurse is describing the purposes of a health care record to a group of nursing students. Which purposes will the nurse include in the teaching session? (Select all that apply.)

a. Communication
b. Legal documentation
c. Reimbursement
d. Nursing process
e. Research
f. Education

A

ANS: A, B, C, E, F
A patient’s record is a valuable source of data for all members of the health care team. Its purposes include interdisciplinary communication, legal documentation, financial billing (reimbursement), education, research, and auditing/monitoring. Nursing process is a way of thinking and performing nursing care; it is not a purpose of a health care record.

260
Q

A nurse is developing a plan to reduce data entry errors and maintain confidentiality. Which guidelines should the nurse include? (Select all that apply.)

a. Bypass the firewall.
b. Implement an automatic sign-off.
c. Create a password with just letters.
d. Use a programmed speed-dial key when faxing.
e. Impose disciplinary actions for inappropriate access.
f. Shred papers containing personal health information (PHI).

A

ANS: B, D, E, F
When faxing, use programmed speed-dial keys to eliminate the chance of a dialing error and misdirected information. An automatic sign-off is a safety mechanism that logs a user off the computer system after a specified period of inactivity. Disciplinary action, including loss of employment, occurs when nurses or other health care personnel inappropriately access patient information. All papers containing PHI (e.g., Social Security number, date of birth or age, patient’s name or address) must be destroyed immediately after you use or fax them. Most agencies have shredders or locked receptacles for shredding and incineration. Strong passwords use combinations of letters, numbers, and symbols that are difficult to guess. A firewall is a combination of hardware and software that protects private network resources (e.g., the information system of the hospital) from outside hackers, network damage, and theft or misuse of information and should not be bypassed.

261
Q

MATCHING
A nurse is using focused charting. Match the chart entry to the correct letter of the acronym.

a. Applied oxygen, stayed with patient, and instructed to slow breathing.
b. Patient states, “feel better,” respirations 16 with O2 saturations 96%.
c. Patient states, “can’t catch my breath and chest hurts.” Confused.

  1. D
  2. A
  3. R
A

ANS:

  1. C
  2. A
  3. B
262
Q

A nurse is teaching a patient about medications. Which statement from the patient indicates teaching is
effective?

a. “My parenteral medication must be taken with food.”
b. “I will rotate the sites in my left leg when I give my insulin.”
c. “Once I start feeling better, I will stop taking my antibiotic.”
d. “If I am 30 minutes late taking my medication, I should skip that dose.”

A

ANS: B
For daily insulin, rotate site within anatomical area. Rotating injections within the same body part (instrasite rotation) provides greater consistency in absorption of medication. Parenteral medication absorption is not affected by the timing of meals. Taking a medication 30 minutes late is within the 60-minute window of the time medications should be taken. Medications are usually stopped based on the provider’s orders except in extenuating circumstances. With some medications, such as antibiotics, it is crucial that the full course of medication is taken to avoid relapse of infection.

263
Q

. A nurse is preparing to administer an injection to a patient. Which statement made by the patient is an indication for the nurse to use the Z-track method?

a. “I am allergic to many medications.”
b. “I’m really afraid that a big needle will hurt.”
c. “The last shot like that turned my skin colors.”
d. “My legs are too obese for the needle to go through.”

A

ANS: C
The Z-track is indicated when the medication being administered has the potential to irritate sensitive tissues. It is recommended that, when administering IM injections, the Z-track method be used to minimize local skin irritation by sealing the medication in muscle tissue. The Z-track method is not meant to reduce discomfort from the procedure. If a patient is allergic to a medication, it should not be administered. If a patient has additional subcutaneous tissue to go through, a needle of a different size may be selected.

264
Q

A 2-year-old child is ordered to have eardrops daily. Which action will the nurse take?

a. Pull the auricle down and back to straighten the ear canal.
b. Pull the auricle upward and outward to straighten the ear canal.
c. Sit the child up for 2 to 3 minutes after instilling drops in ear canal.
d. Sit the child up to insert the cotton ball into the innermost ear canal.

A

ANS: A
Children up to 3 years of age should have the auricle pulled down and back, children 3 years of age to adults should have the auricle pulled upward and outward. Solution should be instilled 1 cm (1/2 in) above the opening of the ear canal. The patient should remain in the side-lying position 2 to 3 minutes. If a cotton ball is needed, place it into the outermost part of the ear canal.

265
Q

A patient has an order to receive 0.3 mL of U-500 insulin. Which syringe will the nurse obtain to administer the medication?

a. 3-mL syringe
b. U-100 syringe
c. Needleless syringe
d. Tuberculin syringe

A

ANS: D
Because there is no syringe currently designed to prepare U-500 insulin, many medication errors result with this kind of insulin. To prevent errors, ensure that the order for U-500 specifies units and volume (e.g., 150 units, 0.3 mL of U-500 insulin), and use tuberculin syringes to draw up the doses. A 3 mL and U-100 can result in inaccurate dosing. A needleless syringe will not be acceptable in this situation.

266
Q

A patient has an order to receive 12.5 mg of hydrochlorothiazide. The nurse has on hand a 25 mg tablet of hydrochlorothiazide. How many tablet(s) will the nurse administer?

a. 1/2 tablet
b. 1 tablet
c. 1 1/2 tablets
d. 2 tablets

A

ANS: A
1/2 tablet will be given. The nurse is careful to perform nursing calculations to ensure proper medication administration. The dose ordered is 12.5. The dose on hand is 25. 12.5/25 = 1/2 tablet.

267
Q

The patient is to receive phenytoin (Dilantin) at 0900. When will be the ideal time for the nurse to schedule a trough level?

a. 0800
b. 0830
c. 0900
d. 0930

A

ANS: B
Trough levels are generally drawn 30 minutes before the drug is administered. If the medication is administered at 0900, the trough should be drawn at 0830.

268
Q

A patient is receiving vancomycin. Which function is the priority for the nurses to assess?

a. Vision
b. Hearing
c. Heart tones
d. Bowel sounds

A

ANS: B

A side effect of vancomycin is ototoxicity—hearing. It does not affect vision, heart tones, or bowel sounds.

269
Q

A health care provider orders lorazepam (Ativan) 1 mg orally 2 times a day. The dose available is 0.5 mg per tablet. How many tablet(s) will the nurse administer for each dose?

a. 1
b. 2
c. 3
d. 4

A

ANS: B
The nurse will give 2 tablets. It will take 2 tablets (0.5) to equal 1 mg OR ordered dose (1) over dose on hand (0.5). 1/0.5 = 2 tablets.

270
Q

The nurse is preparing to administer an injection into the deltoid muscle of an adult patient. Which needle size and length will the nurse choose?

a. 18 gauge × 1 1/2 inch
b. 23 gauge × 1/2 inch
c. 25 gauge × 1 inch
d. 27 gauge × 5/8 inch

A

ANS: C
For an intramuscular injection into an adult deltoid muscle, a 25-gauge, 1-inch needle is recommended. An 18gauge needle is too big. While a 23-gauge needle can be used, a 1/2-inch needle is too small. A 27-gauge, 5/8 inch needle is used for intradermal.

271
Q

When the nurse administers an IM corticosteroid injection, the nurse aspirates. What is the rationale for the nurse aspirating?

a. Prevent the patient from choking.
b. Increase the force of the injection.
c. Ensure proper placement of the needle.
d. Reduce the discomfort of the injection.

A

ANS: C
The purpose of aspiration is to ensure that the needle is in the muscle and not in the vascular system. Blood return upon aspiration indicates improper placement, and the injection should not be given. While a patient can aspirate fluid and food into the lungs, this is not related to the reason for why a nurse pulls back the syringe plunger after inserting the needle (aspirates) before injecting the medication. Reducing discomfort and prolonging absorption time are not reasons for aspirating medications.

272
Q

The nurse is giving an IM injection. Upon aspiration, the nurse notices blood return in the syringe. What should the nurse do?
a. Administer the injection at a slower rate.

b. Withdraw the needle and prepare the injection again.
c. Pull the needle back slightly and inject the medication.
d. Give the injection and hold pressure over the site for 3 minutes.

A

ANS: B
Blood return upon aspiration indicates improper placement, and the injection should not be given. Instead withdraw the needle, dispose of the syringe and needle properly, and prepare the medication again. Administering the medication into a blood vessel could have dangerous adverse effects, and the medication will be absorbed faster than intended owing to increased blood flow. Holding pressure is not an appropriate intervention. Pulling back the needle slightly does not guarantee proper placement of the needle and medication administration.

273
Q

. The nurse is planning to administer a tuberculin test with a 27-gauge, -inch needle. At which angle will the nurse insert the needle?

a. 15 degree
b. 30 degree
c. 45 degree
d. 90 degree

A

ANS: A
A 27-gauge, -inch needle is used for intradermal injections such as a tuberculin test, which should be inserted at a 5- to 15-degree angle, just under the dermis of the skin. Placing the needle at 30 degrees, 45 degrees, or 90 degrees will place the medication too deep.

274
Q

The nurse closely monitors an older adult for signs of medication toxicity. Which physiological change is the reason for the nurse’s action?

a. Reduced glomerular filtration
b. Reduced esophageal stricture
c. Increased gastric motility
d. Increased liver mass

A

ANS: A
The reduced glomerular filtration rate delays excretion, increasing chance for toxicity. In older adults, gastric motility and liver mass decrease. Esophageal stricture is not a physiological change associated with normal aging.

275
Q

. A registered nurse interprets that a scribbled medication order reads 25 mg. The nurse administers 25 mg of the medication to a patient and then discovers that the dose was incorrectly interpreted and should have been 15 mg. Who is ultimately responsible for the error?

a. Health care provider
b. Pharmacist
c. Hospital
d. Nurse

A

ANS: D
Ultimately, the person administering the medication is responsible for ensuring that it is correct. The nurse administered the medication, so in this case it is the nurse. Accept full accountability and responsibility for all actions surrounding the administration of medications. Do not assume that a medication that is ordered for a patient is the correct medication or the correct dose. This is the importance of verifying the six rights of medication administration. The ultimate responsibility and accountability are with the nurse, not the health care provider, pharmacist, or hospital.

276
Q

. A patient is to receive a proton pump inhibitor through a nasogastric (NG) feeding tube. Which is the most important nursing action to ensure effective absorption?

a. Thoroughly shake the medication before administering.
b. Position patient in the supine position for 30 minutes to 1 hour.
c. Hold feeding for at least 30 minutes after medication administration.
d. Flush tube with 10 to 15 mL of water, after all medications are administered.

A

ANS: C
If a medication needs to be given on an empty stomach or is not compatible with the feeding (e.g., phenytoin, carbamazepine [Tegretol], warfarin [Coumadin], fluoroquinolones, proton pump inhibitors), hold the feeding for at least 30 minutes before or 30 minutes after medication administration. Thoroughly shaking the medication mixes the medication before administration but does not affect absorption. Flushing the tube after all medications should be 30 to 60 mL of water; 15 to 30 mL of water is used for flushing between medications. Patients with NG tubes should never be positioned supine but instead should be positioned at least to a 30-degree angle to prevent aspiration, provided no contraindication condition is known.

277
Q

A health care provider prescribes aspirin 650 mg every 4 hours PO when febrile. For which patient will this order be appropriate?

a. 7 year old with a bleeding disorder
b. 21 year old with a sprained ankle
c. 35 year old with a severe headache from hypertension
d. 62 year old with a high fever from an infection

A

ANS: D
Aspirin is an analgesic, an antipyretic, and an anti-inflammatory medication. The provider wrote the medication to be given for a fever (febrile). Fevers are common in infections. If a child is bleeding, aspirin would be contraindicated; aspirin increases the likelihood of bleeding. Although it can be used for inflammatory problems (sprained ankle) and pain/analgesia (severe headache), this is not how the order was written.

278
Q

A patient is in need of immediate pain relief for a severe headache. Which medication will the nurse administer to be absorbed the quickest?

a. Acetaminophen 650 mg PO
b. Hydromorphone 4 mg IV
c. Ketorolac 8 mg IM
d. Morphine 6 mg SQ

A

ANS: B
IV is the fastest route for absorption owing to the increase in blood flow. The richer the blood supply to the site of administration, the faster a medication is absorbed. Medications administered intravenously enter the bloodstream and act immediately, whereas those given in other routes take time to enter the bloodstream and have an effect. Oral, subcutaneous (SQ), and intramuscular (IM) are others ways to deliver medication but with less blood flow, slowing absorption.

279
Q

While preparing medications, the nurse knows one of the drug is an acidic medication. In which area does the nurse anticipate the drug will be absorbed?

a. Stomach
b. Mouth
c. Small intestine
d. Large intestine

A

ANS: A
Acidic medications pass through the gastric mucosa rapidly. Medications that are basic are not absorbed before reaching the small intestine.

280
Q

. The nurse administers a central nervous system stimulant to a patient. Which assessment finding indicates to the nurse that an idiosyncratic event is occurring?

a. Falls asleep during daily activities
b. Presents with a pruritic rash
c. Develops restlessness
d. Experiences alertness

A

ANS: A
An idiosyncratic event is a reaction opposite to what the effects of the medication normally are, or the patient overreacts or underreacts to the medication. Falls asleep is an opposite effect of what a central nervous system stimulant should do. A stimulant should make a patient restless and alert. A pruritic (itch) rash could indicate an allergic reaction.

281
Q

An order is written for phenytoin 500 mg IM q3-4h prn for pain. The nurse recognizes that treatment of pain is not a standard therapeutic indication for this drug. The nurse believes that the health care provider meant to write hydromorphone. What should the nurse do?

a. Call the health care provider to clarify the order.
b. Give the patient hydromorphone, as it was meant to be written.
c. Administer the medication and monitor the patient frequently.
d. Refuse to give the medication and notify the nurse supervisor.

A

ANS: A
If there is any question about a medication order because it is incomplete, illegible, vague, or not understood, contact the health care provider before administering the medication. The nurse cannot change the order without the prescriber’s consent; this is out of the nurse’s scope of practice. Ultimately, the nurse can be held responsible for administering an incorrect medication. If the prescriber is unwilling to change the order and does not justify the order in a reasonable and evidence-based manner, the nurse may refuse to give the medication and notify the supervisor.

282
Q

A patient needs assistance in eliminating an anesthetic gaseous medication (nitrous oxide). Which action will the nurse take?

a. Encourage the patient to cough and deep-breathe.
b. Suction the patient’s respiratory secretions.
c. Suggest voiding every 2 hours.
d. Increase fluid intake.

A

ANS: A
Gaseous and volatile medications are excreted through gas exchange (lungs). Deep breathing and coughing will assist in clearing the medication more quickly. It is a gaseous medication and cannot be suctioned out of the lungs. It is not excreted through the kidneys so fluids and voiding will not help.

283
Q

A nurse has withdrawn a narcotic from the medication dispenser and must waste a portion of the medication. What should the nurse do?

a. Have another nurse witness the wasted medication.
b. Return the wasted medication to the medication dispenser.
c. Place the wasted portion of the medication in the sharps container.
d. Exit the medication room to call the health care provider to request an order that matches the dosages.

A

ANS: A
The nurse should follow Nurse Practice Acts and safe narcotic administration guidelines by having a nurse witness the “wasted” medication. The nurse cannot return the wasted medication to the medication dispenser. Wasted portions of medications are not placed in sharps containers. The nurse should not leave the narcotic unattended and call the health care provider to obtain matching dosages; the nurse is expected to obtain the correct dose.

284
Q

A nurse teaches the patient about the prescribed buccal medication. Which statement by the patient indicates teaching by the nurse is successful?

a. “I should let the medication dissolve completely.”
b. “I will place the medication in the same location.”
c. “I can only drink water, not juice, with this medication.”
d. “I better chew my medication first for faster distribution.”

A

ANS: A
Buccal medications should be placed in the side of the cheek and allowed to dissolve completely. Buccal medications act with the patient’s saliva and mucosa. The patient should not chew or swallow the medication or take any liquids with it. The patient should rotate sides of the cheek to avoid irritating the mucosal lining.

285
Q

What is the nurse’s priority action to protect a patient from medication error?

a. Reading medication labels at least 3 times before administering
b. Administering as many of the medications as possible at one time
c. Asking anxious family members to leave the room before giving a medication
d. Checking the patient’s room number against the medication administration record

A

ANS: A
One step to take to prevent medication errors is to read labels at least 3 times before administering the medication. The nurse should address the family’s concerns about medications before administering them. Do not discount their anxieties. The medication administration record should be checked against the patient’s hospital identification band; a room number is not an acceptable identifier. Medications should be given when scheduled, and medications with special assessment indications should be separated. Giving medications at one time can cause the patient to aspirate.

286
Q

The nurse prepares a pain injection for a patient but had to check on another patient and asks a new nurse to give the medication. Which action by the new nurse is best?

a. Do not give the medication.
b. Administer the medication just this once.
c. Give the medication for any pain score greater than 8.
d. Avoid the issue and pretend to not hear the request.

A

ANS: A
Because the nurse who administers the medication is responsible for any errors related to it, nurses administer only the medications they prepare. You cannot delegate preparation of medication to another person and then administer the medication to the patient. The right medication cannot be verified by the new nurse; do not violate the six rights. Do not administer the medication even one time. Do not administer the medication regardless of the pain rating. Avoiding the issue is not appropriate or safe.

287
Q

A patient is at risk for aspiration. Which nursing action is most appropriate?

a. Give the patient a straw to control the flow of liquids.
b. Have the patient self-administer the medication.
c. Thin out liquids so they are easier to swallow.
d. Turn the head toward the stronger side.

A

ANS: B
Aspiration occurs when food, fluid, or medication intended for GI administration inadvertently enters the respiratory tract. To minimize aspiration risk, allow the patient, if capable, to self-administer medication. Patients should also hold their own cup to control how quickly they take in fluid. Some patients at risk for aspiration may require thickened liquids; thinning liquids does not decrease aspiration risk. Patients at risk for aspiration should not be given straws because use of a straw decreases the control the patient has over volume intake. Turning the head toward the weaker side helps the medication move down the stronger side of the esophagus.

288
Q

. A patient refuses medication. Which is the nurse’s first action?

a. Educate the patient about the importance of the medication.
b. Discreetly hide the medication in the patient’s favorite gelatin.
c. Agree with the patient’s decision and document it in the chart.
d. Explore with the patient reasons for not wanting to take the medication.

A

ANS: D
The first response is to explore reasons the patient does not want the medication. After the assessment, the nurse can decide what to do next. Educating is important, but it is not the first action. Ultimately, the patient does have the right to refuse the medication; however, the nurse should first try to find out reasons for the refusal and provide education if needed based upon the assessment findings. Hiding medication and deceiving a patient into taking a medication is unethical and violates right to autonomy.

289
Q

A patient who is being discharged today is going home with an inhaler. The patient is to administer 2 puffs of the inhaler twice daily. The inhaler contains 200 puffs. When should the nurse appropriately advise the patient to refill the medication?

a. 6 weeks from the start of using the inhaler
b. As soon as the patient leaves the hospital
c. When the inhaler is half empty.
d. 50 days after discharge

A

ANS: A
Six weeks will be about the time the inhaler will need to be refilled. The inhaler should last the patient 50 days (2 puffs × 2/twice daily = 4; 200/4 = 50); the nurse should advise the patient to refill the prescription when there are 7 to 10 days of medication remaining. Refilling it as soon as the patient leaves the hospital or when the inhaler is half empty is too early. If the patient waits 50 days, the patient will run out of medication before it can be refilled.

290
Q

The supervising nurse is watching nurses prepare medications. Which action by one of the nurses will the supervising nurse stop immediately?

a. Rolls insulin vial between hands
b. Administers a dose of correction insulin
c. Draws up glargine (Lantus) in a syringe by itself
d. Prepares NPH insulin to be given intravenously (IV)

A

ANS: D
The only insulin that can be given IV is regular. NPH cannot be given IV and must be stopped. All the rest demonstrate correct practice. Insulin is supposed to be rolled, not shaken. Glargine is supposed to be given by itself; it cannot be mixed with another medication. Correction insulin, also known as sliding-scale insulin, provides a dose of insulin based on the patient’s blood glucose level. The term correction insulin is preferred because it indicates that small doses of rapid- or short-acting insulins are needed to correct a patient’s elevated blood sugar

291
Q

Which patient does the nurse most closely monitor for an unintended synergistic effect?
a. The 4 year old who has mistakenly taken a half bottle of vitamins

b. The 35 year old who has ingested meth mixed with several household chemicals
c. The 50 year old who is prescribed a second blood pressure medication
d. The 72 year old who is seeing four different specialists

A

ANS: D
The 72 year old seeing four different providers is likely to experience polypharmacy. Polypharmacy places the patient at risk for unintended mixing of medications that potentiate each other. When two medications have a synergistic effect, their combined effect is greater than the effect of the medications when given separately. The child taking too much of a medication by mistake could experience overdose or toxicity. The 50 year old is prescribed two different blood pressure medications for their synergistic effect, but this is a desired, intended event. A patient taking meth and mixing chemicals can be toxic.

292
Q

. Which patient using an inhaler would benefit most from using a spacer?

a. A 15 year old with a repaired cleft palate who is alert
b. A 25 year old with limited coordination of the extremities
c. A 50 year old with hearing impairment who uses a hearing aid
d. A 72 year old with left-sided hemiparesis using a dry powder inhaler

A

ANS: B
A spacer is indicated for a patient who has difficulty coordinating the steps, like patients with limited mobility/coordination. An alert adolescent with a repaired cleft palate would not need a spacer. Hearing impairment may make teaching the patient to use the inhaler difficult, but it does not indicate the need for a spacer. Although a patient with left-sided hemiparesis could have coordination problems, a patient using a dry powder inhaler does not require the use of spacers.

293
Q

The prescriber wrote for a 40-kg child to receive 25 mg of medication 4 times a day. The therapeutic range is 5 to 10 mg/kg/day. What is the nurse’s priority?

a. Change the dose to one that is within range.
b. Administer the medication because it is within the therapeutic range.
c. Notify the health care provider that the prescribed dose is in the toxic range.
d. Notify the health care provider that the prescribed dose is below the therapeutic range.

A

ANS: D
The dosage range is 200 to 400 mg a day (5 × 40 = 200 and 10 × 40 = 400). The prescribed dose is 100 mg/day (4 × 25 = 100), which is below therapeutic range. The nurse should notify the health care provider first and ask for clarification on the order. The dose is not above the therapeutic range and is not at a toxic level. The nurse should never alter an order without the prescriber’s approval and consent.

294
Q

The supervising nurse is observing several different nurses. Which action will cause the supervising nurse to intervene?

a. A nurse administers a vaccine without aspirating.
b. A nurse gives an IV medication through a 22-gauge IV needle without blood return.
c. A nurse draws up the NPH insulin first when mixing a short-acting and intermediate-acting insulin.
d. A nurse calls the health care provider for a patient with nasogastric suction and orders for oral meds.

A

ANS: C
The supervising nurse must intervene with the nurse who is drawing up the NPH insulin first; if regular and intermediate-acting (NPH) insulin is ordered, prepare the regular insulin first to prevent the regular insulin from becoming contaminated with the intermediate-acting insulin. All the other actions are appropriate and do not need follow-up. The CDC no longer recommends aspiration when administering immunizations to reduce discomfort. In some cases, especially with a smaller gauge (22) IV needle, blood return is not aspirated, even if the IV is patent. If the IV site shows no signs of infiltration and IV fluid is infusing without difficulty, proceed with IV push slowly. Oral meds are contraindicated in patients with nasogastric suction.

295
Q

A nurse is caring for a patient who is receiving pain medication through a saline lock. After obtaining a good blood return when the nurse is flushing the patient’s peripheral IV, the patient reports pain. Upon assessment, the nurse notices a red streak that is warm and tender to the touch. What is the nurse’s initial action?

a. Do not administer the pain medication.
b. Administer the pain medication slowly.
c. Apply a warm compress to the site.
d. Apply a cool compress to the site.

A

ANS: A
The patient has phlebitis; the initial nursing action is do not administer the medication. The medication should not be given slowly. A cool or warm compress may be used later depending upon protocol, but it is not the first action.

296
Q

. The nurse is preparing to administer medications to two patients with the same last name. After the administration, the nurse realizes that did not check the identification of the patient before administering medication. Which action should the nurse complete first?

a. Return to the room to check and assess the patient.
b. Administer the antidote to the patient immediately.
c. Alert the charge nurse that a medication error has occurred.
d. Complete proper documentation of the medication error in the patient’s chart.

A

ANS: A
When an error occurs, the patient’s safety and well-being are the top priorities. You first assess and examine the patient’s condition and notify the health care provider of the incident as soon as possible. The nurse’s first priority is to establish the safety of the patient by assessing the patient. Second, notify the charge nurse and the health care provider. Administer antidote if required. Finally, the nurse needs to complete proper documentation.

297
Q

The nurse is caring for two patients with the same last name. In this situation which right of medication administration is the priority to reduce the chance of an error?

a. Right medication
b. Right patient
c. Right dose
d. Right route

A

ANS: B
The nurse should ask the patient to verify identity and should check the patient’s ID bracelet against the medication record to ensure right patient. Acceptable patient identifiers include the patient’s name, an identification number assigned by a health care facility, or a telephone number. Do not use the patient’s room number as an identifier. To identify a patient correctly in an acute care setting, compare the patient identifiers on the MAR with the patient’s identification bracelet while at the patient’s bedside. Right medication, right dose, and right route are equally as important, but in this situation, right patient is the priority (two patients with the same last name).

298
Q

. A patient prefers not to take the daily allergy pill this morning because it causes drowsiness throughout the day. Which response by the nurse is best?

a. “The physician ordered it; therefore, you must take your medication every morning at the same time whether you’re drowsy or not.”
b. “Let’s see if we can change the time you take your pill to 9 PM, so the drowsiness occurs when you would normally be sleeping.”
c. “You can skip this medication on days when you need to be awake and alert.”
d. “Try to get as much done as you can before you take your pill, so you can sleep in the afternoon.”

A

ANS: B
The nurse should use knowledge about the medication to educate the patient about potential response to medications. Then the medication schedule can be altered based on that knowledge. It is the patient’s right to refuse medication; however, the nurse should educate the patient on the importance and effects of the medication. Asking a patient to fit a schedule around a medication is unreasonable and will decrease compliance. The nurse should be supportive and should offer solutions to manage medication effects.

299
Q

. A nurse is preparing to administer a medication from a vial. In which order will the nurse perform the steps, starting with the first step?

  1. Invert the vial.
  2. Fill the syringe with medication.
  3. Inject air into the airspace of the vial.
  4. Clean with alcohol swab and allow to dry.
  5. Pull back on the plunger the amount to be drawn up.
  6. Tap the side of the syringe barrel to remove air bubbles.

a. 4, 1, 5, 3, 6, 2
b. 1, 4, 5, 3, 2, 6

c. 4, 5, 3, 1, 2, 6
d. 1, 4, 5, 3, 6, 2

A

ANS: C
When preparing medication from a vial, the steps are as follows: Firmly and briskly wipe the surface of the rubber seal with an alcohol swab and allow to dry; pull back on the plunger to draw an amount of air into the syringe equal to the volume of medication to be aspirated from the vial; inject air into the airspace of the vial; invert the vial while keeping firm hold on the syringe and plunger; fill the syringe with medication; and tap the side of the syringe barrel carefully to dislodge any air bubbles

300
Q

A nurse is attempting to administer an oral medication to a child, but the child refuses to take the medication. A parent is in the room. Which statement by the nurse to the parent is best?

a. “Please hold your child’s arms down, so I can give the full dose.”
b. “I will prepare the medication for you and observe if you would like to try to administer the medication.”
c. “Let’s turn the lights off and give your child a moment to fall asleep before administering the medication.”
d. “Since your child loves applesauce, let’s add the medication to it, so your child doesn’t resist.”

A

ANS: B
Children often have difficulties taking medication, but it can be less traumatic for the child if the parent administers the medication and the nurse supervises. Another nurse should help restrain a child if needed; the parent acts as a comforter, not a restrainer. Holding down the child is not the best option because it may further upset the child. Never administer an oral medication to a sleeping child. Don’t mix medications into the child’s favorite foods, because the child might start to refuse the food.

301
Q

An older-adult patient needs an IM injection of antibiotic. Which site is best for the nurse to use?

a. Deltoid
b. Dorsal gluteal
c. Ventrogluteal
d. Vastus lateralis

A

ANS: C
The ventrogluteal site is the preferred and safest site for all adults, children, and infants. While the vastus lateralis is a large muscle that could be used it is not the preferred and safest. The dorsal gluteal site is a location for a subcutaneous injection, and this patient requires an IM injection. The deltoid is easily accessible, but this muscle is not well developed and is not the preferred site.

302
Q

A nurse is preparing an intravenous IV piggyback infusion. In which order will the nurse perform the steps, starting with the first one?

  1. Compare the label of the medication with the medication administration record at the patient’s bedside.
  2. Connect the tubing of the piggyback infusion to the appropriate connector on the upper Y-port.
  3. Hang the piggyback medication bag above the level of the primary fluid bag.
  4. Clean the main IV line port with an antiseptic swab.
  5. Connect the infusion tubing to the medication bag.
  6. Regulate flow.

a. 5, 2, 1, 4, 3, 6
b. 5, 2, 1, 3, 4, 6
c. 1, 5, 4, 3, 2, 6
d. 1, 5, 3, 4, 2, 6

A

ANS: D
When preparing and administering IV piggybacks, use the following steps: Compare the label of medication with the medication administration record at the patient’s bedside; connect the infusion tubing to the medication bag; hang the piggyback medication bag above the level of the primary fluid bag; clean the main IV line port with an antiseptic swab; connect the tubing of the piggyback infusion to the appropriate connector on the upper Y-port; and regulate flow.

303
Q

A nurse is administering oral medications to patients. Which action will the nurse take?

a. Remove the medication from the wrapper, and place it in a cup labeled with the patient’s information.
b. Place all of the patient’s medications in the same cup, except medications with assessments.
c. Crush enteric-coated medication, and place it in a medication cup with water.
d. Measure liquid medication by bringing liquid medication cup to eye level.

A

ANS: B
Placing medications that require preadministration assessment in a separate cup serves as a reminder to check before the medication is given, making it easier for the nurse to withhold medication if necessary. Medications should not be removed from their package until they are in the patient’s room because this makes identification of the pill easier and reduces contamination. When measuring a liquid, the nurse should use the meniscus level to measure; make sure it is at eye level on a hard surface like a countertop. Enteric coated medications should not be crushed.

304
Q

A nurse is performing the three accuracy checks before administering an oral liquid medication to a patient. When will the nurse perform the second accuracy check?

a. At the patient’s bedside
b. Before going to the patient’s room
c. When checking the medication order
d. When selecting medication from the unit-dose drawer

A

ANS: B
Before going to the patient’s room, compare the patient’s name and name of medication on the label of prepared drugs with MAR for the second accuracy check. Selecting the correct medication from the stock supply, unit-dose drawer, or automated dispensing system (ADS) is the first check. The third accuracy check is comparing names of medications on labels with MAR at the patient’s bedside. Checking the orders is not one of the three accuracy checks but should be done if there is any confusion about an order.

305
Q

A nurse is preparing to administer an antibiotic medication at 1000 to a patient but gets busy in another room. When should the nurse give the antibiotic medication?

a. By 1030
b. By 1100
c. By 1130
d. By 1200

A

ANS: A
Give time-critical scheduled medications (e.g., antibiotics, anticoagulants, insulin, anticonvulsants, and immunosuppressive agents) at the exact time ordered (within 30 minutes before or after scheduled dose). Give non–time critical scheduled medications within a range of either 1 or 2 hours of scheduled dose. 1100, 1130, and 1200 are too late.

306
Q

The nurse is administering medications to several patients. Which action should the nurse take?

a. Advise a patient after a corticosteroid inhaler treatment to rinse mouth with water.
b. Administer an intravenous medication through tubing that is infusing blood.
c. Pinch up the deltoid muscle of an adult patient receiving a vaccination.
d. Aspirate before administering a subcutaneous injection in the abdomen.

A

ANS: A
If the patient uses a corticosteroid, have him or her rinse the mouth out with water or salt water or brush teeth after inhalation to reduce risk of fungal infection. Piercing a blood vessel during a subcutaneous injection is very rare. Therefore, aspiration is not necessary when administering subcutaneous injections. When giving immunizations to adults: to avoid injection into subcutaneous tissue, spread the skin of the selected vaccine administration site taut between the thumb and forefinger, isolating the muscle. Never administer IV medications through tubing that is infusing blood, blood products, or parenteral nutrition solutions.

307
Q

A nurse is following safety principles to reduce the risk of needlestick injury. Which actions will the nurse take? (Select all that apply.)

a. Recap the needle after giving an injection.
b. Remove needle and dispose in sharps box.
c. Never force needles into the sharps disposal.
d. Use clearly marked sharps disposal containers.
e. Use needleless devices whenever possible.

A

ANS: C, D, E
Needles should not be forced into the box. Clearly mark receptacles to warn of danger. Using needleless systems when possible will further reduce the risk of needlestick injury. To prevent the risk of needlesticks, the nurse should never recap needles. The syringe and sheath are disposed of together in a receptacle.

308
Q

Which methods will the nurse use to administer an intravenous (IV) medication that is incompatible with the patient’s IV fluid? (Select all that apply.)

a. Start another IV site.
b. Administer slowly with the IV fluid.
c. Do not give the medication and chart.
d. Flush with 10 mL of sterile water before and after administration.
e. Flush with 10 mL of normal saline before and after administration.

A

ANS: A, D, E
When IV medication is incompatible with IV fluids, stop the IV fluids, clamp the IV line above the injection site, flush with 10 mL of normal saline or sterile water, give the IV bolus over the appropriate amount of time, flush with another 10 mL of normal saline or sterile water at the same rate as the medication was administered, and restart the IV fluids at the prescribed rate. Do not administer the drug slowly with the IV; this is contraindicated when incompatibility exist. Not giving the medication and charting is inappropriate; this is not a prudent or safe action by the nurse.

309
Q

A patient is taking 1 tablet of hydrocodone bitartrate 5 mg and acetaminophen 500 mg (Vicodin) every 4 hours. The patient is also taking 2 tablets of acetaminophen (Tylenol) 325 mg every 12 hours. How many grams of acetaminophen is the patient taking daily? Record your answer using one decimal place. ______ g

A

ANS:
3.3
The nurse should calculate the dosage taken via the first medication and add it to the daily intake of the second medication. Then, convert milligrams to grams.
500 mg × 6 doses a day = 3000 mg/day + (2 tablets × 325 mg) × 2 doses a day = 1300 mg/day = 3300 mg/day total of acetaminophen; 3300 mg converted to grams = 3.3 grams.

310
Q

The nurse is administering 250 mg of a medication elixir to the patient. The medication comes in a dose of 1000 mg/5 mL. How many milliliters should the nurse administer? Record your answer using two decimal places. ____ mL

A

ANS:
1.25
The nurse needs to first determine how many milligrams are in each milliliter of the elixir. Then the nurse calculates how many milliliters would contain 250 mg.
1000 mg/5 mL = 200 mg/1 mL
250 mg/(X mL) = 200 mg/mL = 1.25 mL. OR Dose ordered over dose on hand (250/1000) × volume or amount on hand (5). 250/1000 = 0.25 × 5 = 1.25 mL.

311
Q

The patient is to receive amoxicillin 500 mg q8h; the medication is dispensed at 250 mg/5 mL. How many milliliters will the nurse administer for one dose? Record your answer using a whole number. ___ mL

A

ANS:
10
The drug is dispensed at 250 mg/5 mL. The nurse is to give 500 mg, which is 10 mL. OR Dose ordered over dose on hand (500/250) × volume or amount on hand (5). 500/250 × 5 = 10 mL.

312
Q

. The nurse is caring for a surgical patient, when the family member asks what perioperative nursing means. How should the nurse respond?

a. Perioperative nursing occurs in preadmission testing.
b. Perioperative nursing occurs primarily in the postanesthesia care unit.
c. Perioperative nursing includes activities before, during, and after surgery.
d. Perioperative nursing includes activities only during the surgical procedure.

A

ANS: C
Perioperative nursing care occurs before, during, and after surgery. Preadmission testing occurs before surgery and is considered preoperative. Nursing care provided during the surgical procedure is considered intraoperative, and in the postanesthesia care unit, it is considered postoperative. All of these are parts of the perioperative phase, but each individual phase does not explain the term completely.

313
Q

The nurse is caring for a patient who is scheduled to undergo a surgical procedure. The nurse is completing an assessment and reviews the patient’s laboratory tests and allergies and prepares the patient for surgery. In which perioperative nursing phase is the nurse working?

a. Perioperative
b. Preoperative
c. Intraoperative
d. Postoperative

A

ANS: B
Reviewing the patient’s laboratory tests and allergies is done before surgery in the preoperative phase. Perioperative means before, during, and after surgery. Intraoperative means during the surgical procedure in the operating suite; postoperative means after the surgery and could occur in the postanesthesia care unit, in the ambulatory surgical area, or on the hospital unit.

314
Q

The nurse is caring for a patient in the postanesthesia care unit. The patient has developed profuse bleeding from the surgical site, and the surgeon has determined the need to return to the operative area. How will the nurse classify this procedure?

a. Major
b. Urgent
c. Elective
d. Emergency

A

ANS: D
An emergency procedure must be done immediately to save a life or preserve the function of a body part. An example would be repair of a perforated appendix, repair of a traumatic amputation, or control of internal hemorrhaging. An urgent procedure is necessary for a patient’s health and often prevents additional problems from developing. An example would be excision of a cancerous tumor, removal of a gallbladder for stones, or vascular repair for an obstructed artery. An elective procedure is performed on the basis of the patient’s choice; it is not essential and is not always necessary for health. An example would be a bunionectomy, plastic surgery, or hernia reconstruction. A major procedure involves extensive reconstruction or alteration in body parts; it poses great risks to well-being. An example would be a coronary artery bypass or colon resection.

315
Q

The nurse is caring for a patient in preadmission testing. The patient has been assigned a physical status classification by the American Society of Anesthesiologists of ASA III. Which assessment will support this classification?

a. Normal, healthy patient
b. Denial of any major illnesses or conditions
c. Poorly controlled hypertension with implanted pacemaker
d. Moribund patient not expected to survive without the operation

A

ANS: C
An ASA III rating is a patient with a severe systemic disease, such as poorly controlled hypertension with an implanted pacemaker. ASA I is a normal healthy patient with no major illnesses or conditions. ASA II is a patient with mild systemic disease. ASA V is a moribund patient who is not expected to survive without the operation and includes patients with ruptured abdominal/thoracic aneurysm or massive trauma.

316
Q

The patient has presented to the ambulatory surgery center to have a colonoscopy. The patient is scheduled to receive moderate sedation (conscious sedation) during the procedure. How will the nurse interpret this information?

a. The procedure results in loss of sensation in an area of the body.
b. The procedure requires a depressed level of consciousness.
c. The procedure will be performed on an outpatient basis.
d. The procedure necessitates the patient to be immobile.

A
ANS: B
Moderate sedation (conscious sedation) is used routinely for procedures that do not require complete anesthesia but rather a depressed level of consciousness. Not all patients who are treated on an outpatient basis receive moderate sedation. Regional anesthesia such as local anesthesia provides loss of sensation in an area of the body. General anesthesia is used for patients who need to be immobile and to not remember the surgical procedure.
317
Q

The nurse is caring for a patient in the postanesthesia care unit who has undergone a left total knee arthroplasty. The anesthesia provider has indicated that the patient received a left femoral peripheral nerve block. Which assessment will be an expected finding for this patient?

a. Sensation decreased in the left leg
b. Patient report of pain in the left foot
c. Pulse decreased at the left posterior tibia
d. Left toes cool to touch and slightly cyanotic

A

ANS: A
Induction of regional anesthesia results in loss of sensation in an area of the body—in this case, the left leg. The peripheral nerve block influences the portions of sensory pathways that are anesthetized in the targeted area of the body. Decreased pulse, toes cool to touch, and cyanosis are indications of decreased blood flow and are not expected findings. Reports of pain in the left foot may indicate that the block is not working or is subsiding and is not an expected finding in the immediate postoperative period.

318
Q

The nurse is preparing a patient for surgery. Which goal is a priority for assessing the patient before surgery?

a. Plan for care after the procedure.
b. Establish a patient’s baseline of normal function.
c. Educate the patient and family about the procedure.
d. Gather appropriate equipment for the patient’s needs.

A

ANS: B
The goal of the preoperative assessment is to identify a patient’s normal preoperative function and the presence of any risks to recognize, prevent, and minimize possible postoperative complications. Gathering appropriate equipment, planning care, and educating the patient and family are all important interventions that must be provided for the surgical patient; they are part of the nursing process but are not the priority reason/goal for completing an assessment of the surgical patient.

319
Q

The nurse is completing a medication history for the surgical patient in preadmission testing. Which medication should the nurse instruct the patient to hold (discontinue) in preparation for surgery according to protocol?

a. Warfarin
b. Vitamin C
c. Prednisone
d. Acetaminophen

A

ANS: A
Medications such as warfarin or aspirin alter normal clotting factors and thus increase the risk of hemorrhaging. Discontinue at least 48 hours before surgery. Acetaminophen is a pain reliever that has no special implications for surgery. Vitamin C actually assists in wound healing and has no special implications for surgery. Prednisone is a corticosteroid, and dosages are often temporarily increased rather than held.

320
Q

The nurse is prescreening a surgical patient in the preadmission testing unit. The medication history indicates that the patient is currently taking an anticoagulant. Which action should the nurse take when consulting with the health care provider?

a. Ask for a radiological examination of the chest.
b. Ask for an international normalized ratio (INR).
c. Ask for a blood urea nitrogen (BUN).
d. Ask for a serum sodium (Na).

A

ANS: B
INR, PT (prothrombin time), APTT (activated partial thromboplastin time), and platelet counts reveal the clotting ability of the blood. Anticoagulants can be utilized for different conditions, but its action is to increase the time it takes for the blood to clot. This action can put the surgical patient at risk for bleeding tendencies. Typically, if at all possible, this medication is held several days before a surgical procedure to decrease this risk. Chest xray, BUN, and Na are diagnostic screening tools for surgery but are not specific to anticoagulants.

321
Q

The nurse is encouraging the postoperative patient to utilize diaphragmatic breathing. Which priority goal is the nurse trying to achieve?

a. Manage pain
b. Prevent atelectasis
c. Reduce healing time
d. Decrease thrombus formation

A

ANS: B
After surgery, patients may have reduced lung volume and may require greater effort to cough and deep breathe; inadequate lung expansion can lead to atelectasis and pneumonia. Purposely utilizing diaphragmatic breathing can decrease this risk. During general anesthesia, the lungs are not fully inflated during surgery and the cough reflex is suppressed, so mucus collects within airway passages. Diaphragmatic breathing does not manage pain; in some cases, if splinting and pain medications are not given, it can cause pain. Diaphragmatic breathing does not reduce healing time or decrease thrombus formation. Better, more effective interventions are available for these situations.

322
Q

The nurse is caring for a postoperative patient on the medical-surgical floor. Which activity will the nurse encourage to prevent venous stasis and the formation of thrombus?

a. Diaphragmatic breathing
b. Incentive spirometry
c. Leg exercises
d. Coughing

A

ANS: C
After general anesthesia, circulation slows, and when the rate of blood slows, a greater tendency for clot formation is noted. Immobilization results in decreased muscular contractions in the lower extremities; these promote venous stasis. Coughing, diaphragmatic breathing, and incentive spirometry are utilized to decrease atelectasis and pneumonia.

323
Q

The nurse is caring for a preoperative patient. The nurse teaches the principles and demonstrates leg exercises for the patient. The patient is unable to perform leg exercises correctly. What is the nurse’s best next step?

a. Encourage the patient to practice at a later date.
b. Assess for the presence of anxiety, pain, or fatigue.
c. Ask the patient why exercises are not being done.
d. Evaluate the educational methods used to educate the patient.

A

ANS: B
If the patient is unable to perform leg exercises, the nurse should look for circumstances that may be impacting the patient’s ability to learn. In this case, the patient can be anticipating the upcoming surgery and may be experiencing anxiety. The patient may also be in pain or may be fatigued; both of these can affect the ability to learn. Evaluation of educational methods may be needed, but in this case, principles and demonstrations are being utilized. Asking anyone “why” can cause defensiveness and may not help in attaining the answer. The nurse is aware that the patient is unable to do the exercises. Moving forward without ascertaining that learning has occurred will not help the patient in meeting goals.

324
Q

Which nursing assessment will indicate the patient is performing diaphragmatic breathing correctly?
a. Hands placed on the border of the rib cage with fingers extended will touch as the chest wall contracts.

b. Hands placed on the chest wall with fingers extended will separate as the chest wall contracts.
c. The patient will feel upward movement of the diaphragm during inspiration.
d. The patient will feel downward movement of the diaphragm during expiration

A

ANS: A
Positioning the hands along the borders of the rib cage allows the patient to feel movement of the chest and abdomen as the diaphragm descends and the lungs expand. As the patient takes a deep breath and slowly exhales, the middle fingers will touch while the chest wall contracts. The fingers will separate as the chest wall expands. The patient will feel normal downward movement of the diaphragm during inspiration and normal upward movement during expiration.

325
Q

The nurse is caring for a postoperative patient with an abdominal incision. The nurse provides a pillow to use during coughing. Which activity is the nurse promoting?

a. Pain relief
b. Splinting
c. Distraction
d. Anxiety reduction

A

ANS: B
Deep breathing and coughing exercises place additional stress on the suture line and cause discomfort. Splinting incisions with hands and a pillow provides firm support and reduces incisional pull. Providing a pillow during coughing does not provide distraction or reduce anxiety. Providing a pillow does not provide pain relief. Coughing can increase anxiety because it can cause pain. Analgesics provide pain relief.

326
Q

The nurse is encouraging a reluctant postoperative patient to deep breathe and cough. Which explanation can the nurse provide that may encourage the patient to comply?

a. “If you don’t deep breathe and cough, you will get pneumonia.”
b. “You will need to cough only a few times during this shift.”
c. “Let’s try clearing the throat because that will work just as well.”
d. “Deep breathing and coughing will clear out the anesthesia.”

A

ANS: D
Deep breathing and coughing expel retained anesthetic gases and facilitate a patient’s return to consciousness. Although it is correct that a patient may experience atelectasis and pneumonia if deep breathing and coughing are not performed, the way this is worded sounds threatening and could be communicated in a more therapeutic manner. Deep breathing and coughing are encouraged every 2 hours while the patient is awake. Just clearing the throat does not remove mucus from deeper airways.

327
Q

The nurse and the nursing assistive personnel are assisting a postoperative patient to turn in bed. To assist in minimizing discomfort, which instruction should the nurse provide to the patient?
a. “Close your eyes and think about something pleasant.”

b. “Hold your breath and count to three.”
c. “Grab my shoulders with your hands.”
d. “Place your hand over your incision.”

A

ANS: D
Instruct the patient to place the right hand over the incisional area to splint it, providing support and minimizing pulling during turning. Closing one’s eyes, holding one’s breath, and holding the nurse’s shoulders do not help support the incision during a turn.

328
Q

The nurse is preparing to assist the patient in using the incentive spirometer. Which nursing intervention should the nurse provide first?

a. Perform hand hygiene.
b. Explain use of the mouthpiece.
c. Instruct the patient to inhale slowly.
d. Place in the reverse Trendelenburg position.

A

ANS: A
Performing hand hygiene reduces microorganisms and should be performed first. Placing the patient in the correct position such as high Fowler’s for the typical postoperative patient or reverse Trendelenburg for the bariatric patient would be the next step in the process. Demonstration of use of the mouthpiece followed by the instruction to inhale slowly would be the last step in this scenario.

329
Q

The nurse and the nursing assistive personnel (NAP) are caring for a group of postoperative patients who need turning, coughing, deep breathing, incentive spirometer, and leg exercises. Which task will the nurse assign to the NAP?

a. Teach postoperative exercises.
b. Do nothing associated with postoperative exercises.
c. Document in the medical record when exercises are completed.
d. Inform the nurse if the patient is unwilling to perform exercises.

A

ANS: D
The nurse can delegate to the NAP to encourage patients to practice postoperative exercises regularly after instruction and to inform the nurse if the patient is unwilling to perform these exercises. The skills of demonstrating and teaching postoperative exercises and documenting are not within the scope of practice for the nursing assistant. Doing nothing is not appropriate.

330
Q

. The nurse is providing preoperative teaching for the ambulatory surgery patient who will be having a cyst removed from the right arm. Which will be the best explanation for diet progression after surgery?
a. “Start with clear liquids, soup, and crackers. Advance to a normal diet as tolerated.”

b. “Stay with ice chips for several hours. After that, you can have whatever you want.”
c. “Stay on clear liquids for 24 hours. Then you can progress to a normal diet.”
d. “Start with clear liquids for 2 hours and then full liquids for 2 hours. Then progress to a normal diet.”

A

ANS: A
Patients usually receive a normal diet the first evening after surgery unless they have undergone surgery on GI structures. Implement diet intake while judging the patient’s response. For example, provide clear liquids such as water, apple juice, broth, or tea after nausea subsides. If the patient tolerates liquids without nausea, advance the diet as ordered. There is no need to stay on ice chips for several hours or clear liquids for 24 hours after this procedure. Putting a time frame on the progression is too prescriptive. Progression should be adjusted for the patient’s needs.

331
Q

The nurse explains the pain relief measures available after surgery during preoperative teaching for a surgical patient. Which comment from the patient indicates the need for additional education on this topic?

a. “I will be asked to rate my pain on a pain scale.”
b. “I will have minimal pain because of the anesthesia.”
c. “I will take the pain medication as the provider prescribes it.”
d. “I will take my pain medications before doing postoperative exercises.”

A

ANS: B
Anesthesia will be provided during the procedure itself, and the patient should not experience pain during the procedure; however, this will not minimize the pain after surgery. Pain management is utilized after the postoperative phase. Inform the patient of interventions available for pain relief, including medication, relaxation, and distraction. The patient needs to know and understand how to take the medications that the health care provider will prescribe postoperatively. During the stay in the facility, the level of pain is frequently assessed by the nurses. Coordinating pain medication with postoperative exercises helps to minimize discomfort and allows the exercises to be more effective.

332
Q

The nurse is making a preoperative education appointment with a patient. The patient asks if a family member should come to the appointment. Which is the best response by the nurse?

a. “There is no need for an additional person at the appointment.”
b. “Your family can come and wait with you in the waiting room.”
c. “We recommend including family members at this appointment.”
d. “It is required that you have a family member at this appointment.”

A

ANS: C
Including family members in perioperative education is advisable. Often a family member is a coach for postoperative exercises when the patient returns from surgery. If anxious relatives do not understand routine postoperative events, it is likely that their anxiety will heighten the patient’s fears and concerns. Preoperative preparation of family members before surgery helps to minimize anxiety and misunderstanding. An additional person is needed at the appointment if at all possible, and he or she needs to be involved in the process, not just waiting in the waiting room; however, it is certainly not a requirement for actually completing the surgery that someone comes to this appointment.

333
Q

The nurse is reviewing the surgical consent with the patient during preoperative education and finds the patient does not understand what procedure will be completed. What is the nurse’s best next step?

a. Notify the health care provider about the patient’s question.
b. Explain the procedure that will be completed.
c. Continue with preoperative education.
d. Ask the patient to sign the form.

A

ANS: A
Surgery cannot be legally or ethically performed until the patient fully understands the need for a procedure and all the implications. It is the surgeon’s responsibility to explain the procedure, associated risks, benefits, alternatives, and possible complications. It is important for the nurse to pause with preoperative education to notify the health care provider of the patient’s questions. It is not within the nurse’s scope to explain the procedure. The nurse can certainly reinforce what the health care provider has explained, but the information needs to come from the health care provider. It is not prudent to ask a patient to sign a form for a procedure that he/she does not understand.

334
Q

. During preoperative assessment for a 7:30 AM (0730) surgery, the nurse finds the patient drank a cup of coffee this morning. The nurse reports this information to the anesthesia provider. Which action does the nurse anticipate next?

a. A delay in or cancellation of surgery
b. Questions regarding components of the coffee
c. Additional questions about why the patient had coffee
d. Instructions to determine what education was provided in the preoperative visit

A

ANS: A
The recommendations before nonemergent procedures requiring general and regional anesthesia or sedation/analgesia include fasting from intake of clear liquids for 2 or more hours. A delay in or cancellation of surgery will be in order for this case. Questions regarding components of the coffee, asking why, and evaluating the preoperative education may all be items to be addressed, especially from a performance improvement perspective, but at this time in caring for this patient, a delay or cancellation is in order to prevent aspiration.

335
Q

The nurse has administered a preoperative medication to the patient going to surgery. Which action will the nurse take next?

a. Notify the operating suite that the medication has been given.
b. Instruct the patient to call for help to go to the restroom.
c. Waste any unused medication according to policy.
d. Ask the patient to sign the consent for surgery.

A

ANS: B
Once a preoperative medication has been administered, instruct the patient to call for help when getting out of bed to prevent falls. For patient safety, explain the purpose of a preoperative medication and its effects. Notifying the operating suite that the medication has been given may be part of a facilities procedure but is not the best next step. It is important to have the patient sign consents before the patient has received medication that may make him/her drowsy. Wasting unused medication according to policy is important but is not the best next step.

336
Q

The nurse has completed a preoperative assessment for a patient going to surgery and gathers assessment data. Which will be the most important next step for the nurse to take?

a. Notify the operating suite that the patient has a latex allergy.
b. Document that the patient had a bath at home this morning.
c. Administer the ordered preoperative intravenous antibiotic.
d. Ask the nursing assistive personnel to obtain vital signs.

A

ANS: A
The most important step is notifying the operating suite of the patient’s latex allergy. Many products that contain latex are used in the operating suite and the postanesthesia care unit (PACU). When preparing for a patient with this allergy, special considerations are required from preparation of the room to the types of tubes, gloves, drapes, and instruments utilized. Obtaining vital signs, documenting, and administering medications are all part of the process and should be done—with the latex allergy in mind. However, making sure that the patient has a safe environment is the first step.

337
Q

The nurse is preparing a patient for a surgical procedure on the right great toe. Which action will be most important to include in this patient’s preparation?

a. Place the patient in a clean surgical gown.
b. Ask the patient to remove all hairpins and cosmetics.
c. Ascertain that the surgical site has been correctly marked.
d. Determine where the family will be located during the procedure.

A

ANS: C
Because errors have occurred in the past with patients undergoing the wrong surgery on the wrong site, the universal protocol guidelines have been implemented and are used with all invasive procedures. Part of this protocol includes marking the operative site with indelible ink. Knowing where the family is during a procedure, placing the patient in a clean gown, and asking the patient to remove all hairpins and cosmetics are important but are not most important in this list of items.

338
Q

The circulating nurse is caring for a patient intraoperatively. Which primary role of the circulating nurse will be implemented?

a. Suturing the surgical incision in the OR suite
b. Managing patient care activities in the OR suite
c. Assisting with applying sterile drapes in the OR suite
d. Handing sterile instruments and supplies to the surgeon in the OR suite

A

ANS: B
The circulating nurse is an RN who remains unscrubbed and uses the nursing process in the management of patient care activities in the OR suite. The circulating nurse also manages patient positioning, antimicrobial skin preparation, medications, implants, placement and function of intermittent pneumatic compression (IPC) devices, specimens, warming devices and surgical counts of instruments, and dressings. The RN first assistant collaborates with the surgeon by handling and cutting tissue, using instruments and medical devices, providing exposure of the surgical area and hemostasis, and suturing. The scrub nurse, who can be a registered nurse, a licensed practical nurse, or a surgical technologist, maintains the sterile field, assists with applying the sterile drapes, and hands sterile instruments and supplies to the surgeon.

339
Q

The nurse is caring for a patient in the preoperative holding area of an ambulatory surgery center. Which nursing action will be most appropriate for this area?

a. Count the sterile surgical instruments.
b. Empty the urinary drainage bag.
c. Check the surgical dressing.
d. Apply a warm blanket.

A

ANS: D
The temperature in the preoperative holding area and in adjacent operating suites is usually cold. Offer the patient an extra warm blanket. Counts are taken by the circulating and scrub nurses in the operating room. Emptying a urinary drainage bag and checking the surgical dressing occur in the postanesthesia care unit, not in the holding area.

340
Q

The nurse is caring for a patient in the operating suite. Which outcome will be most appropriate for this patient at the end of the intraoperative phase?

a. The patient will be free of burns at the grounding pad.
b. The patient will be free of nausea and vomiting.
c. The patient will be free of infection.
d. The patient will be free of pain.

A

ANS: A
A primary focus of intraoperative care is to prevent injury and complications related to anesthesia, surgery, positioning, and equipment use, including use of the electrical cautery grounding pad for the prevention of burns. The perioperative nurse is an advocate for the patient during surgery and protects the patient’s dignity and rights at all times. Signs and symptoms of infection do not have the time to present during the intraoperative phase. During the intraoperative phase, the patient is anesthetized and unconscious and typically has an endotracheal tube that prevents conversation. Nausea, vomiting, and pain typically begin in the postoperative phase of the experience.

341
Q

The nurse is concerned about the skin integrity of the patient in the intraoperative phase of surgery. Which action will the nurse take to minimize skin breakdown?
a. Encouraging the patient to bathe before surgery

b. Securing attachments to the operating table with foam padding
c. Periodically adjusting the patient during the surgical procedure
d. Measuring the time a patient is in one position during surgery

A

ANS: B
Although it may be necessary to place a patient in an unusual position, try to maintain correct alignment and protect the patient from pressure, abrasion, and other injuries. Special mattresses, use of foam padding, and attachments to the operating suite table provide protection for the extremities and bony prominences. Bathing before surgery helps to decrease the number of microbes on the skin. Periodically adjusting the patient during the surgical procedure is impractical and can present a safety issue with regard to maintaining sterility of the field and maintaining an airway. Measuring the time the patient is in one position may help with monitoring the situation but does not prevent skin breakdown.

342
Q

. The nurse is assessing a postoperative patient with a history of obstructive sleep apnea for airway obstruction. Which assessment finding will best alert the nurse to this complication?

a. Drop in pulse oximetry readings
b. Moaning with reports of pain
c. Shallow respirations
d. Disorientation

A

ANS: A
One of the greatest concerns after general anesthesia is airway obstruction, especially in patients with obstructive sleep apnea. A drop in oxygen saturation by pulse oximetry is a sign of airway obstruction in patients with obstructive sleep apnea. Weak pharyngeal/laryngeal muscle tone from anesthetics; secretions in the pharynx, bronchial tree, or trachea; and laryngeal or subglottic edema also contribute to airway obstruction. In the postanesthetic patient, the tongue is a major cause of airway obstruction. Shallow respirations are indicative of respiratory depression. Moaning and reports of pain are common in all surgical patients and are an expected event. Disorientation is common when first awakening from anesthesia but can be a sign of hypoxia.

343
Q

The nurse is caring for a patient in the operating suite who is experiencing hypercarbia, tachypnea, tachycardia, premature ventricular contractions, and muscle rigidity. Which condition does the nurse suspect the patient is experiencing?

a. Malignant hyperthermia
b. Fluid imbalance
c. Hemorrhage
d. Hypoxia

A

ANS: A
A life-threatening, rare complication of anesthesia is malignant hyperthermia. Malignant hyperthermia causes hypercarbia, tachycardia, tachypnea, premature ventricular contractions, unstable blood pressure, cyanosis, skin mottling, and muscular rigidity. It often occurs during anesthesia induction. Hypoxia would manifest with decreased oxygen saturation as one of its signs and symptoms. Fluid imbalance would be assessed with intake and output and can manifest with tachycardia and blood pressure fluctuations but does not have muscle rigidity. Hemorrhage can manifest with tachycardia and decreased blood pressure, along with a thready pulse. Usually some sign or symptom of blood loss is noted (e.g., drains, incision, orifice, and abdomen).

344
Q

The nurse is caring for a postoperative patient who has had a minimally invasive carpel tunnel repair. The patient has a temperature of 97° F and is shivering. Which reason will the nurse most likely consider as the primary cause when planning care?

a. Anesthesia lowers metabolism.
b. Surgical suites have air currents.
c. The patient is dressed only in a gown.
d. The large open body cavity contributed to heat loss.

A

ANS: A
The operating suite and recovery room environments are extremely cool. The patient’s anesthetically depressed level of body function results in lowering of metabolism and a fall in body temperature. Although the patient is dressed in a gown and there are air currents in the operating room, these are not the primary reasons for the low temperature. Also, the patient in this type of case does not have a large open body cavity to contribute to heat loss.

345
Q

The nurse is monitoring a patient in the postanesthesia care unit (PACU) for postoperative fluid and electrolyte imbalance. Which action will be most appropriate for the nurse to take?

a. Encourage copious amounts of water.
b. Start an additional intravenous (IV) line.
c. Measure and record all intake and output.
d. Weigh the patient and compare with preoperative weight.

A

ANS: C
Accurate recording of intake and output assesses renal and circulatory function. Measure and record all sources of intake and output. Encouraging copious amounts of water in a postoperative patient might encourage nausea and vomiting. In the PACU, it is impractical to weigh the patient while waking from surgery, but in the days afterward, it is a good assessment parameter for fluid imbalance. Starting an additional IV is not necessary and is not important at this juncture.

346
Q

The nurse is caring for a patient in the postanesthesia care unit. The patient asks for a bedpan and states to the nurse, “I feel like I need to go to the bathroom, but I can’t.” Which nursing intervention will be most appropriate initially?

a. Assess the patient for bladder distention.
b. Encourage the patient to wait a minute and try again.
c. Inform the patient that everyone feels this way after surgery.
d. Call the health care provider to obtain an order for catheterization.

A

ANS: A
Depending on the surgery, some patients do not regain voluntary control over urinary function for 6 to 8 hours after anesthesia. Palpate the lower abdomen just above the symphysis pubis for bladder distention. Another option is to use a bladder scan or
ultrasound to assess bladder volume. The nurse must assess before deciding if the patient can try again. Not everyone feels as if they need to go but can’t after surgery. Calling the health care provider is not the initial best action. The nurse needs to have data before calling the provider.

347
Q

The postanesthesia care unit (PACU) nurse transports the inpatient surgical patient to the medical-surgical floor. Before leaving the floor, the medical-surgical nurse obtains a complete set of vital signs. What is the rationale for this nursing action?

a. This is done to complete the first action in a head-to-toe assessment.
b. This is done to compare and monitor for vital sign variation during transport.
c. This is done to ensure that the medical-surgical nurse checks on the postoperative patient.
d. This is done to follow hospital policy and procedure for care of the surgical patient.

A

ANS: B
Before the PACU nurse leaves the acute care area, the staff nurse assuming care for the patient takes a complete set of vital signs to compare with PACU findings. Minor vital sign variations normally occur after transporting the patient. The PACU nurse reviews the patient’s information with the medical-surgical nurse, including the surgical and PACU course, physician orders, and the patient’s condition. While vital signs may or may not be the first action in a head-to-toe assessment, this is not the rationale for this situation. While following policy or ascertaining that the floor nurse checks on the patient are good reasons for safe care, they are not the best rationale for obtaining vital signs.

348
Q

The nurse is caring for a patient who will undergo a removal of a lung lobe. Which level of care will the patient require immediately post procedure?

a. Acute care—medical-surgical unit
b. Acute care—intensive care unit
c. Ambulatory surgery
d. Ambulatory surgery—extended stay

A

ANS: B
Patients undergoing extensive surgery and requiring anesthesia of long duration recover slowly. If a patient is undergoing major surgery such as a procedure on the lung, a stay in the hospital and specifically in the intensive care unit is required to monitor for potential risks to well-being. This patient would require more care than can be provided on a medical-surgical unit. It is not appropriate for this type of patient to go home after the procedure or to stay in an extended stay area of an ambulatory surgery area because of the complexity and associated risks.

349
Q

The nurse is caring for a group of patients. Which patient will the nurse see first?

a. A patient who had cataract surgery is coughing.
b. A patient who had vascular repair of the right leg is not doing right leg exercises.
c. A patient after knee surgery is wearing intermittent pneumatic compression devices and receiving heparin.
d. A patient after surgery has vital signs taken every 15 minutes twice, every 30 minutes twice, hourly for 2 hours then every 4 hours.

A

ANS: A
For patients who have had eye, intracranial, or spinal surgery, coughing may be contraindicated because of the potential increase in intraocular or intracranial pressure. The nurse will need to see this patient first to control the cough and intraocular pressure. All the rest are normal postoperative patients. Leg exercise should not be performed on the operative leg with vascular surgery. A patient after knee surgery should receive heparin and be wearing intermittent pneumatic compression devices; while the nurse will check on the patient, it does not have to be first. Monitoring vital signs after surgery is required and this is the standard schedule.

350
Q

The nurse demonstrates postoperative exercises for a patient. In which order will the nurse instruct the patient to perform the exercises?

  1. Turning
  2. Breathing
  3. Coughing4. Leg exercises

a. 4, 1, 2, 3
b. 1, 2, 3, 4
c. 2, 3, 4, 1
d. 3, 1, 4, 2

A

ANS: A

The sequence of exercises is leg exercises, turning, breathing, and coughing.

351
Q

The nurse is participating in a “time-out.” In which activities will the nurse be involved? (Select all that apply.)

a. Verify the correct site.
b. Verify the correct patient.
c. Verify the correct procedure.
d. Perform “time-out” after surgery.
e. Perform the actual marking of the operative site.

A

ANS: A, B, C
A time-out is performed just before starting the procedure for final verification of the correct patient, procedure, site, and any implants. The marking and time-out most commonly occur in the holding area, just before the patient enters the OR. The individual performing surgery and who is accountable for it must personally mark the site, and the patient must be involved if possible.

352
Q

The nurse is using a forced air warmer for a surgical patient preoperatively. Which goals is the nurse trying to achieve? (Select all that apply.)

a. Induce shivering.
b. Reduce blood loss.
c. Induce pressure ulcers.
d. Reduce cardiac arrests.
e. Reduce surgical site infection.

A

ANS: B, D, E
Evidence suggests that pre-warming for a minimum of 30 minutes may reduce the occurrence of hypothermia. Prevention of hypothermia (core temperature < 36° C) helps to reduce complications such as shivering, cardiac arrest, blood loss, SSI, pressure ulcers, and mortality.

353
Q

The nurse is caring for a postoperative patient with an incision. Which actions will the nurse take to decrease wound infections? (Select all that apply.)

a. Maintain normoglycemia.
b. Use a straight razor to remove hair.
c. Provide bath and linen change daily.
d. Perform first dressing change 2 days postoperatively.
e. Perform hand hygiene before and after contact with the patient.
f. Administer antibiotics within 60 minutes before surgical incision.

A

ANS: A, E
Performing hand hygiene before and after contact with the patient helps to decrease the number of microorganisms and break the chain of infection. Maintaining blood glucose levels at less than 150 mg/dL has resulted in decreased wound infection. Removing unwanted hair by clipping instead of shaving decreases the numbers of nicks and cuts caused by a razor and the potential for the introduction of microbes. The patient is postoperative; administration of an antibiotic 60 minutes before the surgical incision supports the defense against infection preoperatively. Providing a bath and linen change daily is positive but is not necessarily important for infection control. Many surgeons prefer to change surgical dressings the first time so they can inspect the incisional area, but this is done before 2 days postoperatively.

354
Q

The nurse is preparing for a patient who will be going to surgery. The nurse screens for risk factors that can increase a person’s risks in surgery. What risk factors are included in the nurse’s screening? (Select all that apply.)

a. Age
b. Race
c. Obesity
d. Nutrition
e. Pregnancy
f. Ambulatory surgery

A

ANS: A, C, D, E
Very young and old patients are at risk during surgery because of immature or declining physiological status. Normal tissue repair and resistance to infection depend on adequate nutrients. Obesity increases surgical risk by reducing respiratory and cardiac function. During pregnancy, the concern is for the mother and the developing fetus. Because all major systems of the mother are affected during pregnancy, risks for operative complications are increased. Race and ambulatory surgery are not risks associated with a surgical procedure.

355
Q

The nurse is providing preoperative education and reviews with the patient what it will be like to be in the surgical environment. Which points should the nurse include in the teaching session? (Select all that apply.)

a. The operative suite will be very dark.
b. The family is not allowed in the operating suite.
c. The operating table or bed will be comfortable and soft.
d. The nurses will be there to assist you through this process.
e. The surgical staff will be dressed in special clothing with hats and masks.

A

ANS: B, D, E
The surgical staff is dressed in special clothing, hats, and masks—all for infection control. Families are not allowed in the operating suite for several reasons, which include infection control and sterility. The nurse is there as the coordinator and patient advocate during a surgical procedure. The rooms are very bright so everyone can see, and the operating table is very uncomfortable for the patient.

356
Q

The operating room nurse is providing a hand-off report to the postanesthesia care unit (PACU) nurse. Which components will the operating room nurse include? (Select all that apply.)

a. IV fluids
b. Vital signs
c. Insurance data
d. Family location
e. Anesthesia provided
f. Estimated blood loss

A

ANS: A, B, E, F
The surgical teams report will include topics such as the type of anesthesia provided, vital sign trends, intraoperative medications, IV fluids, estimated blood and urine loss, and pertinent information about the surgical wound (e.g., dressings, tubes, drains). When the patient enters the PACU, the nurse and members of the surgical team discuss his or her status. A standardized approach or tool for hand-off communications assists in providing accurate information about a patient’s care, treatment and services, current condition, and any recent or anticipated changes. The hand-off is interactive, multidisciplinary, and done at the patient’s bedside, allowing for a communication exchange that gives caregivers the chance to dialogue and ask questions. Insurance data and family location are unnecessary.

357
Q

The nurse is caring for a group of postoperative patients on the surgical unit. Which patient assessments indicate the nurse needs to follow up? (Select all that apply.)

a. Patient with abdominal surgery has patent airway.
b. Patient with knee surgery has approximated incision.
c. Patient with femoral artery surgery has strong pedal pulse.
d. Patient with lung surgery has 20 mL/hr of urine output via catheter.
e. Patient with bladder surgery has bloody urine within the first 12 hours.
f. Patient with appendix surgery has thready pulse and blood pressure is 90/60.

A

ANS: D, F
Thready pulse, low blood pressure, and urine output of 20 mL/hr need to have follow-up by the nurse. Hemorrhage results in a fall in blood pressure; elevated heart and respiratory rates; thready pulse; cool, clammy, pale skin; and restlessness. Notify the surgeon if these changes occur. If the patient has a urinary catheter, there should be a continuous flow of urine of approximately 30 to 50 mL/hr in adults; this patient requires follow-up since the output is 20 mL/hr. All the rest are normal findings. A patent airway, a strong distal pulse, and approximated incision are all normal findings. Surgery involving portions of the urinary tract normally causes bloody urine for at least 12 to 24 hours, depending on the type of surgery.